Welcome to Seasources.net. Established in 1996, we are the ultimate source for online USCG exam preparation assistance. Seasources.net provides free online exam preparation along with downloadable studyguides and standalone software that allows you to continue your studies without an internet connection.

SeaSources.net, Training for Mariners. The First with Online Study for the Maritime Industry.

Click here to create an account in the online study and begin your license preparation.

All of the latest questions and illustrations on "G.M.D.S.S. Questions" are in the Online Study.

Home Page       Exam Question Downloads    Download TesTTaker Software    Free Sample Downloads   Member Comments      Contact SeaSources

 

We now have a whole new website.

coast-guard-exam.net

Click here to redirect.

 

Sample Global Maritime Distress and Safety System Questions

 

A What is the fundamental concept of the GMDSS? It is intended to automate and improve emergency communications in the maritime industry. It is intended to automate and improve existing digital selective calling procedures and techniques. It is intended to provide more effective but lower cost commercial communications. It is intended to provide compulsory vessels with a collision avoidance system when they are operating in waters that are also occupied by non-compulsory vessels.
C The primary purpose of the GMDSS is? Allow more effective control of SAR situations by vessels. Provide additional shipboard systems for more effective company communications. Automate and improve emergency communications for the world's shipping industry. Effective and inexpensive communications.
D What is the basic concept of GMDSS? Search and rescue authorities ashore can be alerted to a Distress situation. Shipping in the immediate vicinity of a ship in Distress will be rapidly alerted. Shoreside authorities and vessels can assist in a coordinated SAR operation with minimum delay. All of these.
D GMDSS is primarily a system based on? Ship-to-ship Distress communications using MF or HF radiotelephony. VHF digital selective calling from ship to shore. Distress, Urgency and safety communications carried out by the use of narrow-band direct printing telegraphy. The linking of search and rescue authorities ashore with shipping in the immediate vicinity of a ship in Distress or in need of assistance.
C What is the responsibility of vessels under GMDSS? Vessels over 300 gross tons may be required to render assistance if such assistance does not adversely affect their port schedule. Only that vessel, regardless of size, closest to a vessel in Distress, is required to render assistance. Every ship is able to perform those communications functions that are essential for the safety of the ship itself and of other ships. Vessels operating under GMDSS, outside of areas effectively serviced by shoreside authorities, operating in sea areas A2, and A4 may be required to render assistance in Distress situations.
C GMDSS is required for which of the following? All vessels capable of international voyages. Vessels operating outside of the range of VHF coastal radio stations. SOLAS Convention ships of 300 gross tonnage or more. Coastal vessels of less than 300 gross tons.
C What equipment is programmed to initiate transmission of Distress alerts and calls to individual stations? NAVTEX GPS DSC controller Scanning Watch Receiver
B What equipment is associated with the land or terrestrial systems? EPIRB VHF-MF-HF Inmarsat-C GPS
B What equipment is associated with the space systems? VHF-MF-HF Inmarsat-C NAVTEX SART
D What equipment is used in or near the survival craft? NAVTEX Fathometer COSPAS-SARSAT EPIRB
A What system provides accurate vessel position information to the GMDSS equipment? GPS COSPAS-SARSAT EPIRB Inmarsat-B
C What is the primary equipment for receiving MSI? SART EPIRB NAVTEX Inmarsat-B
D If a vessel is engaged in local trade and at no point in its voyage travels outside of the range of a VHF shore station with continuous DSC alerting then the vessel is operating in what area? Coastal and international zones Inland and coastal waters Sea areas A1 and A2 Sea area A1
B The Sea Area you are in is determined by: The vessel’s distance from shore. Whether the ship station is in range of a VHF-DSC, MF-DSC, HF-DSC Coast Station or Inmarsat coverage. The types of maintenance available to your vessel. None of the above
D If a vessel is on a voyage from Miami, Florida to Houston, Texas what Sea Areas may it transit through? Sea area A3 Sea area A2 or A3 if it is not within range of a VHF-DSC equipped coast station. Sea area A1 only if within range of a VHF-DSC equipped coast station. All of the above.
B For a vessel to be in GMDSS Sea Area A-1: The vessel must be within VHF range of a Public Correspondence Station. The vessel must be within range of a coast station equipped with continuous VHF-DSC capability. The vessel must be within VHF range of a U. S. C. G. communication station. The vessel must be within VHF range of either a U. S. C. G. or a Public Correspondence Station.
D A vessel is on a coastwise voyage that results in a distance off shore of 10 to 40 nm and therefore: The vessel may not be in Sea Area A-1 at all times. The vessel will always be in Sea Area A-1. For the vessel to be within Sea Area A-1 it must be within range of a coast station equipped with continuous VHF-DSC capability. Answers A and C may both be true.
A What is defined as the area within the radiotelephone coverage area of at least one VHF coast station in which continuous DSC alerting is available as defined by the IMO regulation for GMDSS? Sea Area A1 Ocean Area Regions AOR-E, AOR-W, POR or IOR Sea Area A2 Coastal and Inland Waters
D Which of the following is a functional or carriage requirement for compulsory vessels? A compulsory vessel must carry at least two (2) FCC licensed GMDSS Radio Operators. A compulsory vessel must satisfy certain equipment carriage requirements that are determined by where the vessel sails. A compulsory vessel must be able to transmit and respond to Distress alerts. All of the above.
B Which communications functions must all vessels be capable of performing under GMDSS as defined by the International Maritime Organization? Radio Direction Finding. Distress alerting to and from vessels, search and rescue coordination, on-scene communications, signals for locating, Maritime Safety Information, general and bridge-to-bridge communications. Communications in each of the operational ocean areas. All communications possible within the International Safety-Net service.
C GMDSS-equipped ships will be required to perform which of the following communications functions? Distress alerting and Maritime Safety Information. Search and Rescue coordination and on-scene communications. Bridge-to-bridge and general radio communications. All of these.
D What equipment can be used to receive Maritime Safety Information? NAVTEX EGC receiver HF NBDP All of the above.
A Which of the following is a required GMDSS function? Bridge-to-Bridge communications. Reception of weather map facsimile broadcasts. Both of the above None of the above
C Which of the following is a required GMDSS function? Transmit and receive locating signals. Transmit and receive general communications. Both of the above None of the above
A Which statement is true regarding a vessel equipped with GMDSS equipment that will remain in Sea Area A1 at all times? The vessel must be provided with a radio installation capable of initiating the transmission of ship-to-shore Distress alerting from the position from which the ship is normally navigated. VHF DSC alerting may be the sole means of Distress alerting. HF or MF DSC may satisfy the equipment requirement. HF SSB with 2182 kHz automatic alarm generator may satisfy the equipment requirement.
B What statement is true regarding the additional equipment carriage requirement imposed for the survival craft of vessels over 500 gross tons? Additional carriage of two radio equipped lifeboats aft. A second radar transponder is required. Four additional portable VHF radios are required. The ability to communicate in all modes with any shore station.
C All passenger vessels must have what additional equipment? Inmarsat-B terminal Inmarsat-C terminal Aircraft Transceiver with 121.5 MHz MF-HF SSB Transceiver
B Within a single sea area, what is the primary reason GMDSS imposes carriage requirements for different radio subsystems? Redundancy in duplicating all operational functions in the event of a system failure. Each subsystem has a specific purpose and capabilities that generally cannot be duplicated by other subsystems. Different radio systems may be used by the various authorities. The ability to communicate in all modes with any of the shore stations.
D If operating within Ocean Area A1, and outside of NAVTEX coverage, a GMDSS-equipped vessel must carry? An Inmarsat-B terminal A GPS receiver Equipment capable of maintaining a continuous DSC watch on 2187.5 kHz. Equipment capable of reception of Maritime Safety Information by the Inmarsat enhanced group call system, or HF NBDP.
D What is the equipment carriage requirement for survival craft under GMDSS? At least three approved two-way VHF radiotelephones on every passenger ship and cargo ships of 500 gross tons and upwards. At least two approved two-way VHF radiotelephones on every cargo ship between 300-500 gross tons. At least one radar transponder must be carried on every cargo ship of 300-500 gross tons and two transponders (one for each side) of every passenger ship and every cargo ship of 500 gross tons and upward. All of these.
C Which of the following statements concerning maintenance requirements is false? Compulsory vessels sailing in Sea Areas A1 and A2 must provide any one of the three maintenance options which are duplication of equipment, shore-based, or at-sea maintenance capability. Compulsory vessels sailing in Sea Areas A3 and A4 must provide any two of the three maintenance options which are duplication of equipment, shore-based, or at-sea maintenance capability. If shore-based maintenance is used, maintenance services do not have to be completed or performance verified unless the vessel will be sailing to a non-US port. Equipment warranties do not satisfy GMDSS maintenance requirements.
A Which of the following statements concerning maintenance requirements is true? The options are duplication of equipment, at-sea maintenance, and shore-based maintenance. Compulsory vessels between 300-500 gross tons are required only to provide one maintenance option, while compulsory vessels larger than 500 gross tons and all passenger vessels are required to provide any two of the three maintenance options. The "at-sea" maintenance may be waived if the compulsory vessel carries at least three licensed GMDSS Radio Operators. Compulsory vessels operating in Sea Area A4 are required to carry at least one licensed GMDSS Radio Maintainer.
D Which of the following is a requirement, under GMDSS, for all vessels over 300 gross tons operating within range of a MF-DSC equipped shore station? Ship's Master or radio officer must be on duty at all times. At least 2nd class Telegraphy license or GMDSS Element 9 is required for the radio officer. Spare parts and maintenance kit for repairs. Only one of the three maintenance options is required.
D What statement is generally correct regarding the maintenance requirements for ships under GMDSS? Redundancy of functions of certain equipment will partially meet this requirement. On-board maintenance provided by a person holding a GMDSS Maintainer's license will partially meet the requirements. Shoreside maintenance and scheduled tests and inspections will partially meet this requirement. All of the above.
B A ship operating in sea area A-1 must have the following provisions for maintenance Carry an on-board maintainer plus duplication of equipment. Only one option for maintenance is necessary. Shore maintenance may not be selected. None of the above
D A ship operating in sea area A-1 must have the following provisions for maintenance: Shore maintenance Duplication of equipment At Sea Maintenance Any one of the above.
B What is the frequency range for Medium Frequency? 30-300 kHz 300-3,000 kHz 1,000-10,000 kHz 10-30 MHz
A What is the frequency range for High Frequency? 3-30 MHz 300-3,000 kHz 30-300 MHz 10-30 MHz
C What is the frequency range for Very High Frequency? 3-30 MHz 300-3,000 kHz 30-300 MHz 10-30 MHz
B What is the frequency range for Ultra High Frequency? 3-30 MHz 300-3,000 MHz 30-300 MHz 10-30 MHz
D What is the frequency range for Super High Frequency? 30-300 GHz 300-3,000 MHz 30-300 MHz 3-30 GHz
C What is the frequency range for Maritime VHF operations? 3-30 MHz 88-108 MHz 156-164 MHz 540-1640 kHz
B How often must a compulsory vessel's GMDSS radio station be inspected? Annually, by the U.S. Coast Guard. Annually, by the FCC or designated authority. Annually, by the FCC, and every six months if the vessel sails outside of Sea Areas A1 and A2. The FCC's annual inspection may be waived if and only if monthly inspections are performed by the vessel's on-board GMDSS Radio Maintainer.
B What periodic inspection is required in order to remain in compliance with the regulations regarding GMDSS ship radio station inspections? U.S. Coast Guard annual inspection. An inspection at least once every 12 months by the FCC or a holder of a GMDSS Maintainers license. FCC inspection every five years. Periodic inspections not required if on board maintainers perform routine preventive maintenance.
B Which statement is false regarding a GMDSS-equipped ship? A conditional or partial exemption may be granted, in exceptional circumstances, for a single voyage outside the sea area for which the vessel is equipped. Ships must have the required equipment inspected at least once every five years. The regulations apply to all passenger ships regardless of size and cargo ships of 300 gross tons and upwards. Ships must carry at least two persons holding a GMDSS Radio Operator's license for Distress and Safety radio-communications purposes.
A Which statement is false regarding a GMDSS equipped ship? A conditional or partial exemption may not be granted, in exceptional circumstances, for a single voyage outside the sea area for which the ship is equipped. Ships must have the required equipment inspected at least once every 12 months. The regulations apply to all passenger ships regardless of size and cargo ships of 300 gross tons and upwards. Ships must carry at least two persons holding a GMDSS Radio Operator's license for Distress and Safety radio-communications purposes.
D During an annual FCC inspection __________. All required documents and publications might have to be produced. Licensed GMDSS operators may be required to demonstrate equipment competencies. All required equipment must be fully operational. All of the above.
C Foreign governments or administrations may inspect the radio installation: When the ship's station license cannot be produced without delay. When irregularities are observed. Both of the above None of the above
D Where can GMDSS regulations pertaining specifically to U.S.-flag vessels be found? These are located in CCIR #476. These are located in FCC Part 83. These are published only by the U.S. Coast Guard. These are located in 47 CFR Part 80.
B Which of the following references should the GMDSS Radio Operator consult for information on the proper operation of equipment? ITU List of Equipment Operations. The manufacturer's operator manuals. 47 CFR Part 80 Information is available through SafetyNETTM channels.
D What should the GMDSS Radio Operator consult to review the proper procedures to be followed in Distress situations under GMDSS? IMO Recommendations. The manufacturer's instruction manuals. Part 90 of the FCC Rules and Regulations. Part 80, Subpart W of the FCC Rules and Regulations.
D Which of the following documents are required by GMDSS for vessels on international voyages (other than the Great Lakes)? A copy of the IMO master plan of shore-based facilities. Station logs 47 CFR Part 80 FCC rules and regulations. All of these.
D Which of the following documents are required by Part 80 of the FCC rules for vessels on international voyages (other than the Great Lakes)? Appropriate Safety Convention Certificate. List of Call Signs and Numerical Identities. (ITU List VII-A) List of Radiodetermination and Special Service Stations. (ITU List VI) All of these.
B What publication/s should a GMDSS Operator consult regarding the proper set-up and operation of vessel equipment? ITU Publications The manufacturer's instruction manuals. Part 90 of the FCC Rules and Regulations. Code of Federal Regulations, Title 47, Part 80, Subpart W.
D Which of the following maintenance functions is not the responsibility of the GMDSS Radio Operator? Visual inspection of equipment, including the antenna and associated components. Perform on-the-air verification checks. Perform scheduled testing of the battery's charged condition. Aligning the power output stage for maximum power.
C When may a compulsory vessel not be allowed to leave port? When the vessel is in an over-carriage condition. When the vessel has arranged for both duplication of equipment AND shore-based maintenance. When the vessel has replaced a required piece of GMDSS-related equipment but its performance has not been verified or logged. When the vessel is carrying only two licensed GMDSS Radio Operators and is capable of performing all required functions.
A Which statement is false regarding the maintenance of GMDSS equipment at sea? The GMDSS maintainer may not be the person designated to have primary responsibility for radio-communications during Distress incidents even if licensed as an operator. Ships must carry at least one person who qualifies as a GMDSS maintainer for the maintenance and repair of equipment if the at-sea maintenance option is selected. All at-sea maintenance and repairs must be performed by, or under the supervision of a person holding a GMDSS Maintainer license. The GMDSS maintainer may be the person responsible for ensuring that the watches are properly maintained and that the proper guard channels and the vessel's position are entered into the DSC equipment.
C Which of the following service or maintenance functions may NOT be performed by the holder of a GMDSS Radio Operator License? Reset tripped circuit breakers or replace defective fuses. Routine battery maintenance if used as part of the GMDSS station. Any adjustments or maintenance that may affect the proper operation of the station. Replacement of consumable items such as paper, ribbons, etc.
A What are the conditions, under GMDSS, whereby a ship is NOT allowed to depart from any port? The vessel is not capable of performing all required Distress and Safety functions. The vessel is carrying more than the required number of qualified GMDSS radio operators. The vessel has a temporary waiver of its radio license and Safety Certificate. The vessel is not carrying a GMDSS radio maintainer, but has provided for shoreside maintenance plus duplication of equipment if required.
A What determines the spares and maintenance materials requirements for the VHF-DSC equipment under GMDSS? The recommendations of the manufacturer. 47 CFR Part 80 IMO Circular "Equipment Spares". The GMDSS Maintainer's requirements.
C Which of the following is the minimum license requirement of a GMDSS Radio Operator? Holding the Marine Radio Operator Permit is a pre-requisite before the GMDSS Radio Operator Endorsement can be obtained. Holding the General Radiotelephone Operator License with RADAR endorsement is sufficient. Holding a valid GMDSS Radio Operator license is sufficient. Holding either the General Radiotelephone Operator License or the First or Second Class Radiotelegraph license with GMDSS Radio Maintainer's endorsement is sufficient.
C Which of the following statements concerning GMDSS Radio Operator requirements is false? Each compulsory vessel must carry at least two licensed GMDSS Radio Operators at all times while at sea. Each compulsory vessel must carry at least two licensed GMDSS Radio Operators at all times while at sea and may elect to carry a GMDSS Radio Maintainer as well. Communications involving Safety of life at sea do not have to be logged as long as the compulsory vessel was not involved in such communications. While at sea, adjustments to, and the maintaining of, GMDSS equipment may be performed by the GMDSS Radio Operator as long as the work is supervised by an on-board licensed GMDSS Radio Maintainer.
D What is the minimum operator license required to perform or supervise the performance of at-sea adjustments, servicing or maintenance which may affect the proper operation of the GMDSS station? Marine Radio Operator Permit. General Radiotelephone Operator license and Radar endorsement. Designated maintainer possessing a GMDSS Radio Operator license. GMDSS Radio Maintainer's license.
C Which statement is false regarding the radio operator requirements for a GMDSS-equipped ship station? One of the qualified GMDSS radio operators must be designated to have primary responsibility for radio-communications during Distress incidents. A qualified GMDSS radio operator, and a qualified backup, must be designated to perform Distress, Urgency and Safety communications. Maintaining a record of all incidents connected with the radio-communications service that appear to be of importance to safety of life at sea is not required. While at sea, all adjustments or radio installations, servicing or maintenance of such installations that may affect the proper operation of the GMDSS station must be performed by, or under the supervision of, a qualified GMDSS radio maintainer.
D What are the vessel equipment and personnel requirements of GMDSS? Two licensed GMDSS radio operators. Equipment carriage requirements. Distress alerting and response. All of these.
C What is the minimum requirement of a GMDSS radio operator? Marine Radio Operator Permit and GMDSS Endorsement. General Radiotelephone Operator license and Radar endorsement. GMDSS Radio Operator license. General Radiotelephone license or First or Second Class Radiotelegraph license with GMDSS Radio Maintainer's endorsement.
A Under GMDSS, a compulsory VHF-DSC radiotelephone installation must be tested at what minimum intervals at sea? Daily Annually, by a representative of the FCC. At the annual SOLAS inspection. Monthly
A What is the meaning of “Reserve Source of Energy”? The supply of electrical energy sufficient to operate the radio installations for the purpose of conducting Distress and Safety communications in the event of failure of the ship's main and emergency sources of electrical power. High caloric value items for lifeboat, per SOLAS regulations. Diesel fuel stored for the purpose of operating the powered survival craft for a period equal to or exceeding the U.S.C.G. and SOLAS requirements. None of these
B In the event of failure of the main and emergency sources of electrical power, what is the term for the source required to supply the GMDSS console with power for conducting distress and other radio communications? Emergency power Reserve source of energy Ship's emergency diesel generator Ship's standby generator
D What is the requirement for emergency and reserve power in GMDSS radio installations? An emergency power source for radio communications is not required if a vessel has proper reserve power (batteries). A reserve power source is not required for radio communications. Only one of the above is required if a vessel is equipped with a second 406 EPIRB as a backup means of sending a Distress alert. All newly constructed ships under GMDSS must have both emergency and reserve power sources for radio communications.
A While underway, how frequently is the DSC controller required to be tested? Once a day Once a week Twice a week Once a month
B At sea, all required equipment (other than Survival Craft Equipment) must be proven operational by? Daily testing By either or Operational use of the equipment. Testing at least every 48 hours.
D Proper watchkeeping includes the following: All required frequencies are being monitored in the proper mode. After silencing an alarm all displays and/or printouts are read. Notifying the Master of any Distress alerts. All of the above.
D Proper watchkeeping includes the following: Understanding the GMDSS console's normal operational indicators. Maintaining a proper GMDSS radio station log. Responding to and comprehending alarms. All of the above.
D Which is true concerning a required watch on VHF Ch-16. It is compulsory at all times while at sea until further notice, unless the vessel is in a VTS system. When a vessel is in an A1 sea area and subject to the Bridge-to-Bridge act and in a VTS system, a watch is not required on Ch-16 provided the vessel monitors both Ch-13 and VTS channel. It is always compulsory in sea areas A2, A3 and A4. All of the above.
C Which of the following statements are true? GMDSS Radio Logs are required to contain entries pertaining to all incidents connected with the radio-communications service that appear to be of importance to the safety of life at sea. All Distress communications must be entered in the GMDSS radio log. Both of the above None of the above
D How long must the radio log be retained on board before sending it to the shoreside licensee? At least one year after the last entry. At least two years after the last entry. At least 90 days after the last entry. At least 30 days after the last entry.
C How long must the radio log be archived by the licensee? Two years if there is no Distress or Urgency entries. Three years if there are any Distress or Urgency entries. Both of the above None of the above
B What is the MID? Mobile Identification Number Maritime Identification Digits Marine Indemnity Directory Mobile Interference Digits
C How many digits are in the MID (Maritime Identification Digits)? 7 9 3 10
B What does the MID (Maritime Identification Digits) signify? Port of registry Nationality Gross tonnage Passenger vessel
C Which of the following numbers indicates a U.S. flag ship station? 36627934 243537672 338426791 3382315
B Which of the following MMSI numbers indicates a U.S. flag ship station? 430326890 303236824 33609991 257326819
B Which of the following numbers indicates a ship station MMSI? 3372694 623944326 30356328 3384672
C A DSC call is received from a station with a MMSI number of 003669991. Which of the following types of stations is it from? A vessel operating in Sea Area A3. A group ship station A U.S. coast station An Intercoastal vessel
B A valid MMSI number for a DSC call to a specific group of vessels is: 3664523 30327931 338462941 3036483
C MMSI 030346239 indicates what? Inmarsat-C I.D. number. Coast station. Group MMSI. Ship station.
B Which of the following statements concerning MMSI is true? Coast station MMSI numbers have 9 digits starting with 4. All MMSI numbers are 9 digits and contain an MID. Ship station MMSI numbers can be 7 digits or 9 digits depending on the Inmarsat terminal. Group MMSI numbers must begin with 2 zeros.
D Which of the following statements concerning MMSI is true? All Coast Station MMSI must begin with 2 zeros. The first 3 digits of a ship MMSI comprise the MID. A group call must begin with a single zero followed by the MID. All of the above.
C Which of the following statements concerning MMSI is true? All ship station MMSI must begin with a single zero and include the MID. All group station MMSI must begin with the MID. All Coast Station MMSI must be 9 digits and begin with 2 zeros followed by the MID. None of the above
B When making a routine DSC call to another vessel what information should be included? Time of transmission. Suggested working channel. Your own vessel’s position. Subject matter of the call.
D A “Distress Hot Key” Alert will include what information? The nature of distress. The vessel’s current position, course and speed from the GPS. Answers and are correct None of the above
B When sending a DSC call: Vessel’s position will automatically be sent. Vessel’s position will automatically be sent if the vessel is sending a “Distress Hot Key” alert. Vessel’s MMSI will indicate its ocean region. None of the above
D A "Distress Hot Key" VHF DSC Distress Alert: May go out on Ch-70 or Ch-16 depending on the manufacturer. Must go out on Ch-16 and Ch-70 to indicate “MAYDAY” traffic will follow. Must go out on Ch-16 to alert the nearest vessels and coast stations of imminent “MAYDAY” traffic. None of the above
C Which statement is true regarding vessel position when sending a “Distress Hot Key” Alert? The operator must choose to include the position. The vessel’s position will always be correct if taken from the connected GPS. The position will either be the current Lat/Long from the connected GPS or the updated manual position. All of the above.
B A “Distress Hot Key” Alert will always include what information? The vessel’s current position, course and speed from the GPS. Distress priority, Vessel’s position if available and the vessel’s MMSI number. The vessel’s MMSI number and category of distress. None of the above
C A DSC Distress Alert is received. What action should be taken? Silence the alarm Review the incoming message information. Both and Acknowledge by DSC
B Which of the following statements on DSC acknowledgement and relay of DSC Distress Alerts is true? Operators cannot initiate acknowledgements or relays. Some equipment was designed to allow for DSC acknowledgements and ALL SHIPS relays but under the most recent recommendations these options should not be used. The equipment was designed to allow for DSC acknowledgements and relays and the operator should follow the software menu structure accordingly. None of the above
C What does the DSC control unit do if the GMDSS Radio Operator fails to insert updated information when initiating a DSC Distress alert? It will abort the transmission and set off an audible alarm that must be manually reset. It will initiate the DSC Distress alert but, as no information will be transmitted, rescue personnel will not be able to identify the vessel, its position, or its situation. It will initiate the DSC Distress alert, and default information will automatically be transmitted. It will initiate the DSC Distress alert, but any station receiving it will have to establish contact with the Distressed vessel to determine its identity, position, and situation.
A A DSC Distress Alert is received. What action should be taken? Advise the Master. Transmit a DSC acknowledgement. Call the nearest Coast Guard Station. No action is necessary.
B What is the quickest method of transmitting a DSC distress alert? Make a “MAYDAY” call on Ch-70. Press the “Distress Hot Key”. Make a “MAYDAY” call on Ch-16. Select “Distress” priority from the menu.
C DSC relays of Distress Alerts by vessels: Should be done for all Distress Alerts. Should be transmitted to ships involved in Distress traffic. Should be avoided, however after repeated alerts, should be relayed to a Coast Station nearest the Distress incident. Are the best means to provide for a relay of Distress communications.
B A VHF-DSC “Distress Hot Key” Alert always transmits what information if connected to GPS? Distress designation and follow on communications channel. Position, Time of position update, MMSI number programmed and Distress format specifier. A “Nature of Distress” category. All of the above.
A Which of the following statements is true regarding distress alerting under GMDSS? The distress alert must identify the station in distress and its position and may additionally include information regarding the nature of the distress. Ship to shore distress alerts are used to alert other ships in port of navigational hazards. Ship-to-ship distress alerts are used to alert other ships in the vicinity of navigational hazards and bad weather. The vessel nearest to the emergency must notify the Coast Guard before leaving the vicinity.
C If a GMDSS Radio Operator initiates a DSC Distress transmission but does not insert a message, what happens? The transmission is aborted and an alarm sounds to indicate this data must be provided by the operator. The transmission is not initiated and "ERROR" is indicated on the display readout. The transmission will be made with "default" information provided automatically. The receiving station will poll the DSC unit of the vessel in Distress to download the necessary information.
C Repetition of a DSC Distress call is normally automatic if not acknowledged after a delay of: 1 - 2 minutes 2 - 5 minutes 3.5 - 4.5 minutes Not at all
D A VHF-DSC Distress alert call: Will send the minimal information using the "Distress Button" or "Distress Hot Key". Contains information on the vessel’s course and speed. Will send a more detailed Distress format if time permits and operator data entries are correctly performed. Both answers a) & c) are true
C A VHF-DSC Distress Alert will always be transmitted on what channel? Ch-16 Ch-22A Ch-70 Ch-6
D You send a VHF-DSC Distress alert. What channel do you use for the follow-on voice transmission? Ch-12 Ch-70 Ch-13 Ch-16
D Why should you always follow on with a voice transmission after sending a DSC Distress Alert? A voice follow on transmission is not necessary. To provide more information than is contained in the DSC message. To confirm for coast stations and other mariners that the distress is genuine. Answers b) and c).
C You receive a VHF-DSC Distress Alert. What Channel should you monitor for further information? Ch-06 Ch-22A Ch-16 Ch-70
D What is the proper format for a Distress follow on voice transmission? (3x is three times), All Ships 3x, this is Ship's Name/Call Sign 3x ,Mayday, Position. Mayday 3x, this is Ship's Name/Call Sign 3x, Distress category. Both of the above. None of the above.
D What information should be included in a Distress follow on voice transmission? Ship's Name and Call Sign. Ship's position. Ship's MMSI number. All of the above.
C What information should be included in a Distress follow on voice transmission? Follow on working frequency. ETA at next port. None of the above. Both of the above.
A Which statement is true regarding the receipt and acknowledgement of distress alerts by ship stations? A ship station that receives a distress alert must, as soon as possible, inform the Master or person responsible for the ship of the contents of the distress alert. Ship stations in receipt of a distress alert should acknowledge it immediately to assist the coast station in responding to the distress alert. Ship stations operating in areas where reliable communications with a coast station are not practicable, that receive a distress alert from a vessel in their vicinity, must either acknowledge by DSC or send a DSC relay to inform the Rescue Coordination Center. Alerts concerning navigational hazards are second only to safety traffic.
B What does the acronym “EOS" indicate in the received message? Error of Sequence End of Sequence End of Signals Equal Operating Signals
B What is the proper procedure to be followed upon receipt of a Distress alert transmitted by use of Digital Selective Calling techniques? Set watch on the DSC alerting frequency in the band of frequencies the alert was received. Set watch on the radiotelephone Distress and Safety frequency associated with the Distress and Safety calling frequency on which the Distress alert was received. Set a continuous watch on VHF-FM Channel 13, 16 and DSC on Channel 70. Ship stations equipped with narrow-band direct-printing equipment should respond to the Distress alert as soon as practicable by this means.
C What does the acronym "ECC" indicate in the received message? Every Cipher Counted Error Cannot Confirm Error Check Character Even Characters Counted
D What action should be taken on receipt of a distress alert? Read the display screen and/or printout. Silence the alarm. Listen for any follow on voice transmission on the appropriate frequency. All of the above.
B What precautions should be taken when reviewing an incoming Distress Alert message? If the message is not clear make a DSC call to all ships requesting clarification. Be careful not to activate a DSC acknowledgement or relay. No precautions are necessary. None of the above
D A DSC Distress Relay should always be made under the following circumstances: When the MMSI of the vessel in distress cannot be determined. After there is an acknowledgement from a coast station. Both of the above None of the above
D Your ship received a Distress relay on DSC VHF channel 70, on what channel would you reply? Ch-70 Ch-06 Ch-13 Ch-16
C Under what conditions would you relay a DSC Distress alert? If the mobile unit in Distress is incapable of further Distress Alert communications. If no Coast Station/Mobile Unit acknowledgement is observed. Answers and are both possible. You should never relay such an alert -- the Coast Station & RCC will do that.
D The relay of DSC Distress Alerts: Has completely overburdened the GMDSS system with improperly formatted or inappropriately relayed DSC calls. Was originally an intended function of the GMDSS system. Is no longer the preferred method for passing Distress message traffic to an RCC or Coast Station. All of the above.
D Transmission of a Distress alert by a station not in itself in Distress should occur: When the mobile unit actually in Distress is not itself in a position to transmit the Distress alert. When the Master or responsible person on the mobile unit not in Distress so decides. When the responsible person at the Coast Station determines further help is necessary. In some cases, all of the preceding situations may justify a Distress Alert relay.
C You are in voice communication on Ch-16 with a vessel in distress that advises you they are unable to contact a Coast Station. What action would you take? Send a DSC Distress Relay transmission. Make an all ships call with URGENCY priority. Attempt to contact a Coast Station using voice on Ch-16. If no response initiate a DSC call to the nearest Coast Station. None of the above
B What action should you take after sending a false Distress alert on VHF? Send a DSC cancellation message on Ch-70. Make a voice announcement to cancel the alert on Ch-16. Make a voice announcement to cancel the alert on Ch-13. Make a voice announcement to cancel the alert on Ch-22A.
A A crewmember has accidentally transmitted a VHF-DSC Distress Alert. What action should be taken? Turn off the power to the unit to stop the DSC call then turn power back on and make a voice announcement to cancel the alert on Ch-16. Send a DSC call canceling the Distress Alert. No specific action is necessary. Turn off the power and make a voice announcement to cancel the alert on Ch-70.
D What actions should be taken to prevent the transmissions of false distress alerts? Proper watch officer instruction in training. Insure that the protective cover over the “Distress Hot Key” is secure. Do not use DSC for relaying “ALL SHIPS” distress alerts. All of the above.
C The EPIRB on the bridge wing is observed with the strobe light flashing and the control switch in the “ON” position. What action should be taken? Disable the EPIRB Contact the nearest USCG Coast Station, give them your EPIRB’s I.D. number and advise that your unit was inadvertently activated. Both of the above None of the above
B You have been monitoring your 3-cm radar screen and a series of 12 concentric circles suddenly appears centered on the screen. What is the most likely cause of this situation? There is a survival craft within 3 nm distance. Your own vessel’s SART has been activated. There is a fault in the radar. None of the above
D The EPIRB on the bridge wing is observed with the strobe light flashing and the control switch in the “OFF” position. What action should be taken? Place the control switch in the ‘OFF” position. Assume the unit is transmitting and disable the EPIRB. Contact the nearest USCG Coast Station, give them your EPIRB’s I.D. number and advise that your unit was inadvertently activated. Both B and C are correct
D What is the fundamental purpose for imposing radio silence? To ensure that interference to proprietary communications is minimized. To ensure that only voice communications can be effected on the Distress frequency or channel. To ensure that a Distressed vessel will have a "window" twice each hour for transmitting routine messages. To ensure that interference on a particular frequency or channel for communications concerning emergency traffic is minimized.
C When can routine communications be resumed when radio silence has been imposed? After determining that the frequency or channel appears to be no longer in use. After determining that geographic distance from the Distress situation will prohibit any other signal from interfering with emergency communications. Routine communications can resume after the Rescue Coordination Center transmits a message on the frequency or channel being used for emergency communications stating that such traffic has concluded. If, in the master's opinion, communications on that frequency will interfere with emergency communications.
A What is meant by the term "radio silence"? Stations not directly involved with the on-going Distress communications may not transmit on the Distress frequency or channel. Stations remaining off the air to safeguard proprietary information. Two three-minute silent periods, at 15 and 45 minutes after the hour, that provide a transmitting "window" for Distressed vessels to transmit Distress alerts using J3E. Communications on a Distress frequency or channel is banned for 24 hours following the cessation of the Distress traffic.
A How is "radio silence" imposed? By the On Scene Coordinator (OSC). By the Coast Earth Station (CES) controlling the Distress communications on that frequency. By the nearest Public Correspondence Coast Station. By the vessel first responding to the Distress call.
B What is the reason for imposing radio silence? To keep a clear channel open for safety broadcasts. To prevent interference to distress communications. To allow individual vessels to carry out direct communications. All of the above.
A How are normal working conditions restored after radio silence has been imposed? The Rescue Coordination Center (RCC) that imposed the radio silence must transmit a voice message on the Distress frequency stating "SEELONCE FEENEE". The Coast Earth Station (CES) that imposed the radio silence must transmit a voice message on the Distress frequency stating "SILENCE FINI". The Public Correspondence Station (PCS) that imposed the radio silence must transmit a voice message on the Distress frequency stating "SILENCE FINI". None of the above
B The Radiotelephone Urgency signal is: Mayday Pan Pan Securite Seelonce Feenee
D Which of the following situations would normally use the Urgency priority? A crewmember over the side. A serious medical situation involving a crewmember. A cargo shift or weather situation considered to be of greater hazard than would justify a Safety priority designation. Answers b) and c).
B Which of the following situations would normally use the Urgency priority? Leaking oil from a minor tank fracture. Treatment of crewmember breaking a leg in a cargo hold. A fire in the generator flat/spaces. Answers and are both possible.
B Which of the following situations would normally use the Urgency priority? A crewmember over the side. A serious medical situation involving a crewmember. Both a) and b) Scenarios concerning the Safety of navigation or important meteorological warnings.
C The Urgency Priority should be used for: Messages concerning the Safety of Life At Sea (SOLAS). Messages detailing important navigational warnings. Messages containing information concerning the safety of a mobile unit or person. Messages concerning On-scene communications .
B If the Watch Officer hears "PAN PAN" spoken 3 times it means: A navigation or important meteorological warning should follow. The station is preparing to transmit an Urgency message possibly concerning the safety of a mobile unit or person. A mobile unit is in need of immediate assistance. None of the above
A When the GMDSS Radio Operator on watch hears "SECURITE" spoken three times, he can expect to receive the following information: Message concerning the safety of navigation or important meteorological warnings. Safety of vessel or person is in jeopardy. Vessel in need of immediate assistance. Coast Station Traffic list.
B Which of the following situations would normally use the Voice designation "Securite"? Messages concerning the Safety of Life At Sea (SOLAS). Messages detailing important navigational warnings. Messages containing information concerning the safety of a mobile unit or person. Messages concerning On-scene communications .
A Which of the following situations would normally use the Safety priority? Loss of 5 containers with lashing gear over the side. Treatment of crewmember breaking a leg in a cargo hold. A fire in the generator flat/spaces. Answers and are both possible.
D Which of the following situations would normally use the Safety priority? A crewmember over the side. A serious medical situation involving a crewmember. Both a) and b) Scenarios concerning the Safety of navigation or important meteorological warnings.
A The Radiotelephone Safety signal is: "Securite" repeated 3 times. "Safety Safety Safety". "Pan Pan" repeated 3 times. "Securite Securite" repeated 3 times.
C Which of the following situations would normally use the Safety priority? A crewmember over the side. A serious medical situation involving a crewmember. A scenario concerning an important navigational or meteorological warning. All of the above.
C Which of the following steps should be taken, if possible, when the vessel must be abandoned because of a Distress situation? Alert the U.S. Coast Guard by using the survival craft's portable INMARSAT unit. Program the SART and EPIRB to transmit the vessel's location and situation. Place the SART and EPIRB in the "on" position and secure them to the survival craft. No additional steps are needed as the SART and EPIRB will both automatically float free and operate properly.
D If your vessel has suffered a casualty severe enough to disable both ship’s power and the GMDSS console RSE you should: Use UHF Transceivers to contact other vessels. Activate the EPIRB. Make a “Mayday” call on Ch-16 using the Survival Craft Portable Transceiver. Answers b) and c).
C DSC is used primarily to: Receive weather warnings, navigational notices and other marine safety information. Provide routine communications with the ship owner. Transmit and receive Distress, Urgency and Safety alerts to and from other ships and shore stations via radio. Report ship's position to search-and-rescue authorities via satellite.
B The vessel’s GMDSS operator fails to properly record the particulars of an incoming DSC distress alert. Which statement is true? The details of the DSC alert are obtainable from the DSC address book. The details of the DSC alert should be obtainable by accessing the DSC call data directory. The details of the DSC distress alert are irrevocably lost. None of the above
D What action should be taken when abandoning ship? Send a VHF-DSC Distress Alert. Activate the EPIRB. Take EPIRB, SARTs and Survival Craft Portable Transceivers to the survival craft. All of the above.
B What is the best method of determining whether a distress situation is genuine? Check your 3-cm radar for a SART signal from the distress vessel. Monitor the follow on frequency for actual voice distress communications. Monitor the 406 MHz EPIRB signal to locate the vessel in distress. Check the NAVTEX for U.S.C.G. confirmation of the distress from the RCC.
A What is the purpose of the SART's audible tone alarm? It informs survivors that assistance may be nearby. It informs survivors when the battery's charge condition has weakened. It informs survivors when the SART switches to the "standby" mode. It informs survivors that a nearby vessel is signaling on DSC.
A What indication is given to the personnel of survival craft of the approach of another vessel? The SART will provide a visual or audible indication of interrogation by a 3-cm radar. The Satellite EPIRB will emit an audible signal. The VHF portable radio will emit an audible alarm signal on Ch-70. The VHF portable will provide a visual indication.
B How can a SART's effective range be maximized? The SART should be placed in water immediately upon activation. The SART should be held as high as possible. Switch the SART into the "high" power position. If possible, the SART should be mounted horizontally so that its signal matches that of the searching radar signal.
B In a lifeboat or liferaft, what is a method of maximizing the effectiveness of an SART? Place the SART into the sea as soon as possible to begin transmitting. Hold or mount the unit as high as possible. Extend the length of the transmitting antenna. Replace the internal battery with the AC power adapter.
C At what point does a SART begin transmitting? It immediately begins radiating when placed in the "on" position. It must be manually activated. If it has been placed in the "on" position, it will respond when it has been interrogated by a 9-GHz radar signal. If it has been placed in the "on" position, it will begin transmitting immediately upon detecting that it is in water.
C What causes the SART to begin a transmission? When activated manually, it begins radiating immediately. It is either manually or water activated before radiating. After being activated the SART responds to radar interrogation. It begins radiating only when keyed by the operator.
B How does the searching vessel's radar interrogate a survival craft SART? Activate the IFF interrogation system. The SART responds automatically when it detects the search craft or other vessels' X-Band radar signal. Maintain watch on VHF-FM Ch-70 for the SART's unique identifier. The SART responds automatically when it detects the search craft or other vessel's 3.5 GHz radar signal.
B What does a SART signal sound or look like? It transmits "SOS" and the vessel's name and position in slow speed Morse Code. It will appear on a radar unit's PPI as a line of dots radiating outward, with the innermost dot indicating the SART's position. It will appear on a radar unit's PPI as a line of dots radiating outward, with the outermost dot indicating the SART's position. None of the above
A How can rescue personnel detect that a SART is transmitting in the immediate vicinity? The SART's dots on the PPI will become arcs and then eventually become concentric circles. The DSC unit will react to the SART's signal and respond with the two-tone auto alarm. The SART can provide an approximate location to within a two nautical mile radius, per IMO standards. The SART signal appears as a target which comes and goes; the effect of heavy swells on a SART.
B What signal is detected as originating from an SART? The Morse code Distress series S-O-S repeated three times followed by DE and the vessel's call sign. A line of dots on a radar screen outward from the SART's position along its line of bearing. A line of dots on a radar screen inward from the SART's position to its own ship along its line of bearing. None of these
A How can vessel personnel detect the operation of a SART in its vicinity? A unique radar signal consisting of a 12 dots radiating outward from a SART's position along its line of bearing. A unique two-tone "warbling" signal heard on VHF-FM Ch-70. A unique two-tone alarm signal heard upon the automatic un-muting of the 2182 kHz radiotelephone automatic watch receiver. The SART signal appears as a target which comes and goes; the effect of heavy swells on an SART.
C How should the signal from a Search and Rescue Radar Transponder appear on a radar display? A series of dashes. A series of spirals all originating from the range and bearing of the SART. A series of 12 equally spaced dots. A series of twenty dashes.
B In which frequency band does a search and rescue transponder operate? 3 GHz 9 GHz S-band 406 MHz
C Which of the following would most likely prevent a SART's signal from being detected? Signal absorption by the ionosphere. Heavy sea swells. The rescue personnel were monitoring the 10-CM radar. The rescue personnel were monitoring the 3-CM radar.
C Which statement is NOT true regarding the SART? Responds to interrogations by a vessel's X-Band radar. Transmits on the 9 GHz band reserved for navigational radar. Operates in conjunction with a vessel's S-Band radar. Transmits a distinctive code for easy recognition.
B Which statement is true regarding the SART? This is a performance monitor attached to at least one S-band navigational radar system. This is a 9 GHz transponder capable of being received by vessel's X-band navigational radar system. This is a 9 GHz transponder capable of being received by another vessel's S-band navigational radar system. This is a performance monitor attached to at least one X-band navigational radar system.
B Which statement is NOT true regarding the SART? Responds to interrogations by a vessel's X-Band radar. This is a 6 GHz transponder capable of being received by a vessel's X-band navigational radar system. This is a 9 GHz transponder capable of being received by a vessel's X-band navigational radar system. Transmits a distinctive signal for easy recognition.
C A SART's signal cannot be detected: In poor visibility, or at night. In heavy seas By a search vessel's 10 cm Radar. By a search vessel's 3 cm Radar.
B Which of the following statements concerning testing and maintenance of SARTs is true? An at-sea GMDSS maintainer is not able to test a SART as it is hermetically sealed. Testing a SART should be performed only in controlled environment as a test signal may be misinterpreted as a genuine Distress situation. A SART's battery must be replaced within ninety (90) days after the expiration date imprinted on the unit. All of the above.
B Why is it important to limit the duration of testing a SART? Excessive testing causes "burn in" on the vessel's radar PPI. Testing a SART should be performed only in a controlled environment, as a test signal may be misinterpreted as a genuine Distress situation. To prevent overheating, a SART requires sufficient ventilation that is significantly reduced when the SART is being tested. If another SART is testing at the same time, the two signals will cause damage to the unit that transmitted them.
D What statement is true regarding tests and maintenance that could be provided for the SART? To fully verify operation within manufacturer's specifications would require measuring equipment to generate 9 GHz signals; generally beyond the scope of on-board maintenance. Extreme care should be exercised because testing of the SART may be received by other vessels, and may be interpreted as a Distress condition, or it may interfere with other vessels' safe navigation. Battery should be replaced with a new one before the manufacturer's expiration date shown on the SART. All of these.
D Why should functional testing of a SART be minimized? Potential interference with safe navigation. Minimize power consumption of the battery. Possibility of misinterpretation by other vessels as a Distress situation. All of these.
D Which is not a valid maintenance and testing function for a SART? Operational test with several vessels to determine effective transmitting range. Inspection of container for apparent damage. Inspect battery expiration date and the lanyard condition. Brief operational test utilizing own ship's radar.
A The SART is required to have sufficient battery capacity to operate in the stand-by mode for what period of time? Eight hours Three days Four days Forty-eight hours
D Which is a function of a satellite under COSPAS-SARSAT using satellite EPIRBs? Relayed satellite message includes the EPIRB ID number which provides a reference for retrieval of vessel information from the shore database. Doppler shift of EPIRB signal is measured. Information received from EPIRBs are time-tagged and transmitted to any Local User Terminal in the satellite's view. All of these.
A Which of the following satellite systems is of particular importance to search and rescue missions under GMDSS? COSPAS/SARSAT AMSAT NASA/Arienne COMSAT
D Which of the following statements concerning COSPAS-SARSAT is true? EPIRBs are units that are used as alerting devices. These are satellites in a low-earth polar orbit that detect EPIRB beacons on 406 MHz and relay the information to a Local User Terminal (LUT). The Doppler frequency measurement concept is used to determine the EPIRB's location. All of the above.
D Which of the following statements concerning COSPAS-SARSAT is false? EPIRBs are used primarily for Distress alerting. These satellites are looking for EPIRB signals on 406 MHz. These satellites use Doppler shift measurement to determine the location of the beacons. After initiating a call request and selecting the CES, these satellites may be used for commercial messages.
C Which of the following statements concerning the EPIRB system is true? GOES weather satellites will provide alerting with worldwide coverage. The COSPAS-SARSAT system always provides an alert and position report within 5-20 minutes of reception. The Inmarsat system will not provide alerts and position report for 406 Mhz EPIRBs equipped with GPS receivers. The GPS satellite system will provide an alert and position report within 20 minutes of reception.
A Which of the following statements concerning satellite EPIRBs is true? Once activated, these EPIRBs transmit a signal for use in identifying the vessel and for determining the position of the beacon. The coded signal identifies the nature of the Distress situation. The coded signal only identifies the vessel's name and port of registry. If the GMDSS Radio Operator does not program the EPIRB, it will transmit default information such as the follow-on communications frequency and mode.
D What feature(s) may be found on certain satellite EPIRB units? Strobe light Emergency transmission on 406 MHz. Float-free release bracket. All of these.
B What feature is not found on 406 MHz satellite EPIRB units? 121.5 MHz emergency homing transmitter. Aural locator signal. Emergency transmission on 406.025 MHz. Float-free release bracket.
C What statement is true regarding 406 MHz EPIRB transmissions? Allows immediate voice communications with the RCC. Coding permits the SAR authorities to know if manually or automatically activated. Transmits a unique hexadecimal identification number. Radio Operator programs an I.D. into the SART immediately prior to activation.
D Which of the following is normally found on EPIRBs that are detected by satellites? A strobe light A 5-watt 406-MHz beacon. A bracket designed to allow the EPIRB to automatically float-free. All of the above.
C Which of the following statements concerning EPIRB alerts is false? The COSPAS-SARSAT system may take a full hour or more to provide an alert. The GOES weather satellites are in a geostationary orbit. The Inmarsat system provides worldwide coverage for Distress alerts. 406 MHz EPIRB units may be equipped with GPS receivers.
B Which of the following EPIRBs is most likely to be used to transmit a Distress alert signal? S-Band EPIRBs 406 MHz EPIRBs Class A EPIRBs 121.5/243 MHz EPIRBs
B Which of the following would best be used for visual detection of a distressed vessel? A 9-GHz SART's beacon. An EPIRB's strobe light. A 121.5-MHz EPIRB beacon. A 406-MHz EPIRB beacon.
B Which piece of required GMDSS equipment is the primary source of transmitting locating signals? Radio Direction Finder (RDF). An EPIRB transmitting on 406 MHz. Survival Craft Transceiver. A SART transmitting on 406 MHz.
C What may be used as a homing signal by the search and rescue vessels in the immediate vicinity of the ship in Distress? Flare gun Strobe Light A 121.5 MHz emergency transmitter in a satellite EPIRB. 406 MHz signal from a satellite EPIRB.
B What part of a satellite EPIRB may function as a visual aid to rescue vessels? A 121.5 MHz emergency transmitter in a satellite EPIRB. Strobe light 406 MHz signal from a satellite EPIRB. Loud beeping tone emitted by the unit, once activated.
D What is an example of a locating signal? SSB phone traffic Ship to shore transmissions Loran C A float-free EPIRB
B Which device provides the main means in the GMDSS for locating ships in distress or their survival craft? Radio Direction Finder Satellite EPIRBs MF/HF DSC VHF homing device
D With what other stations may portable survival craft transceivers communicate? Communication is permitted between survival craft. Communication is permitted between survival craft and ship. Communication is permitted between survival craft and rescue unit. All of the above.
A Equipment for radiotelephony use in survival craft stations under GMDSS must have what capability? Operation on Ch-16. Operation on 457.525 MHz. Operation on 121.5 MHz. Any one of these.
D Equipment for radiotelephony use in survival craft stations under GMDSS must have what characteristic(s)? Operation on Ch-16 Watertight Permanently-affixed antenna All of these.
B Which statement is NOT true regarding the requirements of survival craft portable two-way VHF radiotelephone equipment? Operation on Ch-16 Effective radiated power should be a minimum of 2.0 Watts. Simplex (single frequency) voice communications only. All of these.
A Which statement is NOT true regarding the requirements of survival craft portable two-way VHF radiotelephone equipment? Operation on Ch-13 Effective radiated power should be a minimum of 0.25 Watts. Simplex (single frequency) voice communications only. Operation on Ch-16
A Which statement is NOT true regarding the requirements of survival craft portable two-way VHF radiotelephone equipment? Operates simplex on Ch-70 and at least one other channel. Watertight to a depth of 1 meter for 5 minutes. Effective radiated power should be a minimum of 0.25 Watts. The antenna is fixed and non-removable.
A Which action should the GMDSS radio operator take in a Distress situation when embarking in survival craft? Switch on EPIRB and SART immediately and leave on. EPIRB and SART switched on manually prior to embarking; remain aboard vessel in Distress. Notify RCC (Rescue Coordination Center) through VHF DSC in portable equipment. Communicate via Inmarsat-C from the survival craft.
A Which is the key part of the search and rescue system under GMDSS? COSPAS/SARSAT satellites AMSAT satellites NASA satellites U.S. Space Agency satellites
D Which statement is true regarding the COSPAS-SARSAT system? EPIRBs are satellite beacons used aboard vessels as alerting devices. Signals received by low altitude, near-polar orbiting satellites are relayed to a ground receiving station, called a Local User Terminal. Doppler shift is used to locate the beacons. All of these.
D Which statement is NOT true regarding the COSPAS-SARSAT system? EPIRBs are satellite beacons used as alerting/locating devices. Locates Distress beacons transmitting on 406 MHz. Doppler shift is used to locate the beacons. May be used to transmit public correspondence.
B What information is transmitted by a 406 MHz EPIRB alert? Vessel position and nature of Distress. A unique Hexadecimal I.D. number. Vessel name and identification. None of the above
D Which statement is true regarding the COSPAS-SARSAT system and EPIRB operations? The EPIRB’s position is calculated by the system and passed to the RCC. The EPIRB transmits a unique Hex I.D. and vessel position that is passed to the RCC. The EPIRB transmits a unique Hex I.D. that is passed to the RCC. Both A and C are true
D The “On Scene Coordinator” may be which of the following? The vessel in distress. The first search vessel to arrive on the scene. Any involved vessel so designated by the Search and Rescue Mission Coordinator. May be either or depending on the circumstances.
C Which of the following channels is designated as the VHF follow-on communications channel and is required in all portable survival craft equipment? Ch-6 Ch-13 Ch-16 Ch-70
B The determination that the Distress traffic is over is usually made by whom? The vessel in distress. The “On Scene Coordinator” and/or the RCC controlling the distress traffic. The first Coast Station to receive the DSC Distress Alert. None of the above
C On Scene communications are usually initiated using what equipment: EPIRB on 121.5 MHz SART on 9 GHz VHF on Ch-16 VHF on Ch-70
C On Scene communications should be conducted on which of the following channels? 406 MHz VHF Ch-22A VHF Ch-16/06 VHF Ch-70
B On Scene communications should be conducted on which of the following channels? 9 GHz VHF Ch-16/06 VHF Ch-13 VHF Ch-26
D How is mutual interference among NAVTEX stations avoided? Stations are limited to daytime operation only. Transmitter power is limited to that necessary for coverage of assigned area. Transmissions by stations in each NAVAREA are arranged in a time-sharing basis. Both b) and c).
B When do NAVTEX broadcasts typically achieve maximum transmitting range? Local noontime Middle of the night Sunset Post sunrise
A What should a GMDSS Radio Operator do if a NAVTEX warning message is received but it contains too many errors to be usable? Do nothing. Vital NAVTEX messages will be repeated on the next scheduled broadcast. Contact the NAVAREA coordinator and request a repeat broadcast. The hurricane will be upon the vessel; they're in big trouble. Listen to appropriate VHF weather channel for repeat warnings.
B What does a NAVTEX receiver do when it runs out of paper? The unit cannot operate, and all subsequent MSI broadcasts are missed until the paper is replaced. It will give off either an audible and/or visual alarm. The system will automatically change from receiving MSI by NAVTEX to receiving it by SafetyNETTM so that no messages will be lost. All of the above.
A Which of the following is the primary frequency that is used exclusively for NAVTEX broadcasts internationally? 518 kHz 2187.5 kHz 4209.5 kHz VHF channel 16 when the vessel is sailing in Sea Area A1, and 2187.5 kHz when in Sea Area A2.
D What is the transmitting range of most NAVTEX stations? Typically 50-100 nautical miles (90-180 km) from shore. Typically upwards of 1000 nautical miles (1800 km) during the daytime. It is limited to line-of-sight or about 30 nautical miles (54 km). Typically 200-400 nautical miles (360-720 km).
B How is a NAVTEX receiver programmed to reject certain messages? The transmitting station's two-digit identification can be entered to de-select reception of its broadcasts. By choosing a message category's single letter (A-Z) identifier and then deselecting or deactivating. By entering the SELCALL of the transmitting station. By pressing "00" in the transmitter's ID block.
B How can reception of certain NAVTEX broadcasts be prevented? Stations are limited to daytime operation only. The receiver can be programmed to reject certain stations and message categories. Coordinating reception with published broadcast schedules. Automatic receiver desensitization during night hours.
B Which of the following statements is true? The GMDSS Radio Operator can program the NAVTEX receiver to automatically reject any category of messages. The GMDSS Radio Operator can program the NAVTEX receiver to reject all messages except navigation warnings, meteorological warnings, and search and rescue information. The GMDSS Radio Operator can select the "None" option in the message category menu. Upon entering a new NAVTEX station's broadcast range, the GMDSS Radio Operator enters the station's SELCALL number.
B What means are used to prevent the reception of unwanted broadcasts by vessels utilizing the NAVTEX system? Operating the receiver only during daytime hours. Programming the receiver to reject unwanted broadcasts. Coordinating reception with published broadcast schedules. Automatic receiver de-sensitization during night hours.
B What statement is true regarding the control the operator can exercise over the NAVTEX receiver's operation? The operator can set the unit to automatically reject any and all categories of messages if the ship desires to not receive them. The operator can set the unit to reject all messages except navigation, weather and sea warnings, and search and rescue messages. To reduce the number of messages, the operator can select code 00 to indicate "not in coastal passage". Upon entering a coastal area for the first time, the operator enters code KK to indicate "ready to receive NAVTEX".
C Which message subject matter can be programmed to be rejected or disabled by the operator of a NAVTEX receiver? Navigational warnings Meteorological warnings Pilot Service Messages All of these.
A The NAVTEX message header contains the following? A single letter (A-Z) indicates the NAVTEX transmitting station. A two-digit number (01-99) indicates the NAVTEX message category. Message numbers include a date/time group, along with the transmitting station's numerical ID. None of these
D Which of the following message categories cannot be disabled by the GMDSS Radio Operator? Navigational warnings Meteorological warnings Search and Rescue information All of the above.
A How are NAVTEX broadcasts transmitted? Using FEC techniques. NAVTEX is transmitted by commercial coast radio stations following their traffic lists. NAVTEX is transmitted only when an Urgency or Distress broadcast is warranted. No more often than every two hours and should immediately follow the radiotelephone silent periods.
D Which determines whether a NAVTEX receiver does not print a particular type of message content? The serial number and type of message have already been received. The subject indicator matches that programmed for rejection by the operator. The transmitting station ID covering your area has not been programmed for rejection by the operator. Both answers a) and b).
B Which information determines if a NAVTEX message is to be rejected? Transmitter identity (numerals from 1 to 26 identifying transmitting station within the NAVAREA). Subject indicator (single letter from A to Z indicating the type of message). The Answerback of the receiving station has not been entered in the NAVTEX receiver. Only messages having a serial number 00 are rejected.
A NAVTEX broadcasts are sent: In categories of messages indicated by a single letter or identifier. Immediately following traffic lists. On request of maritime mobile stations. Regularly, after the radiotelephone silent periods.
A Where NAVTEX cannot be feasibly established, what system can be implemented to provide an automated service in coastal waters to receive MSI? SafetyNETTM AMVER VHF DSC ARQ SITOR
B What action should a GMDSS Radio Operator take when SafetyNET™ Distress or Urgency messages are received by the vessel's EGC receiver? No immediate action is required, as an audible tone will be generated at the beginning and end of the transmission and a paper printout of the message will be generated. Aural and visual alarms are activated, and require manual deactivation. No immediate action is required by the operator, since the transmission will be automatically acknowledged by the receiving vessel. A periodic alarm tone will be heard until the radio operator prints the message from the unit's memory.
A What system can provide an automated service in coastal waters where it may not be feasible to establish the NAVTEX service or where shipping density is too low to warrant its implementation? SafetyNETTM AMVER VHF DSC ARQ SITOR
C Which equipment can receive SafetyNET™ messages? VHF DSC NAVTEX Receiver Dedicated EGC receiver All of these.
C SafetyNET™ messages can be received by which of the following shipboard equipment? NAVTEX MF and HF NBDP EGC receiver All of these.
B Marine Safety Information is promulgated via satellite through which system? AMVER SafetyNETTM NAVTEX Inmarsat-M SES
A SafetyNET™ promulgates what type of information? MSI Traffic Lists News advisories MARAD
B What kind(s) of broadcasts are not available through SafetyNET™? MSI and messages to specific geographic areas. Vessel traffic lists Storm warnings Distress and Urgency bulletins
B Which satellite system promulgates Maritime Safety Information? AMVER Inmarsat-C SafetyNETTM NAVTEX Inmarsat-M SES
A What information is promulgated by the international SafetyNET™? MSI Traffic Lists Priority Messages MARAD
B A vessel using SafetyNET™: Notify the NAVAREA coordinator you are using SafetyNET™ for MSI (Maritime Safety Information). Set the receiver to your present NAVAREA. Set the receiver to your destination Ocean Region. Notify the NAVAREA coordinator you are using SafetyNETTM for MSI (Maritime Safety Information) and set the receiver to your destination Ocean Region.
C In using SafetyNET for MSI (Maritime safety information): In an area where Ocean Region coverage patterns overlap, the scheduled MSI broadcasts you receive will be determined by the NCS you are currently monitoring. You will always receive all unscheduled broadcasts regardless of which NCS you are monitoring. Both of the above None of the above
D Over what system are Enhanced Group Calls transmitted? COSPAS satellite HF SITOR shore stations NAVTEX shore stations Inmarsat satellite
D What is the purpose of the dedicated EGC receiver for A-1 area GMDSS Vessels? To allow monitoring of the vessels location for AMVER tracking. Simultaneous receipt and transmission of vessel SafetyNETTM messages. To track which NAVAREA the vessel is currently in for receipt of MSI. To insure receipt of MSI in areas without NAVTEX coverage.
B Which of the following provides a unique automated system capable of addressing messages to pre-determined groups of ships or all vessels in both fixed and variable geographic areas? NAVTEX EGC AFRTS NAVAREAs
B What system may be useful for messages, such as local storm warnings or a shore-to-ship Distress alert, for which it is inappropriate to alert all ships in the satellite coverage area? NAVTEX EGC AMVER DSC
A What services are available through Enhanced Group Calls? Maritime Safety Information and messages to pre-defined groups of subscribers. Maritime Safety Information and vessel traffic lists. Hourly NOAA weather broadcasts from the NWS. Coastal weather broadcasts.
A What messages originate from registered information providers anywhere in the world and are broadcast to the appropriate ocean region via a CES? SafetyNETTM messages AMVER broadcasts Urgency messages NAVTEX broadcasts
B Adjusting the volume control has the following results: The higher the volume control is set the greater the sensitivity. Adjusting the volume control has no effect on the sensitivity. The volume control sets the threshold for receiving signals. None of the above
C The Dual Watch (DW) function is used to: Listen to Ch-70 at the same time while monitoring Ch-16. Sequentially monitor 4 different channels. Listening on any selected channel while periodically monitoring Ch-16. None of the above
B Setting the squelch control to just beyond the point where the background noise disappears results in: Reduced sensitivity without background noise. Maximum sensitivity without background noise. Minimum background noise with reduced sensitivity. Greater bandwidth without background noise.
B The “Scan” function is used to: Monitor Ch-16 continuously and switching to either Ch-70 or Ch-13 every 5 seconds. Sequentially scan all or selected channels. Scan Ch-70 for Distress Alerts. None of the above
C Setting the squelch control to the end of its range without any noise being heard results in: Less background noise Maximum sensitivity Minimum sensitivity Does not have any effect on the sensitivity.
D Proper and legal VHF operations require: The channel must be designated as valid for the nature or type of communications desired. The correct bandwidth must be selected by the operator. The power level must be appropriately chosen. Both answers a) and c) are correct
C Which of the following control selections may result in limited receiving range? Setting the squelch control to its minimum level. The power switch is set to the "high" output position, resulting in receiver overloading. Setting the squelch control to its maximum level. Setting the channel selection switch midway between channels 6 and 16.
B While conducting routine communications using the wheelhouse VHF with a station 1 mile distant, your recommended power setting would be: 25 watts after dark. 1 watt, day or night. 25 watts during a clear sunny day. 1 watt using DSC at night.
C Which factors normally determine the range of VHF transmissions: Channel frequency Power level Both and Vessel antenna height
B Causes of much longer than normal VHF transmissions are: Changing power from 1W to 25 W. Atmospheric ducting Ionospheric activity in layers F1/F2. None of the above
D Describing VHF transmissions as "line of sight" means: VHF communications are effective only with nearby stations within visual range of the bridge. Vessel antenna height will affect the radius of propagation. The normal transmission range to a coast station is approximately 25 NM. Both B and C are true
D The effectiveness of VHF communications is maximized by: The adjustment of squelch for maximum receiver sensitivity. Appropriate setting of the transmitter power. Selecting an appropriate channel. All of the above.
D Which channel would be the most likely to be used for routine ship-to-ship voice traffic? Ch-16 Ch-70 Ch-22A Ch-08
C Which channel is utilized for the required bridge-to-bridge watch? DSC on Ch-70 VHF-FM on Ch-16 VHF-FM on Ch-13 in most areas of the continental United States. The vessel's VHF working frequency.
B What channel would you use for routine communications with the U.S.C.G.? Ch-16 Ch-22A Ch-80 Ch-13
C What channel would you use to place a call to a shore telephone? Ch-16 Ch-70 Ch-28 Ch-06
D What channel is always being continuously monitored? Ch-28 Ch-80 Ch-16 Ch-70
B Which of the following channels may be used for duplex communications? Ch-70 Ch-26 Ch-5A Ch-22A
A The nearest Coast Guard station is being called by a vessel on Ch-22. His USA-INT switch is set to INT. What will be the results? The Coast Guard station will probably hear the call and respond but the vessel called will not hear the response. There should be no problem carrying on communications. The Coast Guard station will not hear the call because he is listening on a duplex receiving frequency. Neither station will hear the other’s calls.
B What is the reason for the USA-INT control or function? It changes channels that are normally simplex channels into duplex channels. It changes some channels that are normally duplex channels into simplex channels. When the control is set to “INT” the range is increased. None of the above
C Which of the following statements is true? You should always use the “INT” setting for calling a Public Correspondence station. You should use Ch-22 when calling the U. S. Coast Guard. Using the “USA” setting changes certain channels from duplex to simplex operation. Using the “INT” setting will prevent proper Ch-13 bridge-to-bridge operations.
B The USA-INT control on VHF units: Selects duplex operations for U.S. coastal waters and simplex operations in non-U.S. waters, on the "alpha" channels. Changes selected international duplex channels to simplex channels for use in U.S. waters. Both of the scenarios above may be set up and selected by the operator. None of the above
D The USA-INT control on VHF units: Was made necessary by a desire for more simplex channels in the U.S. Correctly set, will result in duplex operations in U.S. Coastal waters on the "alpha" channels. Correctly set, will result in simplex operations in U.S. Coastal waters on the "alpha" channels. Both a) and c) are true
D What would happen if your VHF is set to “USA” and you called a coast station on Ch-03? The coast station should hear your call and respond. You will be able to conduct normal communications. You will not be able to hear the coast station calling you. Both A and C are correct.
B The quickest way to transmit a DSC Distress Alert is: Select “Distress” priority from the menu and transmit an “all ships” call. Press the “Distress Hot Key” as specified by the equipment manufacturer. Transmit a “MAYDAY” call on Ch-16. None of the above
A A DSC Urgency priority call is usually set up in the following manner: They are sent to “all ships” or “all stations”. They are sent to an individual coast station. They are sent to an individual ship station. None of the above
C A DSC Safety call is usually used under the following conditions: Man overboard Distress situation Navigation Hazard All of the above.
B To send a Distress Alert use the following procedure: Initiate a menu call to select Ch-16 for voice communications. Use the “Distress Hot Key” in an appropriate manner. Always insert the nature of the distress first. None of the above
C DSC Urgency priority calls may be sent using the “Distress Hot Key” under the following circumstances: If no additional information is required to be transmitted. Only if the position information is correct. Under NO circumstances. None of the above
A A DSC Safety priority call would be used under the following circumstances: A lifeboat has been lost over the side in heavy weather and is adrift. A crew member is missing and presumed lost overboard. There is a fire in the engine room. None of the above
C How are Routine calls usually formatted/initiated? By pressing the “Distress Hot Key”. By pressing the “Alarm” button and then selecting from various options. By making the appropriate key strokes to select the appropriate menu choices. None of the above
A What would be the appropriate menu choice when calling another ship station? Simplex Ch-28 Duplex Ch-22
D What would be the appropriate channel selection for follow-on voice communications for a Routine priority call to another vessel? Ch-70 Ch-16 Ch-22 Ch-08
C If the vessel you wish to call is not listed in your VHF address book, what action should be taken? The vessel’s MMSI number must be manually entered during the call setup. The vessel’s name and MMSI number may be entered in the address book for access during call setup. Either of the above None of the above
B What conditions must exist for a completely automatic DSC acknowledgement of a Routine call and change to suggested working channel? Both stations must be monitoring Ch-16. The receiving station must be set up for automatic response. The receiving station must be monitoring the suggested working frequency. None of the above
B What actions will take place upon receipt of a Routine call from another vessel if both vessels are programmed for automatic response? The alarm will sound. The call particulars will appear on the screen and the channel will change to the suggested working frequency. The alarm will sound and the channel will change. All of the above.
B The DSC received call directory usually sorts and stores incoming calls in what manner? All received calls are saved in a single category. Received calls are typically divided into two directories but the names of these directories and which message priorities are stored in them vary -- depending on the equipment manufacturer. Received calls are always divided into Distress and Urgency categories. Only received Distress calls are saved.
C Why would you want to access your MMSI number? To call another vessel. To change it. To determine that the number is correct. There is no reason to access your MMSI number.
A What precautions should be taken when reviewing previously transmitted messages? Take steps not to accidentally send the message again. Press the “Call” key when finished. Always save the message. No precautions are necessary.
C How many times is it possible to change your own MMSI number in a VHF unit? It is not possible. It may be changed by the operator up to 4 times. On some units it can be changed at will if the password is known. None of the above
B What precautions should be taken when reviewing received distress messages? Press the “Call” key when finished. Take steps not to accidentally send a DSC acknowledgement or relay. Always save the message. No precautions are necessary.
C Which of the following is true regarding vessel position information? The position is normally provided from GPS input. If the position is not automatically updated it must be manually updated every 4 hours. Both and are true. None of the above
B What information is normally entered in the address book? The vessel’s IMN. Vessel’s name and MMSI number. The vessel’s call sign and name. All of the above.
C Which of the following statements is true? Some DSC units have both a Transmit and Receive message directory or database. Some DSC units may not contain a database of transmitted DSC calls. Both and are true. None of the above
D Which of the following statements on address book entries is correct? Shore based telephone numbers may be entered for automatic telephone calls ashore. Public Correspondence Stations may be entered. A vessel’s name and MMSI number may be entered. All of the above.
A The VHF-DSC self-test function (if available) usually performs the following: Makes a diagnostic test of the system without actually transmitting a signal. Transmits a very weak DSC signal which is picked up by the Ch-70 receiver. Transmits a weak voice signal which is picked up on Ch-16. Does a check sum of the memory chips.
B What can the operator do to adjust the audible alarm? The operator can choose to disable the audible alarm. The alarm cannot be disabled for distress and URGENCY messages. Both of the above None of the above
A When initiating a DSC call through the menu system what is the most likely information that will be displayed after making the first correct keystroke? The call setup menu. The particulars of the previous call. The priority menu. The Telecom-1 menu.
A What is the fundamental concept of the GMDSS? GMDSS utilizes automated systems and satellite technology to improve emergency communications for the world's shipping industry. It is intended to automate and improve existing digital selective calling procedures and techniques. It is intended to provide more effective but lower cost commercial communications. It is intended to provide compulsory vessels with a collision avoidance system when they are operating in waters that are also occupied by non-compulsory vessels.
B The primary purpose of the GMDSS is: Allow more effective control of SAR situations by vessels. Automate and improve emergency communications for the world's shipping industry. Provide additional shipboard systems for more effective company communications. Effective and inexpensive communications.
C What is the basic concept of GMDSS? Shoreside authorities will rely on reports from nearby vessels to become aware of Distress alerts. Shoreside authorities and vessels can assist in a coordinated SAR operation only after the correct chain of DSC relays takes place. SAR authorities ashore can be alerted to a Distress situation & shipping in the vicinity can be requested to participate in SAR operations. SAR authorities ashore wait to have EPIRB Distress alerts confirmed by satellite follow-on communications.
D GMDSS is primarily a system based on? Ship-to-ship Distress communications using MF or HF radiotelephony. VHF digital selective calling from ship to shore. Distress, Urgency and Safety communications carried out by the use of narrow-band direct printing telegraphy. The linking of search and rescue authorities ashore with shipping in the immediate vicinity of a ship in Distress or in need of assistance.
A What is the responsibility of compulsory GMDSS vessels? Every vessel must be able to perform communications functions essential for its own safety and the safety of other vessels. Vessels must transmit a DSC distress relay upon receipt of a DSC distress alert. Only the vessels closest to a Distress incident must render assistance. Vessels must immediately acknowledge all DSC distress alerts.
B GMDSS is required for which of the following? All vessels capable of international voyages. SOlAS Convention ships of 300 gross tonnage or more. Vessels operating outside of the range of VHF coast radio stations. Coastal vessels of less than 300 gross tons.
D Which GMDSS system utilizes terrestrial radio techniques? F-77 Inmarsat-C GPS VHF-MF-HF-DSC
A What equipment utilizes satellite communications? Inmarsat-C VHF-MF-HF NAVTEX SART
B What equipment is used in or near the survival craft? NAVTEX EPIRB Fathometer COSPAS-SARSAT
C What equipment is programmed to initiate transmission of Distress alerts and calls to individual stations? NAVTEX GPS DSC Controller DSC Scanning Watch Receiver
C What system provides accurate vessel position information to the GMDSS equipment? COSPAS-SARSAT EPIRB GPS Inmarsat-B
D Which of these can be used to receive MSI? SART EPIRB Inmarsat-B NAVTEX
C Which of the following regions lie outside Sea Areas A1, A2, and A3? Sea Areas only apply to Inmarsat footprint areas. Sea Area A3-I Inmarsat coverage and Sea Area A3-S HF SITOR (NBDP) coverage. Sea Area A4 There are no additional Sea Areas.
D What sea area is defined as being within range of a shore-based MF station that provides for continuous DSC alerting? Coastal waters Sea area A3 Sea area A1 Sea area A2
A If a vessel is engaged in local trade and at no point in its voyage travels outside the range of a VHF shore station with continuous DSC alerting then the vessel is operating in what area? Sea area A1 Coastal and international zones Inland and coastal waters Sea areas A1 and A2
B What is defined as an area, excluding sea areas A1 and A2, within the coverage of an Inmarsat geostationary satellite in which continuous alerting is available? Ocean Area Regions AOR-E, AOR-W, POR or IOR Sea Area A3 Sea Area A4 Coastal and Inland Waters
B SITOR (NBDP) equipment is a partial or alternate carriage requirement under GMDSS for vessels operating in which sea area(s)? A1 A3 and A4 A1 and A2 A1, A2, A3 and A4
C What is defined as the area within the radiotelephone coverage area of at least one VHF coast station in which continuous DSC alerting is available as defined by the IMO regulation for GMDSS? Ocean Area Regions AOR-E, AOR-W, POR or IOR Sea Area A2 Sea Area A1 Coastal and Inland Waters
B Which of the following is a functional or carriage requirement for compulsory vessels? A compulsory vessel must carry at least two (2) FCC licensed GMDSS Radio Operators in all sea areas as well as a GMDSS Maintainer in sea areas A3 & A4. A compulsory vessel must satisfy certain equipment carriage requirements based on the intended sea area of operation. A compulsory vessel must be able to transmit and respond to Distress alerts and carry only one (1) FCC licensed GMDSS Radio Operator in sea areas A1 & A2. None of these answers are correct.
C Which GMDSS communication functions must all compulsory vessels be capable of performing to meet International Maritime Organization requirements? Distress alerting and receipt of Maritime Safety Information via Inmarsat for all vessels intending to operate in Sea Area A4. Distress alerting and receipt of MSI in Sea Areas A1, A2, A3, and A4 regardless of the vessel's intended area of operation. Distress alerting, general communications and receipt of Maritime Safety Information in the vessel's intended area of operation. General communications via Inmarsat and receipt of Maritime Safety Information via Enhanced Group Calling in Sea Area A4.
D GMDSS-equipped ships will be required to perform which of the following communications functions? Distress alerting, MSI, SAR and on-scene communications & receipt of satellite alerts from other vessels. SAR and on-scene communications, Bridge-to-Bridge and general radio communications, MSI and relay of satellite alerts from other vessels. Bridge-to-Bridge and general radio communications, RDF of EPIRB homing signals, Distress alerting and MSI. Transmit distress alerts, SAR and on-scene communications, MSI, Bridge-to-Bridge and general radio communications.
A What equipment can be used to receive Maritime Safety Information? NAVTEX, EGC receiver or HF SITOR (NBDP). EGC receiver, Inmarsat B or F77 terminal. HF SITOR (NBDP), Inmarsat B or NAVTEX. All of these answers are correct.
D Which of the following are required GMDSS functions? Bridge-to-Bridge communications, reception of weather map facsimile broadcasts, SAR communications. Reception of weather map facsimile broadcasts, receiving company email, On-scene communications. Reception of VHF weather channels, On-scene communications, general communications. Bridge-to-Bridge communications, general communications, SAR communications.
A Which of the following are required GMDSS functions for vessels? Transmit and receive locating signals, general communications and SAR communications. Transmit and receive general communications, transmit Distress Alerts by at least one means, MSI. Transmit and receive locating signals, send MSI to other ships via EGC, Bridge-to-Bridge communications. Transmit and receive SAR communications, transmit Distress Alerts by at least one means, Bridge-to-Bridge communications.
A Which statement is true regarding a vessel equipped with GMDSS equipment that will remain in Sea Area A1 at all times? The vessel must be provided with a radio installation capable of initiating the transmission of ship-to-shore Distress alerting from the position from which the ship is normally navigated. VHF DSC alerting may be the sole means of Distress alerting. HF or MF DSC may satisfy the equipment requirement. HF SSB with 2182 kHz automatic alarm generator may satisfy the equipment requirement.
B What statement is true regarding the additional equipment carriage requirement imposed for the survival craft of vessels over 500 gross tons? Additional carriage of two radio equipped lifeboats aft. A second radar transponder is required. Four additional portable VHF radios are required. The ability to communicate in all modes with any shore station.
C Vessels operating in which sea area(s) are required to carry either Inmarsat or HF equipment or a combination thereof under GMDSS? All sea areas A4 A3 A1
D Within a single sea area, what is the primary reason GMDSS imposes carriage requirements for different radio subsystems? Redundancy in duplicating all operational functions in the event of a system failure. Different subsystems are required to meet the specific equipment carriage requirements of national authorities. GMDSS vessels must be equipped to communicate in all modes with coast radio stations. The combined capabilities of redundant subsystems mitigate the risk of a single point of failure.
A If operating within Ocean Area A1, and outside of NAVTEX coverage, a GMDSS-equipped vessel must carry: Equipment capable of reception of Maritime Safety Information by the Inmarsat enhanced group call system, or HF SITOR (NBDP). A GPS receiver. Equipment capable of maintaining a continuous DSC watch on 2187.5 kHz. An Inmarsat-B terminal.
B What is the equipment carriage requirement for survival craft under GMDSS? At least three SCT units and two SARTs on every cargo ship between 300-500 gross tons and the same on all passenger ships regardless of tonnage. At least three SCT units and two SARTs on every passenger ship and cargo ships of 500 gross tons and upwards. At least two radar transponders must be carried on every cargo ship of 300-500 gross tons and two radar transponders (one for each side) of every passenger ship regardless of tonnage. All cargo vessels above 300 gross tons and every passenger ship regardless of tonnage must carry three SCT units and two SARTs.
D Which of the following statements concerning maintenance requirements is false? Compulsory vessels sailing in Sea Areas A1 and A2 must provide any one of the three maintenance options which are duplication of equipment, shore-based, or at-sea maintenance capability. Compulsory vessels sailing in Sea Areas A3 and A4 must provide any two of the three maintenance options which are duplication of equipment, shore-based, or at-sea maintenance capability. Equipment warranties do not satisfy GMDSS maintenance requirements. If shore-based maintenance is used, maintenance services do not have to be completed or performance verified unless the vessel will be sailing to a non-US port.
A Which of the following statements concerning GMDSS maintenance requirements is true? The options are duplication of equipment, at-sea maintenance, and shore-based maintenance. Compulsory vessels between 300-500 gross tons are required only to provide one maintenance option, while compulsory vessels larger than 500 gross tons and all passenger vessels are required to provide any two of the three maintenance options. The "at-sea" maintenance may be waived if the compulsory vessel carries at least three licensed GMDSS Radio Operators. Compulsory vessels operating in Sea Area A4 are required to carry at least one licensed GMDSS Radio Maintainer.
B Which of the following is a GMDSS requirement for all vessels over 300 gross tons operating within range of a MF-DSC equipped shore station? Ship's Master or radio officer must be on watch at all times. Only one of the three maintenance options is required. MF communications must be handled by the holder of a General Radiotelephone Operator's License. Only FCC required spare parts and a maintenance kit for repairs are required.
C What statement is correct regarding the maintenance requirements for A3 ships under GMDSS? If the vessel selects at-sea maintenance no additional parts and spares are required. On-board maintenance provided by a person holding a GMDSS Maintainer's license will fully meet the requirements. Redundancy of functions of certain equipment and on-board maintenance provided by a person holding a GMDSS Maintainer's license will partially meet this requirement. Shoreside maintenance and scheduled tests and inspections will not partially meet this requirement.
C A ship operating in sea area A-3 must have the following provisions for maintenance: Duplication of Equipment. Shore Maintenance. Any two of these are required. At Sea Maintenance.
D A ship operating in sea area A-1 must have the following provisions for maintenance: Shore maintenance. Duplication of equipment. At Sea Maintenance. Any one of these is sufficient.
C What is the frequency range for Medium Frequency? 10-30 MHz 1,000-10,000 kHz 300-3,000 kHz 30-300 kHz
D What is the frequency range for High Frequency? 300-3,000 kHz 30-300 MHz 10-30 MHz 3-30 MHz
A What is the frequency range for Very High Frequency? 30-300 MHz 3-30 MHz 300-3,000 kHz 10-30 MHz
B What is the frequency range for Ultra High Frequency? 3-30 MHz 300-3,000 MHz 30-300 MHz 10-30 MHz
B What is the frequency range for Super High Frequency? 30-300 GHz 3-30 GHz 300-3,000 MHz 30-300 MHz
C What is the primary frequency range for terrestrial communications using skywave propagation? 300-3,000 kHz 30-300 MHz 3-30 MHz 10-30 MHz
B Which of the following systems is most susceptible to fading or static interference? Inmarsat HF SITOR (NBDP) DSC on channel 70 VHF ARQ
C Which system is most likely to be affected by atmospheric disturbances? VHF DSC Inmarsat MF/HF radiotelephony SafetyNETTM
D Which of the following systems is least susceptible to fading or static interference? HF SITOR (NBDP) MF-HF DSC Controller VHF ARQ Inmarsat
A Which of the following systems is least susceptible to atmospheric disturbances? Inmarsat NAVTEX MF SITOR (NBDP) HF SITOR (NBDP)
D Which of the following frequency bands would most likely provide reliable communications between two stations that are 100 miles (160 km) apart? The Low Frequency (LF) band. The High Frequency (HF) band. The Very High Frequency (VHF) band. The Medium Frequency (MF) band.
A Which system has the least effective radius of operation? VHF DSC HF SITOR (NBDP) MF SITOR (NBDP) NAVTEX
A What statement best describes modulation? Imposing intelligence onto a radio carrier signal. changing mark-space to 1 and O. Adjusting the frequency to the optimum band for long distance communications. converting the carrier from a low frequency to a higher frequency.
B What statement best describes demodulation? Detuning the receiver to remove interfering signals. Extracting intelligence from the radio carrier signal. Removing atmospheric noise from the signal. Separating the TELEX signals from the voice signals.
C Which statement best describes amplitude modulation? The character data from the terminal is changed to audio tones. The frequency is varied in synchronization with the modulating signal. The amplitudes in the sideband(s) changes but the radio carrier frequency remains constant. The amplitude of the carrier is changed but there is still only a single frequency being transmitted.
D What is the emission designation for MF-HF voice signals? F1B J2B F3E J3E
A Which statement best describes frequency modulation? The information signal changes the radio carrier frequency but the amplitude remains constant. Both the amplitude and frequency are changed by the modulating signal. Frequency modulation is subject to interference by atmospheric noise. High level mixing of the final amplifier signal and the information signal.
B What is the emission designation for VHF-FM? F3c F3E J2B H3E
D Which of the following statements describes the carrier? The carrier consists of at least 3 separate but closely spaced frequencies. The carrier is used to modulate the information signal. There are always sidebands on either side of the carrier. The carrier is a Radio Frequency (RF) signal that is modulated to carry intelligence.
A On what frequencies would the J3E emission be used for GMDSS Distress communications? 2182.0 kHz, 16420.0 kHz or 8291.0 kHz. 2182.0, 2187.5 KHz or 8414.5 kHz. 8414.5 KHz, 8291.0 kHz or 8376.5 kHz. 4125.0 kHz, 16420.0 kHz or 4177.5 kHz.
B How many sidebands are present in the J3E mode? Two sidebands and a carrier. One upper sideband. One lower sideband. Two carriers and one sideband.
C What is the signal transmitted in H3E mode? Two sidebands, upper and lower. A reduced carrier and the lower sideband. A full carrier and the upper sideband. A full carrier and both upper and lower sidebands.
C What is the signal transmitted in J2B mode? A full carrier and one sideband. A full carrier and two sidebands. An upper sideband of 2 alternating tones. An upper sideband of a single tone switched on and off.
D Which of the following statements is true? An RF carrier is always required to carry the information. There is only a single tone used in J2B mode. There are a carrier and two sidebands in H3E mode. There are two tones used in J2B mode.
C What is the channel spacing for MF-HF voice frequencies? 500 Hz 2.8 kHz 3.0 kHz 3.5 kHz
D What is the channel spacing for authorized F1B/J2B frequencies? 170 Hz 300 Hz 3.0 kHz 500 Hz
A You look up a frequency table and all the listings end in either .0 or .5 kHz. What kind of emission is used with these frequencies? F1B/J2B SITOR (NBDP) J3E SSB Voice F3E FM Voice G3E FM Voice
B How many SITOR (NBDP) signals can occupy the space of one voice signal? 10 6 4 2
B Which emission mode occupies the most bandwidth? J2B F3E J3E F1B
C Which mode occupies the least bandwidth? H3E A3E J2B F3E
B A vertical (whip) antenna has a radiation pattern best described by? A figure eight A circle A cardioid An ellipse
C Why is an antenna tuner required for MF-HF transmissions? The antenna tuner calculates the proper spectrum band for the operator to use. To ensure transmissions are restricted to legal marine frequencies. The length of the physically-fixed antenna must be electrically matched to the intended frequency of operation. The antenna tuner indicates whether the ionosphere is ready to reflect a transmission properly.
D A vertical quarter wave antenna with a good ground connection will: Radiate directionally due to being grounded. Not function due to being grounded. Only be used in Satellite communications. Radiate omni-directionally.
A What is the most common type of antenna for GMDSS MF-HF? Vertically polarized whip antenna Horizontally polarized long wire antenna. Horizontally polarized vertical whip antenna. Satellite radome with vertical polarization.
D What is the purpose or function of the antenna coupler? Electrically matching the antenna system to the transmit frequency. changing the overall wavelength of the antenna system (in addition to the fixed length whip.) Enabling maximum transmitted signal on the band chosen by the operator. All of these answers are functions of the antenna coupler.
A What advantage does a vertical whip have over a long wire? It radiates equally well in all directions. It radiates directionally for better propagation. It radiates a strong signal vertically. It radiates more signal fore and aft.
A How often must a compulsory vessel's GMDSS radio station be inspected? Annually, by the FCC or designated authority. Annually, by the U.S. Coast Guard. Annually, by the FCC, and every six months if the vessel sails outside of Sea Areas A1 and A2. The FCC's annual inspection may be waived if and only if monthly inspections are performed by the vessel's on-board GMDSS Radio Maintainer.
B What periodic inspection is required in order to remain in compliance with the regulations regarding GMDSS ship radio station inspections? U.S. Coast Guard annual inspection. An inspection at least once every 12 months by the FCC or a holder of a GMDSS Maintainers license. FCC inspection every five years. Periodic inspections not required if on board maintainers perform routine preventive maintenance.
C Which statement is false regarding a GMDSS-equipped ship? A conditional or partial exemption may be granted, in exceptional circumstances, for a single voyage outside the sea area for which the vessel is equipped. The regulations apply to all passenger ships regardless of size and cargo ships of 300 gross tons and upwards. Ships must have the required equipment inspected at least once every five years. Ships must carry at least two persons holding a GMDSS Radio Operator's license for Distress and Safety radio-communications purposes.
D Which statement is false regarding a GMDSS equipped ship? Ships must have the required equipment inspected at least once every 12 months. The regulations apply to all passenger ships regardless of size and cargo ships of 300 gross tons and upwards. Ships must carry at least two persons holding a GMDSS Radio Operator's license for Distress and Safety radio-communications purposes. A conditional or partial exemption may not be granted, in exceptional circumstances, for a single voyage outside the sea area for which the ship is equipped.
A During an annual GMDSS station inspection: All required documents and publications might have to be produced and GMDSS operators may be required to demonstrate equipment competencies. Licensed GMDSS operators may not be required to demonstrate equipment competencies but all required equipment must be fully operational. All required equipment must be fully operational and any required publications that are not current must be on order. GMDSS operators may be required to demonstrate equipment competencies and any of required equipment that is not fully operational can be repaired at the next port of call as long as there is functional duplication.
A Where can GMDSS regulations pertaining specifically to U.S.-flag vessels be found? These are located in 47 CFR Part 80. These are located in CCIR #476. These are located in FCC Part 83. These are published only by the U.S. Coast Guard.
D Which of the following references should be consulted for information on the proper setup and use of GMDSS equipment? ITU List of Equipment Operations. 47 CFR Part 80 Subpart W. Instructions are available through the Maritime Safety Information (MSI) system. The manufacturer's operating manuals.
B What publications should the GMDSS Radio Operator consult to review the proper procedures to be followed in Distress situations under GMDSS? The manufacturer's technical manuals. 47 CFR Part 80 Subpart W. The manufacturer's operator manuals. 47 CFR Part 90 Subpart V.
C Which of the following documents or publications are required by the FCC for GMDSS vessels on international voyages (other than the Great Lakes)? IMO master plan of shore-based facilities (or substitute), station logs, appropriate operator licenses, Inmarsat handbook for GMDSS. NGA Pub. 117 (or substitute), station logs, appropriate operator licenses, IAMSAR manual volume III. Part 80 FCC rules, station logs, station licenses, ITU publications, ITU manual for Maritime Mobile stations. Part 80 FCC rules, station logs, station licenses, ITU publications, IMO manual for Maritime Mobile stations.
C Which of the following documents or publications are required by Part 80 of the FCC rules for vessels on international voyages (other than the Great Lakes)? Appropriate Safety Convention Certificate, ITU Master Plan of GMDSS Coast stations, and ITU manual for Maritime Mobile stations. ITU Master Plan of GMDSS Coast stations, ITU manual for Maritime Mobile stations, ITU List IV & List V. ITU List IV & List V, Appropriate Safety Convention Certificate & ITU manual for Maritime Mobile stations. ITU List IV & List V, Appropriate Safety Convention Certificate & IMO manual for Maritime Mobile stations.
D Which of the following references should be consulted to identify the name of a vessel based on its Maritime Mobile Service Identity? ITU list of Coast Stations. ITU List of Radio-determination and Ship Stations. ITU Master Plan of Shore-Based Facilities. ITU List of Ship Stations and Maritime Mobile Service Identity Assignments.
C Which of the following maintenance functions is not the responsibility of the GMDSS Radio Operator? Visual inspection of equipment, including the antenna and associated components. Perform on-the-air verification checks. Aligning the power output stage for maximum power. Perform scheduled testing of the battery's charged condition.
D When may a compulsory vessel not be allowed to leave port? When the vessel is in an over-carriage condition. When the vessel has arranged for both duplication of equipment and shore-based maintenance. When the vessel is carrying only two licensed GMDSS Radio Operators and is capable of performing all required functions. When the vessel has replaced a required piece of GMDSS-related equipment but its performance has not been verified or logged.
A Which statement is false regarding the maintenance of GMDSS equipment at sea? The GMDSS maintainer may not be the person designated to have primary responsibility for radio- communications during Distress incidents even if licensed as an operator. Ships must carry at least one person who qualifies as a GMDSS maintainer for the maintenance and repair of equipment if the at-sea maintenance option is selected. All at-sea maintenance and repairs must be performed by, or under the supervision of a person holding a GMDSS Maintainer license. The GMDSS maintainer may be the person responsible for ensuring that the watches are properly maintained and that the proper guard channels and the vessel's position are entered into the DSC equipment.
B Which of the following service or maintenance functions may NOT be performed by the holder of a GMDSS Radio Operator License? Reset tripped circuit breakers or replace defective fuses. Any adjustments or maintenance that may affect the proper operation of the station. Routine battery maintenance if used as part of the GMDSS station. Replacement of consumable items such as paper, ribbons, etc.
B What are the conditions, under GMDSS, whereby a ship is NOT allowed to depart from any port? The vessel is carrying more than the required number of qualified GMDSS radio operators. The vessel is not capable of performing all required Distress and Safety functions. The vessel has a temporary waiver of its radio license and Safety Certificate. The vessel is not carrying a GMDSS radio maintainer, but has provided for shoreside maintenance plus duplication of equipment if required.
C What determines the spares and maintenance materials requirements for the SITOR (NBDP) equipment under GMDSS? 47 CFR Part 80 IMO Circular "Equipment Spares". The recommendations of the manufacturer. The GMDSS Maintainer's requirements.
B Which FCC license meets the requirement to serve as a GMDSS operator? General Radiotelephone Operator's License. GMDSS Radio Operator's License. Marine Radio Operator's Permit. GMDSS Radio Maintainer's License.
C Which of the following statements concerning GMDSS Radio Operator requirements is false? Each compulsory vessel must carry at least two licensed GMDSS Radio Operators at all times while at sea. Each compulsory vessel must carry at least two licensed GMDSS Radio Operators at all times while at sea and may elect to carry a GMDSS Radio Maintainer as well. All communications involving Safety of life at sea must be logged as long as the compulsory vessel was not involved in such communications. While at sea, adjustments to, and the maintaining of, GMDSS equipment may be performed by the GMDSS Radio Operator as long as the work is supervised by an on-board licensed GMDSS Radio Maintainer.
D Which FCC license meets the requirements to perform or supervise the performance of at-sea adjustments, servicing, or maintenance which may affect the proper operation of the GMDSS station? General Radiotelephone Operator's License with Shipboard RADAR endorsement. Marine Radio Operator's Permit or GMDSS Maintainer's license. GMDSS Radio Operator's license or Marine Radio Operator's Permit. GMDSS Operator's/Maintainer's license or GMDSS Maintainer's license.
A Which statement is false regarding the radio operator requirements for a GMDSS-equipped ship station? Maintaining a record of all incidents connected with the radio-communications service that appear to be of importance to Safety of life at sea is not required. One of the qualified GMDSS radio operators must be designated to have primary responsibility for radio- communications during Distress incidents. A qualified GMDSS radio operator, and a qualified backup, must be designated to perform Distress, Urgency and Safety communications. While at sea, all adjustments or radio installations, servicing or maintenance of such installations that may affect the proper operation of the GMDSS station must be performed by, or under the supervision of, a qualified GMDSS radio maintainer.
D Which of the following are personnel, functional, or equipment FCC requirements of the GMDSS? One FCC licensed GMDSS radio operator in sea areas A1 & A2, two FCC licensed GMDSS radio operators in sea areas A3 & A4 and equipment carriage based on intended sea area of operations. Distress alerting and response, two USCG STCW GMDSS watchstanders, equipment carriage based on intended sea area of operations. Equipment carriage reduced for sea areas A3 & A4, Distress alerting and response and two FCC licensed GMDSS radio operators. Equipment carriage based on intended sea area of operations, distress alerting and response and two FCC licensed GMDSS radio operators.
A How many GMDSS radio maintainers must be carried aboard a compulsory vessel if the At-Sea maintenance method is used? One regardless of sea area of operation. Two in Sea Areas A3 and A4. Two in Sea Area A1. None of these answers are correct.
A Which statement is false regarding the GMDSS requirement for ship sources of energy? The reserve sources of energy need to supply independent MF and HF radio installations at the same time. At all times while the vessel is at sea, a sufficient supply of electrical energy to operate the radio installations and charge any batteries which may be part of the reserve source of energy is required. An uninterruptible power supply or other means of ensuring a continuous supply of electrical power to all GMDSS equipment that could be affected by normal variations and interruptions of ship's power is required. If a vessel's position is constantly required for the proper performance of a GMDSS station, provisions must be made to ensure position information is uninterrupted if the ship's source of main or emergency energy fails.
B What is the meaning of "Reserve Source of Energy"? High caloric value items for lifeboat, per SOlAS regulations. Power to operate the radio installation and conduct Distress and Safety communications in the event of failure of the ship's main and emergency sources of electrical power. Diesel fuel stored for the purpose of operating the powered survival craft for a period equal to or exceeding the U.S.C.G. and SOlAS requirements. The diesel fueled emergency generator that supplies AC to the vessel's Emergency power bus.
C Which term describes the source of energy required to supply the GMDSS console with power if the ship's source of main or emergency energy fails? Emergency power Ship's emergency diesel generator Reserve Source of Energy Ship's standby generator
D What characteristics describe the GMDSS Reserve Source of Energy (RSE)? Supplies independent HF and MF installations at the same time. Cannot be independent of the propelling power of the ship. Must be incorporated into the ship's electrical system. Must be independent of the ship's electrical system when the RSE is needed to supply power to the GMDSS equipment.
A What is the requirement for emergency and reserve power in GMDSS radio installations? Compulsory ships must have emergency and reserve power sources for radio communications. An emergency power source for radio communications is not required if a vessel has proper reserve power (batteries). A reserve power source is not required for radio communications. Only one of the above is required if a vessel is equipped with a second 406 EPIRB as a backup means of sending a Distress alert.
B Which of the following terms is defined as a back-up power source that provides power to radio installations for the purpose of conducting Distress and Safety communications when the vessel's main and emergency generators cannot? Emergency Diesel Generator (EDG) Reserve Source of Energy (RSE) Reserve Source of Diesel Power (RSDP) Emergency Back-up Generator (EBG)
D Under GMDSS, a compulsory VHF-DSC radiotelephone installation must be tested at what minimum intervals at sea? Annually, by a representative of the FCC. At the annual SOlAS inspection. Monthly Daily
A Which statement concerning the testing of a compulsory radiotelephone station is false? Calling the USCG on VHF CH-16 or 2182.0 kHz is the most effective method. Tests may be accomplished by using the equipment for normal business. A daily test is necessary unless the equipment was used for routine traffic. The test may not interfere with communications in progress and must wait or be suspended if a request to do so is made.
B While underway, how frequently is the DSC controller required to be tested? Once a week Once a day Twice a week Once a month
C At sea, all required equipment (other than Survival Craft Equipment) must be proven operational by: Testing at least every 48 hours. Weekly testing of all S.C.E. and other compulsory equipment. Daily testing or daily successful use of the equipment. Daily testing of the S.C.E. and weekly tests of the other equipment.
C The best way to test the MF-HF SITOR (NBDP) system is: Make a radiotelephone call to a coast station. Initiate an ARQ call to demonstrate that the transmitter and antenna are working. Initiate an ARQ call to a Coast Station and wait for the automatic exchange of answerbacks. Initiate an FEC call to demonstrate that the transmitter and antenna are working.
D The best way to test the Inmarsat-C terminal is: Send a message to a shore terminal and wait for confirmation. Send a message to another ship terminal. If the "Send" light flashes, proper operation has been confirmed. Compose and send a brief message to your own Inmarsat-C terminal.
C A vessel certified for service in Sea Area A3 is required to maintain a watch on: VHF Channel 70, MF Frequency 2182.0 kHz, HF on 8414.5 kHz and one other HF DSC frequency. MF Frequency 2187.5 kHz, HF on 8414.5 kHz and one other HF DSC frequency, HF on 4125.0 kHz. VHF Channel 70, MF Frequency 2187.5 kHz, HF on 8414.5 kHz and one other HF DSC frequency. VHF Channel 16, VHF Channel 70, MF Frequency 2187.5 kHz, HF on 8414.5 MHz and HF 4177.5 MHz.
D A vessel certified for service in Sea Area A-2 is required to maintain watch on: 2174.5 kHz 2182.0 kHz 2738.0 kHz 2187.5 kHz
A What are the mandatory DSC watchkeeping bands/channels? 8 MHz HF DSC, 1 other HF DSC, 2 MHz MF DSC and VHF Ch-70. 2 MHz MF DSC, 8 MHz DSC, VHF Ch-16 and 1 other HF DSC. VHF Ch-70, 2 MHz MF DSC, 6 MHz DSC and 1 other HF DS VHF Ch-70, 2 MHZ MF DSC, 4 MHZ DSC and 8 MHz DSC.
B Proper watchkeeping includes the following: Monitoring all required frequencies in the proper mode, setting the DSC scanner to 2 MHz, 4 MHZ and 8 MHz for ships in the vicinity, notifying the Master of any Distress alerts. After silencing an alarm all displays and/or printouts are read, monitoring all required frequencies in the proper mode, notifying the Master of any Distress alerts. Notifying the Master of any Distress alerts, setting the DSC scanner to 2 MHz, 4 MHZ and 8 MHz for ships in the vicinity, monitoring all required frequencies in the proper mode. Setting the DSC scanner only to the mandatory 2 MHz & 8 MHz, maintain continuous watch on 2182.0 kHz or 4125.0 kHz, notify the Master of any Distress traffic heard.
B Proper watchkeeping includes the following: Understanding normal operational indicators, setting the DSC scanner frequencies to minimize alarms, maintaining a proper log. Maintaining a proper GMDSS radio station log, understanding normal operational indicators, responding to and comprehending alarms. Responding to and comprehending alarms, logging out of Inmarsat-C terminals while at sea, maintaining a proper GMDSS radio station log. Maintaining a proper GMDSS radio station log, setting the DSC scanner frequencies to minimize alarms, logging out of Inmarsat-C terminals while at sea.
C Which is true concerning a required watch on VHF Ch-16? When a vessel is in an A1 sea area and subject to the Bridge-to-Bridge act and in a VTS system, a watch is required on Ch-16 in addition to both Ch-13 and the VTS channel. It is not compulsory at all times while at sea until further notice, unless the vessel is in a VTS system. When a vessel is in an A1 sea area and subject to the Bridge-to-Bridge act and in a VTS system, a watch is not required on Ch-16 provided the vessel monitors both Ch-13 and the VTS channel. It is not always compulsory in sea areas A2, A3 and A4.
B Which of the following statements meets requirements for 47 CFR 80 Subpart-W? GMDSS Radio Logs may not be retained aboard compulsory vessels in an electronic file (e.g., word processing) format. GMDSS Radio Logs must contain entries of all Distress and Urgency communications affecting your own ship. GMDSS Radio Logs must be retained aboard compulsory vessels for a period of at least 90 days in their original form. Entries in the GMDSS Radio Log are only required for communications within the vessel's intended Sea Area of operation.
C Which of the following statements is false? Key letters or abbreviations may be used in GMDSS Radio Logbooks if their meaning is explained. Urgency communications may need to be entered in the GMDSS radio log. Distress communications heard do not require entries if the vessel did not participate in SAR activity. Log entries of VHF Safety broadcasts are not required.
D Where should the GMDSS radio log be kept on board ship? Captain's office Sea cabin Anywhere on board the vessel. At the GMDSS operating position.
A How long must the radio log be retained on board before sending it to the shoreside licensee? At least 30 days after the last entry. At least one year after the last entry. At least two years after the last entry. At least 90 days after the last entry.
D Which statement concerning radio log archival by the station licensee is false? Retain for two years if there are no Distress entries. Logs related to an investigation may not be destroyed without specific authorization. Retain for three years if there are Distress entries. Retain for one year unless there are Distress or Urgency entries.
A Which of the following log keeping statements is false? Entries of all company communications using GMDSS satellite equipment are required. Entries relating to pre-voyage, pre-departure and daily tests are required. A summary of all Distress communications heard and Urgency communications affecting the station's own ship. Also, all Safety communications (other than VHF) affecting the station's own ship must be logged. Entries related to failures of compulsory equipment are required.
A A typical call sign for a large container ship under U. S. flag would be: KBZY WBX1469 NADN KPH
B What would the number 1090 indicate? A ship DSC MMSI number. A coast station SITOR (NBDP) SELCAL number. A coast station DSC MMSI number. A ship station SITOR (NBDP) SELCAL number.
C Which one of the following is a ship station SELCAL? 1104 1502352 11243 230364973
D Which of the following is the call sign for a U.S.C.G. coast station? NERK KPH WCC NMN
A What type of station would be assigned the call sign WAB2174? Tug boat Container ship Passenger ship Bulk Tanker
B What number will a ship station use to identify itself using SITOR (NBDP)? Four digit SELCAL. Five digit SELCAL or 9 digit SELCAL number identical to MMSI. 9 digit Inmarsat-B I. D. number. 9 digit Inmarsat-C I.D. number.
D What is the MID? Mobile Identification Number Marine Indemnity Directory Mobile Interference Digits Maritime Identification Digits
A How many digits are in the MID (Maritime Identification Digits)? 3 7 9 10
B What does the MID (Maritime Identification Digits) signify? Port of registry Nationality Gross tonnage Passenger vessel
C Which of the following numbers indicates a U.S. flag ship station? 036627934 243537672 338426791 003382315
C Which of the following MMSI numbers indicates a U.S. flag ship station? 430326890 033609991 303236824 257326819
D Which of the following numbers indicates a ship station MMSI? 003372694 030356328 3384672 623944326
C A DSC call is received from a station with a MMSI number of 003669991. What type of station made the call? A vessel operating in Sea Area A3. A group ship station A U.S. coast station An Intercoastal vessel
D A valid MMSI number for a DSC call to a specific group of vessels is: 003664523 338462941 003036483 030327931
A A MMSI 030346239 indicates what? Group MMSI Inmarsat-C I.D. number Coast station Ship station
B Which of the following statements concerning MMSI is true? Coast station MMSI numbers have 9 digits starting with 4. All MMSI numbers are 9 digits and contain an MID. Ship station MMSI numbers can be 7 digits or 9 digits depending on the Inmarsat terminal. Group MMSI numbers must begin with 2 zeros.
B Which of the following statements concerning MMSI is false? All Coast Station MMSI must begin with 2 zeros. All Coast Station MMSI must begin with the MID then 2 zeros. A group call must begin with a single zero followed by the MID. The first 3 digits of a ship MMSI comprise the MID.
C Which of the following statements concerning MMSI is true? All ship station MMSI must begin with a single zero and include the MID. All group station MMSI must begin with the MID. None of these answers are correct. All Coast Station MMSI must be 9 digits and begin with the MID and then two zeros.
B Which of the following would indicate an Inmarsat-B terminal? A 9 digit number beginning with the MID. A 9 digit number always starting with 3. A 7 digit number. A 9 digit number always starting with 4.
C Which of the following would indicate an Inmarsat-B terminal? 003662517 436682011 325468325 1500241
D Which of the following would indicate an F77 terminal? 150036 430364290 1502460 766832922
A Which of the following would indicate an F77 terminal? 765044177 003668202 436682433 1500270
D Which of the following would indicate an Inmarsat-B terminal? 150036 430363275 1502460 366632824
A Which of the following would indicate an F77 terminal? A 9 digit number always starting with 76. A 12 digit number starting with the MID. A 9 digit number ending with the MID. A 9 digit number always starting with 4.
A You receive a TELEX with the sender's I.D. of 433863491. What type of terminal sent this message to your vessel? Inmarsat-C Land TELEX terminal Inmarsat-B Inmarsat-M
B You receive a TELEX with the sender's I.D. of 436640927. What type of terminal sent this message to your vessel? F77 Fleet Inmarsat-C Land TELEX terminal Inmarsat-M
C Which of the following would indicate an Inmarsat-C terminal? 003668202 325468263 436682433 1500270
D Which of the following would indicate an Inmarsat-C terminal? 150036 366294824 1502690 430346275
A Which of the following would indicate an Inmarsat-C terminal? 436682011 003662517 325468325 1500241
B Which of the following would indicate an Inmarsat-C terminal? A 9 digit number beginning with the MID. A 9 digit number always starting with 4. A 7 digit number. A 9 digit number always starting with 3.
D What sequence best describes an MF-HF DSC Distress menu transmission? Vessel name, Distress priority code, vessel position & time, nature of Distress, A string of dots to stop the DSC scanner, vessel MMSI, vessel position & time, nature of Distress, frequency for follow-on communications. Vessel MMSI, frequency & emission for follow-on communications, vessel position & time, Distress priority code. A string of dots to stop the DSC scanner, Distress priority code, vessel position & time, emission for follow-on communications, DSC frequency.
A MF-HF DSC alerts are transmitted using what emission type? J2B to ensure compatibility with DSC receivers. J3E to ensure correct voice follow-on frequencies are used. F1B to ensure correct SITOR (NBDP) follow-on frequencies are used. A3E to achieve the best transmission range.
B When sending a DSC call: Vessel's position will automatically be sent with DSC calls specifying an alternate frequency. Vessel's position will automatically be sent if the vessel is sending a "Distress Hot Key" alert. Vessel's MMSI will indicate its ocean region and vessel position. Vessel's MMSI and position will automatically be sent for all types of DSC calls.
C DSC transmissions are encoded: Using J3E mode for proper follow-on communications. Using F1B mode to ensure proper reception. Using a ten-bit error detecting code. Using J2B mode for correct transmission.
C DSC transmissions are received: Using voice or TELEX modes as appropriate. Using J3E or H3E modes as appropriate. Using digital decoding by the DSC controller. Using F1B and/or J2B decoding by the transceiver.
D Properly formatted MF-HF DSC transmissions can request which of the following emissions for follow on communications? J3E/H3E telex emissions F1B/J2B voice emissions J3E/F3E voice emissions J3E/H3E voice emissions
C Which of the following statements concerning DSC equipment is true? The GMDSS Radio Operator is not responsible for properly selecting HF DSC guard channels because they are done automatically by the watch receiver. All equipment must be type accepted by Inmarsat. The vessel's navigational position must be updated, either automatically or manually, no less often than every four (4) hours. The GMDSS Radio Operator is responsible for properly selecting VHF & MF DSC guard channels but the HF frequencies are done automatically by the watch receiver.
D What is the action that a GMDSS Radio Operator should take when a DSC Distress alert is received requesting J3E? No action is necessary, as the DSC control unit will automatically switch to the SITOR (NBDP) follow-on communications frequency. The Operator should immediately set continuous watch on VHF channel 70. The Operator should immediately set continuous watch on the SITOR (NBDP) frequency that is associated with frequency band on which the Distress alert was received. The Operator should immediately set continuous watch on the radiotelephone frequency that is associated with frequency band on which the Distress alert was received.
A What does the DSC control unit do if the GMDSS Radio Operator fails to insert updated information when initiating a DSC Distress alert? It will initiate the DSC Distress alert and default information will automatically be transmitted. It will abort the transmission and set off an audible alarm that must be manually reset. It will initiate the DSC Distress alert but, as no information will be transmitted, rescue personnel will not be able to identify the vessel, its position, or its situation. It will initiate the DSC Distress alert, but any station receiving it will have to establish contact with the distressed vessel to determine its identity, position, and situation.
B A DSC Distress alert: Must always be sent on VHF Ch-70. Will always be sent on one or more of the DSC Distress frequencies. Must always be sent on MF 2 MHz plus one other HF DSC frequency. Must always be sent on VHF Ch-70, then 2 MHz MF then 8 MHz HF.
B In all cases, the transmit frequency of a MF/HF console DSC Distress alert: Will always go out first on 2187.5 kHz. Depends upon operator DSC Call set up entries or manufacturer's defaults. Will go out on 8 MHz and 2 MHz and one other DSC Distress frequency. Will always go out first on 8414.5 kHz to ensure receipt by a coast station.
C DSC relays of Distress alerts by vessels: Should be done for all Distress alerts received aboard the ship. Should be transmitted to all ships involved in Distress traffic. Should be avoided; unless an acknowledgement has not been heard. Are the best means to provide for a retransmission of Distress communications.
B Which of the following is not a DSC watch frequency? 2187.5 kHz 2182.0 kHz 6312.0 kHz 12577.0 kHz
C Which of the following channels and modes should be used when initiating a Distress alert transmission? Ch-6 DSC Ch-6 Radiotelephony Ch-70 DSC Ch-13 Radiotelephony and Ch-16 DSC
D How many total frequencies are available for DSC Distress alerting? One Two Five Seven
A Which of the following watches must a compulsory Sea Area A1 vessel maintain? A continuous DSC watch on Ch-70. A continuous DSC watch on 8414.5 kHz plus one other HF DSC frequency. A continuous DSC watch on 2187.5 kHz. A continuous DSC watch on Ch-16.
D Which of the following are the MF/HF DSC Distress watch frequencies 2177.5, 4210.0, 6314.0, 8416.5 12579.0, 16806.5 2182.0, 4125.0, 6215.0, 8291.0, 12290.0, 16420.0 2174.5, 4177.5, 6268.0, 8376.5, 12520.0, 16695.0 2187.5, 4207.5, 6312.0, 8414.5, 12577.0, 16804.5
A How many HF DSC Distress watch channels must be guarded by a compulsory vessel underway. 2 3 4 6
A What is usually the first step for a GMDSS Radio Operator to take when initiating a Distress priority message via Inmarsat? By pressing a "Distress Button" or "Distress Hot Key(s)" on the equipment. By dialing the correct code on the telephone remote unit. By contacting the LES operator and announcing a Distress condition is in existence. By contacting the LES operator using the radiotelephone Distress procedure "Mayday"... etc.
B Which of the following statements is true regarding Distress alerting under GMDSS? Ship to shore Distress alerts are used to alert other ships in port of navigational hazards. The Distress alert should identify the station in Distress and its position & time of position update. Also, the alert may include the nature of the Distress, the type of assistance required, or the course and speed of the mobile unit. Ship-to-ship Distress alerts are used to alert other ships in the vicinity of navigational hazards and bad weather. The vessel nearest to the emergency must notify the Coast Guard before leaving the vicinity.
C If a GMDSS Radio Operator initiates a DSC Distress transmission but does not insert a message, what happens? The transmission is aborted and an alarm sounds to indicate this data must be provided by the operator. The transmission is not initiated and "ERROR" is indicated on the display readout. The transmission will be made with "default" information provided automatically. The receiving station will poll the DSC unit of the vessel in Distress to download the necessary information.
D Repetition of a DSC Distress call is normally automatic if not acknowledged after a delay of: 2 - 5 minutes 10-15 minutes 1 - 2 minutes 3.5 - 4.5 minutes
A For a MF/HF DSC Distress call which statement is false? It always requests J3E Voice follow-on communications so other vessels can hear the Mayday. It will send the minimal necessary information using the "Distress Button" or "Distress Hot Key." It will send a more detailed Distress format if time permits and operator data entries are correctly performed. It does not contain all the information normally of interest in on-scene Distress communications.
B Which statement regarding an MF/HF DSC Distress call is true: Follow on communications should be presumed to take place on the SITOR (NBDP) frequency associated with the specific DSC frequency used. Follow on communications should be presumed to take place on the voice frequency associated with the specific DSC frequency used. An alternate emission and frequency may be specified for follow-up communications by the vessel in the original Distress alert. Both the nature of Distress and the alternate emission and frequency must be specified for follow-up communications in the original Distress alert.
D If a VHF-DSC Distress alert is transmitted what channel is used for follow-on voice transmission? Ch-12 Ch-70 Ch-13 Ch-16
A If a MF-DSC Distress alert is transmitted what frequency is used for follow-on voice transmission? 2182.0 kHz 2760.0 kHz 2187.5 kHz 2174.5 kHz
B If a HF-DSC Distress alert is transmitted what frequency is used for follow-on voice transmission? 8376.5 kHz 8291.0 kHz 8401.5 kHz 8201.0 kHz
C What is the proper format for a Distress follow on voice transmission? (3x is three times), All Ships 3x, this is Ship's Name/Call Sign 3x, Ship's position, nature of distress and assistance requested. Mayday 3x, this is Ship's Name/Call Sign once, Ship's position, nature of distress and assistance requested. Mayday 3x, this is Ship's Name/Call Sign 3x, Ship's position, nature of distress and assistance requested. All Stations 3x, this is Ship's Name/Call Sign 3x, Ship's position, nature of distress and assistance requested.
C What information should be included in a Distress follow on voice transmission after a DSC Alert? Ship's Name and Call Sign, MMSI number, DSC frequency used and any other information that might facilitate rescue. Ship's position, Ship's IMN, the nature of distress and assistance requested. Ship's Name and Call Sign, MMSI number & position, the nature of distress and assistance requested. Ship's Name and Call sign, repeat IMN, provide any other information that might facilitate rescue.
D What information is not vital in a Distress follow on voice transmission after a DSC Alert? Ship's position, nature of distress and assistance requested. Ship's Name, Call Sign and MMSI number. Physical description of the vessel and number of POB. Company emergency contact information.
C Which statement is true regarding the receipt and acknowledgement of actual Distress follow-on communications by GMDSS ship stations? Ship stations in receipt of Distress alert should not defer acknowledgement for a short interval, so that receipt may be acknowledged by the coast station. A Coast station has the sole obligation to respond. A ship station should wait for the Coast station MMSI DSC Acknowledgment before taking action. If a Coast station has no response in 15 minutes the ship should DSC acknowledge and inform the RCC. A ship station that receives a Distress call from another vessel must, as soon as possible, inform the Master or person responsible for the ship of the contents of the Distress communications received. Alerts concerning navigational hazards are second only to Safety traffic.
D What is meant by the acronym "EOS" in a DSC message? Error of Sequence End of Signals Equal Operating Signals End of Sequence
A What is the proper procedure to be followed upon receipt of a Distress alert transmitted by use of Digital Selective Calling techniques? Set watch on the radiotelephone Distress and Safety frequency associated with the Distress and Safety calling frequency on which the Distress alert was received. Set watch on the DSC alerting frequency in the band of frequencies the alert was received. Set a continuous watch on VHF-FM Channel 13, 16 and DSC on Channel 70. Ship stations equipped with narrow-band direct-printing equipment should respond to the Distress alert as soon as practicable by this means.
B What is meant by the acronym "ECC" in a DSC message? Every Cipher Counted Error Check Character Error Cannot Confirm Even Characters Counted
B What action should be taken on receipt of a DSC Distress alert? Read the display screen and/or printout, silence the alarm, always call the Master to confirm the alert is real. Silence the alarm, read the display screen and/or printout and listen for any follow on transmissions. Listen for any follow on voice/TELEX transmission on the appropriate DSC frequency. Silence the alarm, read the display screen and/or printout, only call the Master if the alert is within 500 NM.
C What action should be taken if a Distress alert is received on the 12 MHz DSC frequency? Use DSC to acknowledge/relay the alert using the 12 MHz DSC frequency. Do nothing. Ship is too far away to render assistance. Set the transceiver to 12290.0 kHz simplex J3E emission. Set the transceiver to 12520.0 kHz simplex F1B/J2B emission.
B Your ship received a Distress relay from a coast station on DSC freq. 2187.5 kHz. What action should the watch officer take? Retransmit the DSC call on 2187.5 kHz to other vessels in the vicinity to assist in SAR operations. Monitor 2182.0 kHz to determine if there are any genuine Distress communications. Transmit a voice "Mayday Relay" call on 2187.5 kHz to other vessels in the vicinity. Transmit a voice "Mayday Relay" call on 2182.0 kHz to other vessels in the vicinity.
C Your ship received a Distress relay from a coast station on DSC VHF channel 70. What action should the watch officer take? Retransmit the DSC call on Ch-70 to other vessels in the vicinity. Monitor Ch-06 to determine if there are any genuine Distress communications. Monitor Ch-16 to determine if there are any genuine Distress communications. Transmit a voice "Mayday Relay" call on Ch-13.
D Under what condition would you not relay a DSC Distress alert? If the mobile unit in Distress is incapable of further Distress alert communications. If no Coast Station/Mobile Unit acknowledgement of the alert is observed. No distress traffic has been heard and the DSC alert is unacknowledged via DSC. A coast station DSC acknowledgment of the original Distress alert was received by your vessel.
A The relay of DSC Distress alerts: Can quickly overburden the GMDSS systems in the vicinity with improperly transmitted or inappropriately relayed DSC calls. Was not originally an intended function of the GMDSS system but now is the preferred method to notify an RCC. Remains the preferred method for passing Distress message traffic to an RCC or Coast Station. Should always be done immediately to ensure a Coast Station receives the DSC Distress Alert.
D Transmission of a Distress alert by a station on behalf of another vessel actually in Distress should not occur: When the mobile unit actually in Distress is not itself in a position to transmit the Distress alert. When the Master or responsible person on the mobile unit not in Distress so decides. When the responsible person at the Coast Station determines further help is necessary. When communications between the Distress vessel and a Coast station are already in progress.
A DSC Relays of DSC Distress alerts received from other ships should be done? Only when the original DSC call is not acknowledged and no follow-on Distress traffic has been heard. Only by Inmarsat-C TELEX with Distress priority conveying the follow-on Distress traffic that has been heard. Only by Inmarsat-B voice or TELEX with Distress priority if no follow-on Distress traffic has been heard. Preferably by MF/HF voice or TELEX directly to the RCC conveying the follow-on Distress traffic that has been heard.
A What action should you take after sending a false Distress alert on VHF? Make a voice announcement to cancel the alert on Ch-16. Send a DSC cancellation message on Ch-70. Make a voice announcement to cancel the alert on Ch-13. Make a voice announcement to cancel the alert on Ch-22A.
B What action should you take after sending a false Distress alert on MF? Make a voice announcement to cancel the alert on 2187.5 kHz. Make a voice announcement to cancel the alert on 2182.0 kHz. Make a voice announcement to cancel the alert on 2174.5 kHz. Send another DSC alert and follow on with voice on 2182.0 kHz.
C What action should you take after sending a false Distress alert on MF? Send another DSC alert on 2187.5 kHz. and follow on with voice on 2187.5 kHz. No action is necessary. Make a voice announcement to cancel the alert on 2182.0 kHz. Send a DSC alert on all 7 DSC frequencies and follow on voice on 2174.5 kHz.
D What action should you take after sending a false Distress alert on 8 MHz? Make an "ALL STATIONS" call on all 5 H.F. TELEX channels canceling the alert. Make a "MAYDAY" call on 8414.5 kHz canceling the alert. Make an "Urgency" call on 8614.0 kHz canceling the alert. Make an "ALL STATIONS" call on 8291.0 kHz canceling the alert.
A What action should you take after sending a false Distress alert on 12577.0 kHz? Make an "ALL STATIONS" call on the associated 12 MHz J3E frequency canceling the alert. No action is necessary. Make an "ALL STATIONS" call on all 5 H.F. TELEX frequencies canceling the alert. Send a message to the nearest RCC via Inmarsat canceling the alert.
B What action should you take after sending a false or accidental Distress alert on Inmarsat-C? Press the "Distress Hot Keys" then press the "cancel" key. Select "Transmit" or "SEND/REC" menu and send a cancellation message via the LES used for the Distress alert. Do nothing until the RCC contacts your vessel to determine if the Distress alert was genuine. Select the cancel false Distress alert option in the Distress Setup Menu and re-transmit the call.
D What is the fundamental purpose for imposing radio silence? To ensure that interference to proprietary communications is minimized. To ensure that only voice communications can be effected on the Distress frequency or channel. To ensure that a Distressed vessel will have a "window" twice each hour for transmitting routine messages. To mitigate the risk of interference on a frequency or channel being used for emergency communications.
A When can routine communications be resumed when radio silence has been imposed? Routine communications can resume after the Rescue Coordination Center transmits a message on the frequency or channel being used for emergency communications stating that such traffic has concluded. After determining that the frequency or channel appears to be no longer in use. After determining that geographic distance from the Distress situation will prohibit any other signal from interfering with emergency communications. If, in the master's opinion, communications on that frequency will interfere with emergency communications.
B What is meant by the term "Seelonce Mayday"? Stations remaining off the air to safeguard proprietary information. Stations not directly involved with the on-going Distress communications may not transmit on the Distress frequency or channel. Two three-minute silent periods, at 15 and 45 minutes after the hour that provide a transmitting "window" for distressed vessels to transmit Distress alerts using J3E. Communications on a Distress frequency or channel is banned for 24 hours following the cessation of the Distress traffic.
C How is "radio silence" imposed? By the Land Earth Station (LES) controlling the Distress communications on that frequency. By the nearest Public Correspondence Coast Station. By the On Scene Coordinator (OSC) or the RCC chosen by the SAR Mission Coordinator. By the vessel first responding to the Distress call.
C How are normal working conditions restored on a SITOR (NBDP) frequency on which radio silence had been imposed? The LES that imposed the radio silence must transmit a SITOR (NBDP) message stating "SILENCE FINI". The Public Correspondence Station (PCS) that imposed the radio silence must transmit a narrow band direct printing message on the Distress frequency stating "SILENCE FINI". The RCC or Coast station that imposed the radio silence must transmit a SITOR (NBDP) message stating "SILENCE FINI". The High Seas Service (HSS) that imposed the radio silence must transmit a narrow band direct printing message on the Distress frequency stating "SILENCE FINI".
D How are normal working conditions restored after radio silence has been imposed? All of these answers are correct. The Land Earth Station (LES) that imposed the radio silence must transmit a voice message on the Distress frequency stating "SILENCE FINI". The Public Correspondence Station (PCS) that imposed the radio silence must transmit a voice message on the Distress frequency stating "SILENCE FINI". The Rescue Coordination Center (RCC) that imposed the radio silence must transmit a voice message on the Distress frequency stating "SEELONCE FEENEE".
C The Radiotelephone Urgency signal is: Mayday Securite Pan Pan Seelonce Feenee
D Which of the following situations would normally use the Urgency priority? A crewmember falling over the side. A serious medical situation involving a crewmember with potential loss of life. An important meteorological warning concerning hazardous weather. A cargo shift or weather situation considered to be of greater hazard than would justify a Safety priority designation.
A Which of the following situations would not properly use the Urgency priority? Abandoning the vessel just before sinking. Treatment of a crewmember breaking a leg in a cargo hold. Leaking oil from a minor tank fracture requiring a mandatory pollution report. An unexpected deviation in the forecast track line of a typhoon.
B Which of the following situations would normally use the Urgency priority? A collision with the ship taking on water. A serious medical situation involving a crewmember. Important company communications related to an itinerary change. Scenarios concerning the Safety of navigation or important meteorological warnings.
B The Urgency Priority should be used for: Messages concerning the Safety of Life At Sea (SOLAS). Messages containing information concerning the Safety of a mobile unit or person. Messages detailing important navigational warnings. Messages concerning On-scene communications.
C If the Watch Officer hears "PAN PAN" spoken 3 times it means: A navigation or important meteorological warning should follow. The station is preparing to transmit a Safety message possibly concerning the safety of a mobile unit or person. None of these answers is correct. A mobile unit is in need of immediate assistance.
B When the GMDSS Radio Operator on watch hears "Securite" spoken three times, he can expect to receive the following information: The safety of vessel or person is in jeopardy. A message concerning the Safety of navigation. A vessel is in need of immediate assistance. A coast Station sending an important traffic list.
C Which of the following situations would normally use the Voice designation "Securite"? Messages concerning the Safety of Life At Sea (SOLAS). Messages containing information concerning the Safety of a mobile unit or person. Messages detailing important navigational warnings. Messages concerning On-scene communications.
D Which of the following situations would normally use the Safety priority? Treatment of a crewmember with a broken leg that is not life-threatening. Treatment of a crewmember with a serious cardiac emergency. A fire in the generator flat/spaces. Loss of 5 containers with lashing gear over the side.
A Which of the following situations would normally use the Safety priority? Important navigational or meteorological warnings. A serious medical situation involving a crewmember. An unanticipated warning related to piracy or terrorism. Grounding in a way that could lead to imminent danger to the ship's crew.
D The Radiotelephone Safety signal is: "Safety Safety Safety" "Pan Pan" repeated 3 times "Securite Securite" repeated 3 times "Securite" repeated 3 times
A Which of the following situations would normally use the Safety priority? A scenario concerning an important navigational or meteorological warning. A serious medical situation involving a crewmember. A crewmember falling over the side. Important company communications involving weather routing.
A Which of the following frequencies and modes is allocated for Distress alerting in GMDSS? 406 MHz via EPIRB, 1626.5-1645.5 MHz via Inmarsat and Channel 70 DSC plus six (6) MF/HF DSC frequencies. 1626.5-1645.5 MHz via Inmarsat, VHF CH-16 plus six (6) MF/HF DSC frequencies, 406 MHz via EPIRB. Channel 70 DSC plus six (6) MF/HF DSC frequencies, 7 voice follow-on and 6 telex follow-on frequencies. Mayday on VHF Channel 70 and the other six voice follow-on frequencies.
B Which of the following frequencies is designated for On-scene Distress and Safety communications? 4209.5 kHz 2174.5 kHz 518.0 kHz 490.0 kHz
C Which channel is designated for GMDSS Digital Selective Calling? Ch-06 Ch-16 Ch-70 Ch-83
D How many MF frequencies are available for DSC Distress related calls? Five Four Two One
A How many HF frequencies are available for DSC Distress related calls? Five Four Two One
B How many frequencies are available under GMDSS for DSC Distress-related calls? Six Seven Four Five
D Which of the following steps should be taken, if possible, when the vessel must be abandoned because of a Distress situation? Alert the U.S. Coast Guard by using the survival craft's portable Inmarsat unit. Program the SART and EPIRB to transmit the vessel's location and situation. No additional steps are needed as the SART and EPIRB will both automatically float free and operate properly. Secure the EPIRB to the survival craft and mount the SART in a position to maximize its elevation.
A Which action is the most appropriate action for a GMDSS radio Operator to take in a Distress situation where immediate help is needed, but the vessel is not sinking nor needs to be abandoned? Transmit Distress calls by HF/MF/VHF DSC or Inmarsat. Switch off EPIRB and SART manually. Notify the RCC (Rescue Coordination Center) through VHF FM on channel 13. Transmit Distress calls by activating the radiotelegraph automatic alarm signal.
B DSC is used primarily to: Receive weather warnings, navigational notices and other Maritime Safety Information. Transmit and receive Distress, Urgency and Safety alerts and routine calls to and from other ships and coast radio stations. Provide routine communications with the ship owner. Report ship's position to search-and-rescue authorities via satellite.
C GMDSS vessels equipped for Sea Areas A2, A3 or A4 must maintain a continuous DSC watch on 2187.5 kHz. Only in areas beyond Inmarsat coverage. Only outside of areas covered by VHF-DSC. At all times when underway. When directed to do so by a cognizant rescue authority.
C Which statement is true regarding Distress communications under GMDSS? Distress communications by (SITOR) NBDP should be in the BFEC mode when in two-way communications with the Coast Guard or other coast radio stations. The Rescue Coordination Center may not appoint another station to coordinate Distress traffic relating to the incident. The Rescue Coordination Center (RCC) is responsible for controlling a search and rescue operation, will coordinate the Distress traffic relating to the incident and may appoint another station to manage the Distress traffic. Initial Distress communications by (SITOR) NBDP should be in the ARQ mode to broadcast them to the Coast Guard, other coast radio stations and other ship stations.
D When operating in coastal waters (sea area A1), a GMDSS-equipped vessel must: Maintain a continuous DSC watch on 8514.5 kHz. Maintain a continuous aural watch on 2182.0 kHz. Maintain a continuous DSC watch on VHF channel 16. Maintain a continuous DSC watch on VHF channel 70.
C What indication is given to the personnel in a survival craft of the approach of SAR craft? The Satellite EPIRB will change its strobe light pattern to indicate radar interrogation. The SART informs survivors when the SART switches to the "standby" mode. The SART may provide a visual or audible indication of interrogation by a 3-cm radar. The AIS SART will alarm to indicate that SAR craft with radars are getting close.
D Which of the following would most likely not prevent a SART's signal from being detected? The rescue personnel were monitoring the 3-cM radar and the SART was mounted improperly in the lifeboat. The SART was mounted improperly in the survival craft and rescue personnel were monitoring the 10-cM radar. The rescue personnel were monitoring the 10-cM radar and the SART was properly mounted in the lifeboat. The SART was properly mounted in the lifeboat and rescue personnel were monitoring the 3-cM radar.
A How can a SART's detection and effective range be maximized? The SART should be held or mounted as high as possible and in a vertical position. The SART should be placed in water immediately so it will begin transmitting. Switch the SART into the "high" power position. If possible, the SART should be mounted horizontally so that its signal matches that of the searching radar signal.
B Which statement is NOT true regarding the SART? Responds to interrogations by a vessel's X-Band radar and transmits a signal. This is a 6 GHz transponder capable of being received by a vessel's X-band navigational radar system. This is a 9 GHz transponder capable of being received by a vessel's X-band navigational radar system. Transmits a distinctive 12-blip signal for easy recognition.
B At what point does a SART begin transmitting? It immediately begins radiating when placed in the "on" position. If it has been placed in the "on" position, it will respond when it has been interrogated by a 9-GHz radar signal. It must be manually activated or water activated before radiating. If it has been placed in the "on" position, it will begin transmitting immediately upon detecting that it is in water.
C A SART's signal cannot be detected: In poor visibility, or at night. In heavy seas. By a search vessel's 10 cm Radar. By a search vessel's 3 cm Radar.
B How does the searching vessel's radar interrogate a survival craft SART? Activate the IFF interrogation system. The SART responds automatically and transmits the 12-blip signal when it detects the search craft or other vessels' X-Band radar signal. Maintains watch on VHF-FM ch-70 for the SART's unique identifier. The SART responds automatically when it detects the search craft or other vessel's 10-cm radar signal.
C What radar display changes indicate the correct approach to a SART and what care should be taken in a SAR situation? The line of dots indicate the SART's position, the dots become increasing arcs as the distance to the SART lessens, rescuing vessels should increase speed to reach Distress more quickly. A line of dots on a radar screen rotates to indicate the SART's position along its line of bearing; rescuing vessels should steer for the center of the line of dots. The line of dots indicate the SART's position, the dots become increasing arcs as the distance to the SART lessens, rescuing vessels should reduce speed as the arcs get greater in degree. The line of dots indicate the SART's position, the dots become decreasing arcs as the distance to the SART lessens, rescuing vessels should reduce speed as the arcs lessen in degree.
D How can rescue personnel detect that a SART is transmitting in the immediate vicinity? The DSc unit will react to the SART's signal and respond with the two-tone auto alarm. The SART can provide an approximate location to within a two nautical mile radius, per IMO standards. The SART signal appears as a target which comes and goes; the effect of heavy swells on a SART. The SART's dots on the PPI will become arcs and then eventually become concentric circles.
A What signal is detected as originating from an AIS SART and how is the signal displayed? An AIS SART signal is shown on any AIS receiver as a special MMSI-like coded symbol. The 3-cm radar reflections are converted to AIS signals and displayed on EcDIS/ARPA screens. An AIS SART transmits on AIS frequencies and the signals are converted to 3-cm radar targets for display on 3-cm radars. An AIS SART transmits on 9 GHz so that a 3-cm radar can display the signals.
D How can vessel personnel detect the operation of a SART in its vicinity? A unique two-tone "warbling" signal heard on VHF-FM ch-70. It will activate an AIS new signal alarm on the AIS receiver. The SART signal appears as a target that comes and goes--due to the effect of heavy swells on a SART. A unique 3-cm signal consisting of a 12-dot pattern radiating outward from a SART's position along its line of bearing.
A What is not an advantage of an AIS SART signal when compared to a radar-based SART signal? The AIS SART can be detected much farther away than radar SART models. Not every AIS transmission needs to be received to achieve an accurate presentation of the location. The AIS SART position has GPS accuracy and transmits on AIS VHF frequencies. AIS SART units may be easier to find in poor radar target conditions.
A Which of the following statements concerning testing and maintenance of SARTs is true? Testing a SART should be done in a consistent manner & location to ensure a baseline history of proper results. Testing of the SART should never be done in port to prevent interference to other vessel's radars. A SART's battery must be replaced within ninety (90) days after the expiration date imprinted on the unit. An at-sea GMDSS maintainer is not able to test a SART because it is hermetically sealed.
B Why is it important to limit the duration of testing a SART? Excessive testing causes "burn in" on the vessel's radar display. Testing in port or even at sea may cause interference to other radars or a test signal may be misinterpreted as a genuine Distress situation. To prevent overheating, a SART requires sufficient ventilation that is significantly reduced when the SART is being tested. If another SART is testing at the same time, the two signals will cause damage to the unit that transmitted them.
C What statement is true regarding tests and maintenance that could be provided for the SART? Full verification within manufacturer's specifications by the on-board maintainer would be a requirement for all vessels in the A3 & A4 sea areas using measuring equipment to generate 9 GHz signals. Battery should be replaced within the 90 day grace period following the manufacturer's expiration date shown on the SART and the SART should only be tested at-sea to reduce interference to other vessels. Extreme care should be exercised because testing of the SART may be received by other vessels, may be interpreted as a Distress condition, or it may interfere with other vessels' safe navigation. Battery should be replaced with a new one before the manufacturer's expiration date shown on the SART and the SART should only be tested in port to reduce interference to other vessels.
D Why should functional testing of a SART be minimized? Potential interference with safe navigation, notifying other vessels of an actual Distress and minimize power consumption. Minimize power consumption of the battery and only test at sea to reduce potential interference or confusion. Possibility of misinterpretation by other vessels as a Distress situation and only test in port to prevent potential interference with safe navigation or at-sea vessels. Potential interference with safe navigation, possible misinterpretation of an actual Distress, minimizes draining the battery.
A Which is NOT a valid maintenance and testing function for a SART? Operational test with several vessels to determine effective transmitting range. Inspection of container for apparent damage. Inspect battery expiration date and the lanyard condition. Brief operational test utilizing own ship's radar.
B The SART is required to have sufficient battery capacity to operate in the stand-by mode for what period of time? Three days Four days Eight hours Forty-eight hours
D Which is not a function of a satellite under COSPAS-SARSAT using satellite EPIRBs? Relayed satellite message includes the EPIRB ID number which provides a reference for retrieval of vessel information from the shore database. Doppler shift of EPIRB signal is measured and the EPIRB's position is calculated. Information received from EPIRBs is time-tagged and transmitted to any Local User Terminal in the satellite's view. After the EPIRB's position is calculated using the Doppler shift COSPAS-SARSAT satellites provide follow-on SAR communications.
A Which of the following satellite systems is of particular & dedicated importance to search and rescue missions under GMDSS? COSPAS/SARSAT Inmarsat GPS Iridium
B Which of the following statements concerning COSPAS-SARSAT is false? 406 MHz EPIRBs are units that are used as alerting devices. Doppler frequency measurements provide more precise locations than GPIRB signals. The Doppler frequency measurement concept is used to determine the EPIRB's location. Satellites in a low-earth polar orbit detect EPIRB beacons on 406 MHz and relay the information to a Local User Terminal (LUT).
C Which of the following statements concerning COSPAS-SARSAT is false? EPIRBs, ELTs, and PLBs use the system primarily for Distress alerting. These satellites monitor 406 MHz for EPIRB signals. After initiating a call request and selecting the LES, these satellites may be used for commercial messages. These satellites use Doppler shift measurement to determine the location of the beacons.
C Which of the following statements concerning the EPIRB system is true? GOES weather satellites will provide alerting with complete worldwide coverage. COSPAS-SARSAT satellites always provides an alert and position report within 10 minutes of reception. The Inmarsat system will not provide alerts and position report for 406 MHz EPIRBs equipped with GPS receivers. The GPS satellite system will relay an alert and position report within 20 minutes of reception.
D Which of the following statements concerning satellite EPIRBs is true? The coded EPIRB signal identifies the nature of the Distress situation. The coded EPIRB signal only identifies the vessel's name and port of registry. If the GMDSS Radio Operator does not program the EPIRB, it will transmit default information such as the follow-on communications frequency and mode. Once activated, these EPIRBs transmit a signal for use in identifying the vessel and for determining the position of the beacon.
C What features may be found on GMDSS satellite EPIRB units? Strobe light, Distress homing transmission on 406 MHz, float-free release bracket. Emergency transmission on 406 MHz, hydrostatic release, AIS homing frequency. Float-free release bracket, strobe light & Distress alert transmission on 406 MHz. Hydrostatic release, Distress alert transmission on 121.5 MHz, strobe light.
D What feature is not a component of a 406 MHz satellite EPIRB? 121.5 MHz emergency homing transmitter. Emergency transmission on 406.025 MHz. Float-free release bracket. Aural locator signal.
A What statement is true regarding 406 MHz EPIRB transmissions? Transmits a unique hexadecimal identification number. Allows immediate voice communications with the RCC. Coding permits the SAR authorities to know if manually or automatically activated. GMDSS Radio Operator programs an I.D. into the SART immediately prior to activation.
B Which of the following is normally part of 406 MHz satellite EPIRBs? A strobe light, automatic float-free bracket, 1-watt 406-MHz alert beacon. A 5-watt 406-MHz alert beacon, Automatic Hydrostatic Release (ARM), strobe light. Automatic float-free bracket, 5-watt 121.5 MHz homing beacon, strobe light. Automatic Hydrostatic Release (ARM), 1-watt 121.5 MHz alerting beacon, strobe light.
B Which of the following statements concerning EPIRBs is false? The COSPAS-SARSAT system may take a full hour or more to provide an alert. The Inmarsat system provides worldwide coverage for Distress alerts. The GOES weather satellites are in a geostationary orbit. 406 MHz EPIRB units may be equipped with GPS receivers.
C Which of the following EPIRBs is most likely to be used to transmit a Distress alert signal? S-Band EPIRBs X-Band EPIRBs 406 MHz EPIRBs 121.5/243 MHz EPIRBs
B Which of the following would best be used for visual detection of a distressed vessel? A 9-GHz SART's beacon. An EPIRB's strobe light. A 121.5-MHz EPIRB beacon. A 406-MHz EPIRB beacon.
C Which piece of required GMDSS equipment is the primary source of transmitting locating signals? Radio Direction Finder (RDF). Survival Craft Transceiver. An EPIRB transmitting on 406 MHz. A SART transmitting on 406 MHz.
D What may be used as a homing signal by the search and rescue vessels in the immediate vicinity of the ship in Distress? Flare gun Strobe Light 406 MHz signal from a satellite EPIRB. A 121.5 MHz emergency transmitter in a satellite EPIRB.
A What part of a satellite EPIRB may function as a visual aid to rescue vessels? Strobe light A 121.5 MHz emergency transmitter in a satellite EPIR 406 MHz signal from a satellite EPIRB. Loud beeping tone emitted by the unit, once activated.
D What is an example of a locating signal? SSB phone traffic Ship to shore transmissions Loran C A float-free EPIRB
A Which device provides the best method to locate a ship in Distress or survival craft in the GMDSS? Satellite EPIRBs Radio Direction Finder MF/HF DSC VHF homing device
A With what other stations may portable survival craft transceivers not communicate? Communication between the ship and survival craft transceivers ashore. Communication between the ship and its survival craft. Communication between rescue units and survival craft. Communication between multiple survival craft and with aircraft.
B Equipment for radiotelephony use in survival craft stations under GMDSS must have what capability? Operation on 457.525 MHz. Operation on Ch-16. Operation on 121.5 MHz. Operation on Ch-70.
C Equipment for radiotelephony use in survival craft stations under GMDSS must have what characteristics? Permanently-affixed antenna, watertight, power 1W or 25W. Watertight, power a minimum of 1W, operation on CH-16, Ch-13 & Ch-70. Operation on Ch-16, watertight, permanently-affixed antenna. Operation on Ch-16, Ch-13 & Ch-70, power 1W, permanently-affixed antenna.
D Which statement is NOT true regarding the requirements of survival craft portable two-way VHF radiotelephone equipment? Operation on Ch-16 Antenna must be permanently-affixed. Simplex (single frequency) voice communications only. Effective radiated power should be a minimum of 2.0 Watts.
A Which statement is NOT true regarding the requirements of VHF Survival Craft Transceivers? Operation on Ch-13 is mandatory. Effective radiated power should be a minimum of 0.25 Watts. Simplex (single frequency) voice communications only. Operation on Ch-16 is mandatory.
B Which statement is NOT true regarding the requirements of survival craft portable two-way VHF radiotelephone equipment? Watertight to a depth of 1 meter for 5 minutes. Operates simplex on Ch-70 and at least one other channel. Effective radiated power should be a minimum of 0.25 Watts. The antenna is fixed and non-removable.
D Which of the following has been designated for "On-scene" communications in GMDSS? Ch-24 Ch-2182 Ch-70 Ch-16
A Which of the following channels is designated as the VHF follow-on communications channel and is required in all portable survival craft equipment? Ch-16 Ch-6 Ch-13 Ch-70
B Which of the following frequencies have been designated for "On-scene" communications in the Global Maritime Distress and Safety System? VHF Ch-22 VHF Ch-16 and SITOR (NBDP) on 2174.5 kHz. HF radiotelephone on 21.820 MHz. SITOR (NBDP) on 2177.0 kHz and VHF Ch-16.
C Which of the following frequencies have NOT been designated for GMDSS "On-scene" or SAR communications? VHF Ch-16 MF radiotelephony on 2182.0 kHz SITOR (NBDP) on 2182.0 kHz HF radiotelephony on 4125.0 kHz
C "On-scene" communications would best be represented by? Using Inmarsat-C "hot-key" function Sending DSC alert on VHF Ch-70 SITOR (NBDP) on 2174.5 kHz SITOR (NBDP) on 4125.0 kHz
D For "On-scene" communications, vessels in Distress and SAR Aircraft should use? VHF Ch-70, 4125 kHz J3E, 5680 kHz J3E VHF Ch-70, 4125 kHz J2B, 5680 kHz J3E VHF Ch-16, 4125 kHz F1B, 3023 kHz J3E VHF Ch-16, 4125 kHz J3E, 3023 kHz J3E
C Which action should the GMDSS radio operator take in a Distress situation when embarking in survival craft? EPIRB and SART switched on manually prior to embarking; remain aboard vessel in Distress. Notify RCC (Rescue Coordination Center) through VHF DSC in portable equipment. Switch on EPIRB and SART immediately and leave on. Communicate via Inmarsat-C from the survival craft.
D Which of these would be vital to a GMDSS SAR situation in polar regions? GOES satellites to receive Distress Alerts and HF Voice for follow-on and SAR activity. GOES satellites to receive Distress Alerts and Inmarsat Voice for follow-on and SAR activity. Inmarsat satellites to receive Distress Alerts and HF Voice for follow-on and SAR activity. COSPAS/SARSAT satellites to receive Distress Alerts and HF Voice for follow-on and SAR activity.
A Which statement is not true regarding the COSPAS-SARSAT system? The position of the EPRIB is always transmitted in the outgoing transmission (unless the unit is a GPIRB). Signals received by low altitude, near-polar orbiting satellites are relayed to a ground receiving station (LUT). Doppler shift is used to locate the position of the EPIRB. EPIRBs are satellite beacons used as alerting & homing devices.
B Which statement is NOT true regarding the COSPAS-SARSAT system? EPIRBs are satellite beacons used as alerting/locating devices. May be used to transmit public correspondence. Locates Distress beacons transmitting on 406 MHz. Doppler shift is used to locate the beacons.
B What information is transmitted by a 406 MHz EPIRB alert? Vessel position and nature of Distress. A unique Hexadecimal I.D. number. Vessel name and identification. Vessel MMSI number and position.
C Which statement is false regarding the COSPAS-SARSAT system and EPIRB operations? The EPIRB's position is calculated by the system and passed to the MCC. The EPIRB transmits a unique Hex I.D. and vessel position that may be passed to the RCC. The EPIRB's position and Hex I.D. is passed instantaneously to the RCC. The EPIRB transmits a unique Hex I.D. that is passed to the RCC if it cannot be determined to be inadvertent by the MCC.
B What actions should the GMDSS radio operator take prior to any potential Distress situation? Create a table or chart of all the DSC coast stations that might be used during the vessel's itinerary. All of these answers are good operational practice and should be consistently done. Prepare a detailed Distress message file on both satellite & MF-HF SITOR (NBDP) equipment containing all information needed in a Distress so it will be available for last-minute editing. Ensure all LES choices are correct and then updated properly as the vessel transits different SAR jurisdictions.
C What information should be contained in a detailed Distress message that was not transmitted by an initial Distress "hot-key" alert? Vessel position, course & speed and the nature of Distress. The distress vessel's IMN and position at the time of alert. Vessel name & call sign, POB and all potential means to communicate with the vessel. Vessel name & call sign, distress vessel's IMN & vessel position.
D Which GMDSS equipment is best suited to simultaneous long-range communications with an RCC/coast station and OSC vessels or SAR aircraft? MF-HF SITOR (NBDP) transmitters using telex follow on frequencies. Inmarsat Signals routed via the RCC to the SAR vessels & aircraft under their control. VHF transmitters to reach SAR aircraft and OSC vessels as well as the RCC/coast station. MF-HF SSB transmitters using voice follow-on frequencies.
A Which statement is NOT true regarding an Inmarsat Distress Alert? USCG coast stations will receive the alert and immediately notify the correct RCC. The operator selection of LES will determine which associated RCC will receive the alert. If the operator selects an invalid or inoperative LES code the NCS for that service will intercept the call and reroute the alert. If the LES choice is not updated properly the Distress Alert might be routed to a non-optimum RCC, introducing delays and confusion into the Distress situation.
D What are the best resources for researching and planning equipment setups and updates prior to any potential Distress situation? NGA Pub. 117, Inmarsat handbook or manufacturer's equipment manuals. Inmarsat handbook, FCC Part 80 or ITU List of Coast stations. ITU List of Coast stations, IMO GMDSS handbook, FCC Part 80. Inmarsat handbook, NGA Pub, 117 or ITU List of Coast stations.
A Which statement is true regarding Inmarsat "hot-key" Distress Alerts? The LES programmed by the watch officers into the Distress Alert Update menu determines which RCC will receive your initial Distress Alert. The vessel's position is checked against the SAR jurisdictions and the proper LES updated as the vessel changes NAVAREAS. The GPS position updates the Distress Alert Update menu to the correct LES choice to ensure proper communications with an RCC. The Distress Alert defaults are set correctly by the manufacturer and then automatically updated.
A Which of the following control selections may result in limited receiving range? Setting the squelch control to its maximum level. Setting the squelch control to its minimum level. The power switch is set to the "high" output position, resulting in receiver overloading. Setting the channel selection switch midway between channels 6 and 16.
B At mid-day, what would be the best choice in attempting to communicate with a shore station 15 miles distant? 16 MHz band 156-164 MHz band 12 MHz band 22 MHz band
C Which of the following factors does not normally affect the range of VHF transmissions? Salt water ingress into the antenna coaxial cable. Power level setting. Ionospheric refraction. Vessel antenna height.
D Much longer than normal VHF communications distances are typically caused by: changing power from 1W to 25 W. Skywave reflections from the D layer. Ionospheric activity in layers F1/F2. Atmospheric ducting or tropospheric propagation.
A Describing VHF transmissions as "line of sight" does not mean: VHF communications are effective only with nearby stations within visual range of the bridge. Vessel antenna height will not affect the radius of propagation. The normal transmission range to a coast station is approximately is 10 NM. coast station antenna height has no effect on the radius of transmission.
B The effectiveness of VHF communications is maximized by: The adjustment of squelch for maximum receiver sensitivity, setting transmitter power to 1W & selecting an appropriate channel. Appropriate setting of the transmitter power, selecting an appropriate channel & adjustment of squelch for maximum receiver sensitivity. Selecting an appropriate channel, adjustment of squelch for minimum receiver sensitivity & setting transmitter power to 1W. Selecting an appropriate channel, adjustment of squelch for minimum receiver sensitivity, setting transmitter power to 25W.
D A VHF frequency channel pair of TX 157.200 MHz and RX 161.800 MHz would most likely be: A VTS frequency for VTS - Ship communications. A simplex Public Correspondence Coast Radio Station frequency. A simplex Private Coast Radio Station frequency. A duplex Public Correspondence Coast Radio Station frequency.
A Which channel is utilized for the required Bridge-to-Bridge watch? VHF-FM on Ch-13 in most areas of the continental United States. DSC on Ch-70 VHF-FM on Ch-16 The vessel's VHF working frequency.
B While conducting routine communications using the wheelhouse VHF with a station 1 mile distant, your recommended power setting would be: 25 watts after dark. 1 watt, day or night. 25 watts during a clear sunny day. 1 watt using DSC at night.
C The USA-INT control on VHF units: Selects duplex operations for U.S. coastal waters and simplex operations in non-U.S. waters, on the "alpha" channels. Ensures that the "alpha" channels are correctly set to duplex for use in U.S. waters & on VTS channels. Changes selected international duplex channels to simplex channels for use in U.S. waters, on the "alpha" channels. Changes selected international simplex channels to duplex channels for use in U.S. waters, on the "alpha" channels.
C The USA-INT control on VHF units: Was made necessary by a desire for more duplex channels in the U.S. Correctly set, will result in duplex operations in U.S. Coastal waters on the "alpha" channels. Correctly set, will result in simplex operations in U.S. Coastal waters on the "alpha" channels. Was made necessary by a desire to convert simplex international channels to duplex channels in the U.S.
D Proper and legal VHF operations require all of these except? The channel must be designated as valid for the nature or type of communications desired. Simplex, duplex and alpha channel modes must be correctly selected. The power level must be appropriately chosen by the operator. The correct bandwidth must be selected by the operator.
C How is mutual interference on 518 kHz among NAVTEX stations avoided? All stations transmit at the same time but stations are limited to daytime operation only to reduce the radius of propagation. Transmitter power is limited, station assignment codes are not shared by other NAVAREAS and stations alternate between daytime and nighttime operations. Transmissions scheduled on a time-sharing basis, power limited and station assignment codes are geographically separated. Station codes are not shared by other NAVAREAS, transmissions scheduled on a time-sharing basis and power is limited.
D When do NAVTEX broadcasts typically achieve maximum transmitting range? Local noontime Afternoon Sunset Middle of the night
A What should a GMDSS Radio Operator do if a NAVTEX warning message is received but it contains too many errors to be usable? Do nothing. Vital NAVTEX messages will be repeated on the next scheduled broadcast. contact the NAVAREA coordinator and request a repeat broadcast. Initiate a request for category A, B, L and D messages. Listen to appropriate VHF weather channel for repeat warnings.
B Which of these cannot happen when a paper model NAVTEX receiver runs out of paper? The unit is unable to print messages and all subsequent MSI broadcasts may be missed until the paper is replaced. The system will automatically change from receiving MSI by NAVTEX to receiving it by SafetyNETTM so that no messages will be lost. It may give off either an audible and/or visual alarm. MSI messages may be missed because the unit cannot print them out.
B Which of the following is the primary frequency that is used exclusively for NAVTEX broadcasts internationally? Typically 50-100 nautical miles (90-180 km) from shore. 518 kHz 4209.5 kHz VHF channel 16 when the vessel is sailing in Sea Area A1, and 2187.5 kHz when in Sea Area A2.
C What is the transmitting range of most NAVTEX stations? Typically 50-100 nautical miles (90-180 km) from shore. Typically upwards of 1000 nautical miles (1800 km) during the daytime. Typically 200-400 nautical miles (360-720 km). It is limited to line-of-sight or about 30 nautical miles (54 km).
B How is a NAVTEX receiver programmed to reject certain messages? The transmitting station's two-digit identification can be entered to de-select reception of its broadcasts. By choosing a message category's single letter (A-Z) identifier and then deselecting or deactivating. By entering the SELCAL of the NAVTEX transmitting station. By pressing "00" in the transmitter's ID block.
C How can reception of certain NAVTEX broadcasts be prevented? Stations are limited to daytime operation only. Coordinating reception with published broadcast schedules. The receiver can be programmed to reject certain stations and message categories. Automatic receiver desensitization during night hours.
D Which of the following statements is true? No NAVTEX receiver can be programmed to reject category A, B, D and L messages since they are mandatory to be received via NAVTEX. Upon entering a new NAVTEX station's broadcast range, the GMDSS Radio Operator enters the station's SELCAL number. The GMDSS Radio Operator can select the "None" option in the message category menu. A GMDSS Radio Operator may choose to program certain NAVTEX receivers to reject category A, B, D and L messages if they are being received by another MSI system.
A What means are used to prevent the reception of unwanted broadcasts by vessels utilizing the NAVTEX system? Programming the receiver to reject certain stations and message categories. Operating the receiver only during daytime hours. Coordinating reception with published broadcast schedules. Automatic receiver de-sensitization during night hours.
D What statement is true regarding the control the operator can exercise over the NAVTEX receiver's operation? The operator can set the unit to automatically reject any and all categories of messages if the ship desires to not receive them. Upon entering a coastal area for the first time, the operator enters code KK to indicate "ready to receive NAVTEX". To reduce the number of messages, the operator can select code 00 to indicate "not in coastal passage". The operator can set most units to reject all messages except navigation, meteorological warnings, and search and rescue messages. If the unit will reject such messages it may be unsafe to do so.
A Which messages are mandatory to be received and should not typically be rejected or disabled by the operator of a NAVTEX receiver? Navigational warnings, meteorological warnings, SAR information. Meteorological warnings, SAR information, Pilot Service Messages. Meteorological warnings, meteorological forecasts, navigational warnings. SAR information, navigational warnings, ice reports.
A The NAVTEX message header contains the following? The first letter (from A to Z) indicates the NAVTEX transmitting station. A two-digit number (01-99) indicates the NAVTEX message category. Message numbers include a date/time group, along with the transmitting station's numerical ID. None of these answers is correct.
B If the Inmarsat-c terminal is inoperative but the vessel remains within NAVTEX coverage -- which of the following message categories should not be disabled by the GMDSS Radio Operator? Navigational warnings, meteorological warnings and metrological forecasts. Meteorological warnings, Search and Rescue information and Navigational warnings. Search and Rescue information, navigational warnings and other electronic navaid messages. Search and Rescue information, Meteorological warnings and ice reports.
C How are NAVTEX broadcasts transmitted? NAVTEX is transmitted by commercial coast radio stations following their traffic lists. NAVTEX is transmitted only when an Urgency or Distress broadcast is warranted. Using FEc techniques. No more often than every two hours and should immediately follow the radiotelephone silent periods.
D What determines whether a NAVTEX receiver prints a particular type of message content from a programmed NAVTEX station? The serial number and type of message have already been received but additional printouts are generated to ensure receipt aboard the vessel. B, The subject indicator has been programmed for rejection by the operator but the message contains a priority override print command. The transmitting station ID covering your area has been programmed for rejection by the operator or has not been previously received. The serial number and type of message has not been previously received or the subject indicator has not been programmed for rejection.
A Which information determines if a NAVTEX message is to be rejected? The second letter (from A to Z) in the header indicating the type of message. Transmitter identity (numerals from 1 to 26 identifying transmitting station within the NAVAREA). The Answerback of the receiving station has not been entered in the NAVTEX receiver. Only messages having a serial number 00 are rejected.
B NAVTEX broadcasts are sent: Immediately following traffic lists. In categories of messages indicated by a single letter or identifier. On request of maritime mobile stations. Regularly, after the radiotelephone silent periods.
D Where NAVTEX cannot be feasibly established, what system can be implemented to provide an automated service in coastal waters to receive MSI? AMVER VHF DSC ARQ SITOR (NBDP) SafetyNETTM
A What action should a GMDSS Radio Operator take when SafetyNET™ Distress or Urgency messages are received by the vessel's EGC receiver? Aural and/or visual alarms are activated and require manual deactivation. No immediate action is required, as an audible tone will be generated at the beginning and end of the transmission and a paper printout of the message will be generated. No immediate action is required by the operator, since the transmission will be automatically acknowledged by the receiving vessel. A periodic alarm tone will be heard until the radio operator prints the message from the unit's memory.
B What system can provide an automated service in coastal waters where it may not be feasible to establish the NAVTEX service or where shipping density is too low to warrant its implementation? AMVER SafetyNETTM VHF DSC ARQ SITOR (NBDP)
C Aboard ship, SafetyNET™ messages can be received by which equipment/methods? VHF DSC on the weather channels. NAVTEX Receiver on 518 kHz or the Tropical Navtex frequency. EGC receiver of the vessel's Inmarsat-C SES. HF SITOR (NBDP) MSI frequencies.
C SafetyNET™ messages can be received by which of the following shipboard equipment? NAVTEX MF and HF SITOR (NBDP) Inmarsat-C EGC receiver Inmarsat F77 EGC receiver
D Maritime Safety Information is promulgated via satellite through which system? AMVER NAVTEX Inmarsat-M SES SafetyNETTM
C SafetyNET™ promulgates what type of information? Traffic Lists News advisories MSI MARAD
D What kind(s) of broadcasts are not available through SafetyNET™? MSI and messages to specific geographic areas. Storm warnings Distress and Urgency bulletins Vessel traffic lists
A Which satellite system promulgates Maritime Safety Information? Inmarsat-C SafetyNETTM AMVER NAVTEX Inmarsat-M SES
B What information is promulgated by the international SafetyNET™? Traffic Lists MSI Priority Messages MARAD
B To receive all mandatory MSI using the SafetyNET™ system the vessel must: Notify the NAVAREA coordinator you are using SafetyNET™ for the receipt of MSI (Maritime Safety Information). Log-in and ensure the position is accurate to receive MSI for the NAVAREA the vessel is currently within. Set the receiver to your destination Inmarsat Ocean Region. Notify the NAVAREA coordinator you are using SafetyNETTM for the receipt of MSI (Maritime Safety Information) and set the receiver to your destination Ocean Region.
C In using SafetyNET™ for the receipt of MSI (Maritime Safety Information): Only unscheduled Urgency and Distress messages will be received if the Inmarsat-C SES is not logged in. Both scheduled MSI and unscheduled Urgency and Distress messages will be received if the Inmarsat-C SES is logged in. All of these answers are correct. The Inmarsat-C SES must have Enhanced Group Calling (EGC) capability to receive MSI.
B Over what system are Enhanced Group Calls transmitted? COSPAS satellite Inmarsat satellite HF SITOR (NBDP) shore stations NAVTEX shore stations
C How is a MSI (Maritime Safety Information) broadcast received by an Inmarsat-C SES that is engaged in communications? The broadcast message is missed and the Radio Operator must request a retransmission. The broadcast message is stored in the EGC memory and will automatically be printed at the conclusion of the ongoing traffic. There is no loss of information since broadcasts of "vital" messages will be repeated. The radio operator can request retransmission of messages missing from numeric serial number succession.
D Which of the following provides a unique automated system capable of addressing messages to pre- determined groups of ships or all vessels in both fixed and variable geographic areas? NAVTEX AFRTS NAVAREAs EGC
A What system may be useful for messages, such as local storm warnings or a shore-to-ship Distress alert, for which it is inappropriate to alert all ships in the satellite coverage area? EGC NAVTEX AMVER DSC
D What services are available through Enhanced Group Calls? Maritime Safety Information and vessel traffic lists. Hourly NOAA weather broadcasts from the NWS. Coastal weather broadcasts. Maritime Safety Information and messages to pre-defined groups of subscribers.
A What messages originate from registered information providers anywhere in the world and are broadcast to the appropriate ocean region via a LES? SafetyNETTM messages AMVER broadcasts Urgency messages NAVTEX broadcasts
A Which HF SITOR (NBDP) mode would be selected to receive MSI broadcasts from high seas shore stations? FEC AM RTTY ARQ
B The U.S. Coast Guard communications station providing HF MSI broadcast coverage for NAVAREA IV is: NOJ (Kodiak) NMF (Boston) NMC (San Francisco) NMO (Honolulu)
C The U.S. Coast Guard communications station providing HF MSI (Maritime Safety Information) broadcast coverage for NAVAREA XII is: NMA (Miami) NMF (Boston) NMO (Honolulu) NMR (San Juan)
D Frequencies for receiving HF MSI (Maritime Safety Information) are: The same as used for NAVTEX The same as used for contact a Coast Radio Station using FEC Specified HF voice frequencies Specified HF SITOR (NBDP) frequencies
A Which frequency/mode is authorized for use internationally for Maritime Safety Information transmissions? 4209.5 kHz using FEC mode 4209.5 kHz using ARQ mode 4125.0 kHz using simplex mode 4125.0 kHz using FEC mode
B How many frequencies are assigned specifically for HF MSI broadcasts? 6 8 5 7
D Which NAVAREA is associated with the western North Atlantic and the Caribbean Sea? NAVAREA X NAVAREA XI NAVAREA XII NAVAREA IV
A Which sequence is associated with the 5 new Arctic Ocean NAVAREAs? NAVAREA XVIII, NAVAREA XX, NAVAREA XXI NAVAREA III, NAVAREA VII, NAVAREA XV NAVAREA IV, NAVAREA XII, NAVAREA X NAVAREA XII, NAVAREA X, NAVAREA XI
B NAVAREAs referred to in NAVTEX are the same as used in __________. GMDSS sea areas Inmarsat SafetyNETTM International Vessel Traffic Service Inmarsat ocean regions
C A vessel operating in the Western Atlantic or along the East coast of North America and Central America from Canada to Venezuela, including the Caribbean and Panama, would be located in which NAVAREA? X XI IV XIII
C A vessel operating in the Eastern Pacific or along the West coast of North and Central America from Alaska to Ecuador, including Panama and Hawaii, would be operating in which NAVAREA? X XI XII IV
D A vessel on a voyage between Miami and Los Angeles via the Panama Canal would be operating in which NAVAREAS? II and III IV and V V and VI IV and XII
C Which of the following actions should be taken once the vessel is berthed and will not leave port again for several weeks? The GMDSS Radio Operator must notify the NCS that the vessel will be off-line, and wait for the NCS to acknowledge with a confirmation number that must be logged. The Inmarsat-C system can be powered down without taking additional steps once the GMDSS Radio Operator has ensured that all incoming SafetyNETTM messages have been received and stored. The GMDSS Radio Operator may log out of the Inmarsat-C system and turn the power off (unless the vessel decides to leave the unit on during the port stay.) The GMDSS Radio Operator must transmit an all-ships alert, to notify vessels within the satellite's footprint that the vessel will be off-line.
D What action should always be taken before powering down an Inmarsat-C terminal or leaving one satellite footprint for another? An Inmarsat-C system must never be powered down or mandatory MS1 messages will be lost. Send a message to the NCS advising arrival in port or request the NCS log your terminal in with the new satellite. No action is required -- the terminal will automatically log in with the new satellite when the NCS common channel is detected or after power up. Log out with the current NCS to inform them you are off the air or to enable a proper log in procedure with the new satellite.
A With most Inmarsat-C systems what should the indicator lamps do when powering up? All lamps should illuminate in a particular sequence, as per the operator's manual. The power on lamp should illuminate. Other lights remain off until a message is received. All lamps should light and stay illuminated. All lamps should light except the RED light.
B What is the importance of a successful log in indication, after power-up, on an Inmarsat-C terminal? The antenna Azimuth and Elevation controls are correctly adjusted. The terminal is enabled for routine incoming and outbound traffic. The receiver gain is properly adjusted for maximum signal. Unscheduled EGC MS1 messages can no longer be received.
B On an Inmarsat-C system, what is the importance of a successful "SYNC" indication after power up? The system is not yet locked on to the NCS signal until a log-in command is performed. Sufficient signal strength on the NCS common channel is being received. There is company telex traffic being received on the NCS common channel. The NCS has confirmed log-in status is on so routine traffic can be sent and received.
C On an Inmarsat-C system an incoming EGC alarm sounds: When first powered on and when receiving Distress traffic. When receiving Distress traffic and all of the BALD MS1 messages. To draw the operator's attention to an unscheduled Distress or Urgency message. To indicate the loss of NCS CC sync -- preventing the reception of unscheduled EGC messages.
B Which satellite(s) would most likely be selected for use when the vessel is operating off the eastern shore of the United States? IOR AOR-W POR Either AOR-W or IOR will work.
C Which satellite should be chosen when operating in the Eastern Gulf of Mexico? POR IOR AOR-W AOR-E
D Which longitude corresponds to the AOR-W satellite for Inmarsat-B/C communications? 64E 178E 15.5W 54W
A Which longitude corresponds to the AOR-E satellite for Inmarsat-B/C communications? 15.5W 64E 178E 54W
D Which longitude corresponds to the POR's satellite location for Inmarsat-B/C communications? 64E 15.5W 54W 178E
A Which longitude corresponds to the IOR's satellite location for Inmarsat-B/C communications? 64E 178E 15.5W 54W
A Which action must be taken to ensure that incoming message traffic of all priority levels will be received through Inmarsat-C? The GMDSS Radio Operator must log-in to the desired satellite (if the unit did not automatically do so.) No additional action is necessary after turning on the receiver and aiming the antenna at the desired satellite. The system needs only to be commissioned and turned on. The GMDSS Radio Operator must log-in to the desired satellite and receive the message reference number (MRN) from the LES.
B When logging into the Inmarsat system using Inmarsat-C, it is necessary to: Enter your IMN Select the Ocean Region. Enter the LES answer back. Call the LES and inform them that you are now operating in the appropriate ocean region.
C What action should be taken on changing from one ocean region to another? Power the system down and turn the power back on again. Manually realign the antenna. Log out of the current satellite and log in to the correct satellite. No action is required -- the unit will scan for another satellite and log in.
D What is the primary danger of improperly logging out? The vessel will be barred in the future from sending traffic through an LES. The NCS ensures that improper log-out procedures do not occur. Scheduled MSI may not be available through the Inmarsat-C terminal. Your company or another ship may accrue significant charges when their traffic is repetitively sent to your unavailable terminal.
A Which of these can take place on an Inmarsat-C terminal that has synch with the NCS CC but has not yet performed a successful login? Reception of unscheduled MSI Urgency and Distress messages. Transmission of synoptic weather reports and company traffic. Reception of the mandatory scheduled BALD MSI messages. Reception of company traffic related to ship's business.
B Which of the following is not an example of a failure to log-out properly? Turning off the power prior to logging out with the NCS. A message on the screen or printer from the NCS. Sailing the vessel into a shadowing or local RF interference situation before logging out. Sailing out of the footprint of a satellite before logging out.
D What is the primary function of an NCS? To provide direct communications between the Inmarsat station placing a call and the station receiving the call. To provide multi-mode communications between the Inmarsat station placing a call and the coast radio station that will deliver it. To determine which satellite is best suited to provide communications between the Inmarsat station placing a call and the station receiving the call. To monitor and control communications through the Inmarsat satellite for which it is responsible.
A What is the primary function of a LES? To provide direct communications between the Inmarsat station placing a call and the station receiving the call. To monitor and control communications through the Inmarsat satellite for which it is responsible. To provide multi-mode communications between the Inmarsat station placing a call and the coast radio station that will deliver it. To determine which satellite is best suited to provide communications between the Inmarsat station placing a call and the station receiving the call.
B Messages are transmitted by an Inmarsat LES according to what criteria? First In, First Out Priority Last In, First Out Serial Number
C How is maximum coverage provided by satellites in the maritime satellite service? Four satellites in polar orbit. Four satellites in geo-stationary orbit for each Inmarsat Service (B, C, F77 and M). Four satellites in geo-stationary orbit approximately 22,184 miles above the equator. Through coordinated use of COSPAS-SARSAT satellites.
C What is meant by the characters GA+ on an Inmarsat terminal? Go ahead for store & forward telex operations. The instruction to "give address". Go ahead for live telex operations. General Address (to all stations).
D What is the purpose of a CODEC? Noise and echo-canceling used in TELEX operation. To enable Distress communications. To enable data communications. To convert analog voice signals to digital transmissions.
C What is an MRN? Mobile Registration Number, provided by the FCC. Mobile Registration Number, provided by IMO. Message Reference Number, provided by the LES. Vessel's call sign
D To keep the Inmarsat-B or F77 antenna pointing at the desired satellite, regardless of the ship's position and course, it has an input from the vessel's: Operational radar Automated Radar Plotting Aid (ARPA equipped radar) Steering control system Gyrocompass
A The Inmarsat telephone and TELEX communications channel usage scheme is: One ship per telephone channel and many ships per TELEX channel. Many ships on the same TELEX analog channel and many ships on the same TDM telephone channel. One ship per analog telephone channel and one ship per TELEX analog channel frequency. One ship per channel whether telephone or TELEX.
B What is an "Inmarsat Mobile Number"? This identifies the vessel's selective calling (SELCAL) number. This is the Inmarsat number that is assigned to a unit for incoming or outgoing calls. This is the vessel's Inmarsat registration number for accounting authority purposes. This number is used for receiving news and other optional services in FleetNETTM.
B Which of the following statements concerning Inmarsat geostationary satellites is true? They are in a polar orbit, in order to provide true global coverage. They provide coverage to vessels in nearly all of the world's navigable waters. They are in an equatorial orbit, in order to provide true global coverage. Vessels sailing in equatorial waters are able to use only one satellite, whereas other vessels are able to choose between at least two satellites.
C What is the primary purpose of an Inmarsat LES? The LES is required for ship-ship Inmarsat communications, ship-shore communications are handled by terrestrial Coast Radio Stations. The LES monitors all operations of the satellite and assigns channels and frequencies to the vessel. The LES makes the satellite connection between the vessel and the requested destination for all types of services. The LES is dedicated only to processing all vessel Distress priority calls.
B How is a signal radiated from an Inmarsat-B or F77 system's antenna? It is usually radiated in an omni-directional pattern, but an optional feature allows it to be directional for use when the vessel is on the fringe of the satellite's footprint. It is a highly focused directional signal that must be beamed at the desired satellite. It is radiated in an omni-directional pattern. It is radiated in an omni-directional pattern that can be reversed by the Operator to attain directional beaming to an alternate satellite.
C Which mode of Inmarsat-B communications may be possible with a lower received signal strength? Fax Voice communications TELEX Binary computer file transfers
D What is the purpose of the second 1.D. in an Inmarsat-B or F77 SES? To provide an alternate number which may be called if a busy signal is received by the calling party. To provide an additional speech path, which may be used to communicate while the first channel is engaged in active communications. To provide for an emergency working frequency. To provide an additional number which may be dedicated to computers, fax, etc.
A What is the effect of having five periods (e.g. .....) in the text of a TELEX transmission on an Inmarsat-B SES? The transmission will automatically terminate after those characters are transmitted. Only the first period will be routed to the receiving party. Only the first two periods will be routed to the receiving party. This will automatically trigger the reversal of charges to the receiving party.
D Why is the automatic answerback request (WRU) first used by the LES after an Inmarsat-B TELEX call has been placed from a ship? Let the LES operator know the printer is functioning correctly. Get the identity of the ship station and start the channel assignment process. Verify that there is a good connection with no transmission errors. 1dentify the SES making the request.
A When engaging in voice communications via an Inmarsat-B or F77 terminal, what procedures are used? CODECs are used to digitize the voice signal. Noise-blanking must be selected by the operator. The voice signal must be compressed to fit into the allowed bandwidth. The voice signal will be expanded at the receiving terminal.
A Which mode of communications is NOT possible through an Inmarsat-C SES? Shore-to-ship Facsimile Data TELEX Emergency Activation
B What is the average length of time required for a TELEX sent by Inmarsat-C to be delivered to the addressee? All Inmarsat-C communications are made with real-time connectivity so there is no delay in message delivery. The average delivery time for a message sent by Inmarsat-C is about 10 minutes. Date/time notification of delivery is possible only through Inmarsat-B. The average delivery time for a TELEX sent by Inmarsat-C is about 10 minutes, but fax and data messages sent by Inmarsat-C require about 30 minutes for delivery.
C How is a signal radiated from an Inmarsat-C system's antenna? It is a highly focused directional signal that must be beamed at the desired satellite. It is usually radiated in an omni-directional pattern, but an optional feature allows it to be directional for use when the vessel is on the fringe of the satellite's footprint. It is radiated in an omni-directional pattern. It is radiated in an omni-directional pattern that can be reversed by the Operator to attain directional beaming to an alternate satellite.
D What statement is true regarding Inmarsat-C? There is a propagation delay, but a direct connection is made between the ship and shore users. There are delays in establishing communications. Then a direct real-time connection is maintained with the other party. The TELEX message is stored until the mailbox is accessed by the station desiring to retrieve their message. This is a store and forward network, with an intermediate step that means there is no direct connection between ship and shore users.
A With an Inmarsat-C LES, how are messages routed to receiving stations? All messages are forwarded via a store and forward network. Direct connections are made to the receiving stations via gateways. Intermediary stations are used to connect the sending station with the receiving station in a real-time mode. Messages are stored until the network is polled by the receiving station.
B What are the directional characteristics of the Inmarsat-C SES antenna? Highly directional parabolic antenna requiring stabilization. Omni-directional. Wide beam width in a cardioid pattern off the front of the antenna. Very narrow beam width straight-up from the top of the antenna.
D Which of the following best describes Inmarsat-C operation? 1s an analog-based system. Requires a stabilized directional antenna. Provides for voice, TELEX, high and low-speed data and compressed video communications. 1s a digital store-and-forward system that also provides Enhanced Group Call, data reporting, polling and Distress alerting capabilities.
A Which of the following best describes a shipboard Inmarsat-C system? A small, lightweight terminal capable of providing satellite store-and-forward message communications. A satellite communications system that provides real-time connectivity. A small, lightweight terminal used to transmit messages over high frequency (HF) bands to communicate through a satellite. A satellite communications system that also provides continuous Digital Selective Calling coverage for all ocean regions.
B Which of the following modes of communications are available when using Inmarsat-C? Fax reception TELEX and e-mail 14400 BPS Data Voice transmissions
C Which mode of communication is possible through an Inmarsat-C SES? SITOR (NBDP) Radiotelephone TELEX and e-mail DSC
C It is possible to transmit all of the following via Inmarsat-C from a vessel except? TELEX Text for delivery by fax. Voice x.400 data services
D Which of the following best describes the full range of services provided by the Inmarsat-C Satellite system? Polling, enhanced group call, and one-way position and data reporting via satellite. FM voice communications via satellite. Two-way messaging and data communications on a store-and-forward basis. Polling, enhanced group call, one-way position and data reporting via satellite, two-way messaging and data communications on a store-and-forward basis.
C Which statement concerning Inmarsat-B and Inmarsat-C terminals is correct? Both Inmarsat-B and Inmarsat-C units are capable of fax and voice communications. Inmarsat-B units are not capable of data communications, but Inmarsat-C units are capable of data communications. Both Inmarsat-B and Inmarsat-C units can send data as well as send messages to fax machines. None of these answers is correct.
D When Inmarsat-B and Inmarsat-C terminals are compared: Inmarsat-B antennas are larger, but omni-directional, while Inmarsat-C antennas are smaller and parabolic, for aiming at the satellite. Inmarsat-B antennas are parabolic and smaller for higher gain, while Inmarsat-C antennas are larger but omni-directional. Inmarsat-C antennas are smaller, but omni-directional, while Inmarsat-B antennas are parabolic for lower gain. Inmarsat-B antennas are larger, but directional for higher gain, while Inmarsat-C antennas are smaller and non-parabolic, and do not require aiming at the satellite.
A Which statement concerning Inmarsat-B and Inmarsat-C terminals is correct? Inmarsat-B terminals require AZ/EL setup and gyro input, in order to enable automatic satellite tracking. Inmarsat-B terminals require gyro and GPS input, in order to enable automatic satellite tracking. Inmarsat-C terminals require only GPS input, in order to enable automatic satellite tracking. Inmarsat-C terminals require AZ/EL setup and GPS input, in order to enable automatic satellite tracking.
B When Inmarsat-B and Inmarsat-C terminals are compared: Inmarsat-C antennas are smaller, with active parabolic antennas but no rewind capability. Inmarsat-C antennas are smaller, with passive non-parabolic antennas but no rewind capability. Inmarsat-B antennas are larger, with passive non-parabolic antennas that require rewind capability. Inmarsat-B antennas are larger, with stationary parabolic antennas but no rewind capability.
B Which statement concerning Inmarsat-B and Inmarsat-C terminals is correct? Both Inmarsat-B and Inmarsat-C units are subject to shadowing effects due to their omni-directional antennas. Both Inmarsat-B and Inmarsat-C units are subject to shadowing effects, but Inmarsat-B units have directional antennas. Both Inmarsat-B and Inmarsat-C units are subject to shadowing effects, but Inmarsat-C units have directional antennas. Both Inmarsat-B and Inmarsat-C units are subject to shadowing effects, due to their directional antennas.
C When Inmarsat-B and Inmarsat-C terminals are compared: Inmarsat-B units provide greater communications capabilities, with the benefits of greater size, weight, installation expense and initial cost. Inmarsat-C provides lesser communications capabilities, with the trade-offs of greater size, weight, installation expense and initial cost. Inmarsat-B units provide greater communications capabilities, with the trade-offs of greater size, weight, installation expense and initial cost. Inmarsat-C units are of smaller size, weight, installation expense and initial cost and provide greater communications capabilities due to modern technology.
B Which Inmarsat Earth stations would be available for Inmarsat-C traffic if the vessel is off the Pacific Coast of the United States but logged-in to the AOR-W satellite? EIK (Norway), Tangua (Brazil), Thermopylae (Greece) Southbury (USA), Burum (Netherlands), or EIK (Norway). Santa Paula (USA), Psary (Poland), Yamaguchi (Japan). Beijing (PRC), Fucino (Italy), Nudol (Russia).
C Which Inmarsat Earth stations would be available for Inmarsat-B traffic if the vessel is off the Atlantic Coast of the United States and tracking the AOR-E satellite? Fucino (Italy), Thermopylae (Greece) or Haiphong (Vietnam). Santa Paula (USA), Beijing (PRC), Sentosa (Singapore). Southbury (USA), Burum (Netherlands), or EIK (Norway). Haiphong (Vietnam), Burum (Netherlands), or EIK (Norway).
D Which LES should a GMDSS Radio Operator select to update a Distress alert message if the vessel is in the southern Pacific Ocean near the Dateline? Santa Paula (USA) or Beijing (PRC) would be the best choice depending on SAR jurisdiction. Kumsan (South Korea) or Sentosa (Singapore) would be the best choice depending on SAR jurisdiction. Yamaguchi (Japan) or Nakhoda (Russia) would be the best choice depending on SAR jurisdiction. Auckland (New Zealand) or Santa Paula (USA) would be the best choice depending on SAR jurisdiction.
A Which Inmarsat Earth stations could a GMDSS Radio Operator select for routine Inmarsat-C traffic if the vessel is in the Sea of Japan and logged-in to the POR satellite? Beijing (PRC), Santa Paula (USA), Auckland (New Zealand) or Sentosa (Singapore). Auckland (New Zealand), EIK (Norway), Sentosa (Singapore) or Beijing (PRC). Yamaguchi (Japan), Southbury (USA), Pune (India) or Haifa (Israel). Southbury (USA), Thermopylae (Greece), Psary (Poland) or Tangua (Brazil).
D Which Inmarsat Earth Stations could a vessel utilize for Inmarsat-B traffic if operating off the Pacific Coast of the United States and tracking the POR satellite? Lakhdaria (Algeria), Yamaguchi (Japan) or Sentosa (Singapore). Southbury (USA), Burum (Netherlands), EIK (Norway). Kumsan (South Korea), Burum (Netherlands), or Nakhoda (Russia). Santa Paula (USA), Auckland (New Zealand) or Yamaguchi (Japan).
A Which Inmarsat Earth Stations could a vessel utilize for Inmarsat-C traffic if operating off the Atlantic Coast of the United States and synched to the AOR-W satellite? Southbury (USA), Burum (Netherlands), Eik (Norway). Santa Paula (USA), Aussaguel (France) or Eik (Norway). Southbury (USA), Santa Paula (USA) or Yamaguchi (Japan). Southbury, (USA), Nakhoda (Russia) or Beijing (PRC).
A A vessel is tracking the AOR-W satellite. To initiate an automatic ship-to-shore telephone contact to a shoreside party in the U.S. phone # 123-456-7890, via Southbury Earth Station, a valid Inmarsat operations/dialing sequence is? Select LES 001# then dial 0011234567890# Select LES #002 then dial 11234567890# Select LES 104# then dial 11234567890+ Select LES 001+ then dial 123 4567890#
B A vessel is tracking the AOR-E satellite. To initiate an operator assisted ship-to-shore telephone contact to a shoreside party in the U.S.A., # 202-456-7890, through a European Earth Station, a valid Inmarsat operations/dialing sequence would be? Select LES: 001# then dial 1112024567890 Select LES: 012# then dial 1112024567890# Select LES: 001# then dial 0012024567890# Select LES: 001+ then dial 202 4567890
C To request medical assistance from Inmarsat via voice (using an Inmarsat-B SES) what would be the correct procedure? Enter 36# and then the phone number for the Inmarsat medical advisory system. Enter 32# and then the phone number for the Inmarsat medical advisory system. Enter 32# to request the Inmarsat medical advisory system. Enter 32+ to request the Inmarsat medical advisory system.
D Which of the following would be a valid sequence to request an automatically dialed ship-ship call to an F77 terminal on a vessel tracking the IOR satellite? 00583430662888# 00870430662888# 00583763240864 # 00870763240864#
A If you are tracking the POR satellite and wish to communicate by voice using LES Santa Paula with another ship tracking the AOR-E satellite what is the proper procedure? Select LES: 001# then dial 00870323972310# Select LES: 004# then dial 11582430315036# Select LES: 001# then dial 00851636824323# Select LES: 003+ then dial 00581430326430+
B Which of the following is a correct dialing sequence for a vessel tracking the AOR-W satellite to send a FAX to a U.S. destination via Southbury LES -- using an Inmarsat-B terminal and charging it to a credit card? 003# 1112125365110# 001# 3615036943254# 001+ 110243674932+ 001# 0015036684376#
D Which key is used to indicate the end of a selection in a TELEX call via an Inmarsat-B SES? * ENTER # +
A Which number, if typed at an Inmarsat-B SES by a vessel in the Western Atlantic, would result in placing an automatic TELEX call to a subscriber number of 123456? 00230123456+ 00230123456# 0012131234567# 00 1 213 1234567+
B Which of the following numbers is appropriate for an Inmarsat-B terminal placing a TELEX call via Southbury to another ship logged-in to the AOR-E? 001+ 00584436824246+ 001+ 00581436824246+ 002+ 00584436824246+ 013+ 00581436824246+
C You are on a ship in the Pacific ocean. What keyboard entry would you make on an Inmarsat-B terminal to obtain a TELEX connection via Santa Paula to another ship that is 300 miles west of San Francisco? 001# 00582430353680# 003+ 00582430353680+ 001+ 00582336657450+ 002+ 00582336850450+
C What keyboard entry would you make on an Inmarsat-B terminal for an automatic TELEX connection to 882419 in Nebraska, USA (TELEX country code 230)? 11230882419+ 00230882419# 00230882419+ 230882419
D Which of the following is a correctly formatted Inmarsat-B address for sending communications to a shoreside TELEX terminal number 440122 in the United Kingdom (TELEX country code 51)? 51440122 (51)440122+ 51440122# 51440122+
C From an Inmarsat-C terminal, which of the following are correctly formatted address for sending TELEX messages to two vessels in the AOR-W? First to an Inmarsat-B and second to an Inmarsat-C. ( 870366719020 first and 870436671929 second. ( 584436671929 first and 584366719020 second. ( 584366719020 first and 584436671929 second. ( 58136671920 first and 58136671929 second.
D Which of the following is a correctly formatted Inmarsat-C address book entry for sending TELEX communications to a vessel in the AOR-E? ( 870436772983 ( 571436772983 ( 581323500120+ ( 581323500120
A Which of the following is a correctly formatted Inmarsat-C address book entry for sending TELEX communications to a vessel in the POR? ( 582436559121 ( 870436559121 ( 582436559121+ ( 583436559121+
B Which of the following is a correctly formatted Inmarsat-C address book entry for sending TELEX communications to a vessel in the IOR? ( 853446323868 ( 583446976519 ( 582446323868 ( 870446976519
B Which of the following is a correctly formatted Inmarsat-C address book entry for sending TELEX communications to an Inmarsat-B terminal in the AOR-W? ( 584765044177 ( 584331014013 ( 870331014013 ( 584331014013+
C Which of the following is a correctly formatted Inmarsat-C address book entry for sending TELEX communications to two vessels in the POR - the first Inmarsat-C and the second Inmarsat-B? ( 583452998777 first and 583323500120 second. ( 582450302113 first and 582761579051 second. ( 582452998777 first and 582323500120 second. ( 582600267098 first and 582450302113 second.
B Which of the following is a correctly formatted Inmarsat-C address book entry for sending communications to a shoreside TELEX terminal number 45992 in Taiwan (TELEX country code 769)? ( 76945992+ ( None of these answers is correct. ( 769 45992+ ( (769)45992
C Which of the following is a correctly formatted Inmarsat-C address book entry for sending communications to a shoreside TELEX terminal number 440122 in the United Kingdom (TELEX country code 51)? ( 51440122+ ( (51)440122 ( 51440122 ( 51440122#
D If your vessel is in the POR, which of the following is a correctly formatted Inmarsat-C address book entry for sending communications to a shoreside TELEX terminal number 42267 in Ecuador (TELEX country code 308)? ( 58230842267 ( 30842267+ ( (582)30842267 ( 30842267
A Which of the following is a correctly formatted Inmarsat-C address book entry for sending communications to a shoreside terminal number 276992 in New Jersey via TRT (TELEX country code 238)? ( 238276992 ( (238)276992 ( 238276992# ( 238276992+
D If your vessel is in the IOR, which of the following is a correctly formatted Inmarsat-C address book entry for sending communications to a shoreside TELEX terminal number 77829 in the Philippines (TELEX country code 758)? ( 75877829+ ( 87375877829 ( 58375877829 ( 75877829
A If your vessel is in the AOR-E, which of the following is a correctly formatted Inmarsat-C address book entry for sending communications to a shoreside TELEX terminal number 776424 in Canada via the TWX system (TELEX country code 26)? ( 26776424 ( 58126776424 ( 582776424 ( 26776424+
A Which of the following are correctly formatted Inmarsat-C address book entries for sending communications to two fax machines on vessels in the AOR-W? First is an F77 terminal and second an Inmarsat-B terminal. ( 870768790319 and then 870334600340 ( 8704336837925 and then 870452998777 ( 584768790319 and then 584334600340 ( 1 870 768790319 and then 1 870 334600340
B Which of the following is a correctly formatted Inmarsat-C address book entry for sending communications to a fax machine number 516-229-4339 in Long Beach, CA, U.S.? ( 015162294339# ( 15162294339 ( 015162294339 ( 1 516-229-4339
C Which of the following is a correctly formatted Inmarsat-C address book entry for sending communications to a fax machine on a vessel's Inmarsat-B terminal in the AOR-E? ( 581366269025 ( 870466269025 ( 870323972514 ( 581761138138
D Which of the following is a correctly formatted Inmarsat-C address book entry for sending communications to a fax machine number 1424-8821-902 in the United Kingdom (voice country code 44)? ( 44 1424-8821-902 ( 4414248821902# ( 44+ 14248821902+ ( 4414248821902
A Which of the following is a correctly formatted Inmarsat-C address book entry for sending communications to a fax machine on a vessel's F77 terminal in the IOR? ( 870763240864 ( 583442519372 ( 1870442519372 ( 870323500121
B Which of the following is a correctly formatted Inmarsat-C address book entry for sending communications to a fax machine number (045) 334-5678 in Japan (voice country code 81)? ( 810453345678# ( 810453345678 ( 81(045)3345678 ( 81 (045) 334-5678
D How may an Inmarsat-C unit be used to send text communications to an Internet destination? Call up the file menu, insert the internet address in the first line of the file and use the SEND/REC or TRANSMIT menu to send the message. Use the SEND/REC or TRANSMIT menu, compose a text file and use the file attach function in the file menu. Internet addresses cannot be reached from an Inmarsat-C terminal without a separate e-mail computer. Compose a text file, use the SEND/REC or TRANSMIT menu & specify the recipient using the address book function (or by manual entry) for e-mail.
A To send message traffic to an Internet address using an Inmarsat- C terminal -- what action would be required? Select the recipient using the e-mail code from the address book (or manually select the code.) Insert the Internet address into the correct field when creating the message file. Select the recipient using the e-mail code into the DNID/ENID setup e-mail function. Insert the Internet address into the correct field of the System setup e-mail function.
B What menu function in Inmarsat-C terminals would typically be used to specify an Internet address destination? The System Setup menu to store all required e-mail destinations. The Address book function (or manual entry) in the SEND/REC or TRANSMIT menu. The Data Report menu to initiate the SEND/REC or TRANSMIT process. The menus for File Operations are used for adding the Internet address to the file.
C Which of the following is not correct with regard to Inmarsat-C e-mail? It is possible to send e-mail messages from your ship's Inmarsat-C terminal to any e-mail address in the world. To receive e-mails from shore, your ship's e-mail address must be registered with an Inmarsat C Land Earth Station Service Provider. An additional hardware upgrade is required to enable your ship's Inmarsat-C terminal to send and receive e- mails. Not all land earth station service providers support e-mail service.
C How would a notification of arrival be sent as a text to a mobile phone number using an Inmarsat-C unit? Mobile phone numbers cannot be reached by Inmarsat-C terminals because they lack voice capability. Use the Inmarsat-C voice handset to compose and transmit the text. Compose a text file, use the SEND/REC or TRANSMIT menu & specify the recipient using the address book function (or by manual entry) for SMS destinations. The mobile phone number is inserted as the first line of the text file and sent with the SEND/REC or TRANSMIT menu.
D How may Inmarsat-C terminals be used for sending Urgency or Safety priority communications (such as synoptic weather reports)? Urgency & Safety EGC messages can only be received by an Inmarsat-C terminal. Enter PAN PAN or SECURITE into the heading of the message file so the message will be routed by priority. Only Routine or Distress priority messages can be sent from an Inmarsat-C terminal. Use the appropriate special 2-digit codes selected from the address book or manually enter the correct code into the SEND/REC or TRANSMIT menu.
C Which statement is false regarding an Inmarsat Distress request? The NCS in each ocean region automatically monitors the processing of such calls by other LESs in that region and processes calls if any anomaly exists in the system. If all satellite channels are busy then one of them will be preempted by a Distress request. Any Distress request is automatically switched to an Inmarsat Distress working frequency. Any request message with Distress priority is automatically recognized by the LES and a satellite channel is instantly assigned.
D In what way(s) may a Distress message be initiated through Inmarsat? By transmitting the Distress message on the U.S. Coast Guard's dedicated monitoring channel. By adding the word "Distress" in the first line of the message's preamble. All Inmarsat units must use a dedicated key that can be pressed for immediate action. By pressing dedicated key(s) for this purpose or using menu-driven features to produce a Distress priority message.
A How is an Inmarsat Distress priority message ordinarily initiated on board the vessel? By pressing one or more dedicated "Distress key" controls on the equipment. By dialing the correct code on the telephone remote unit. By contacting the LES operator and announcing a Distress condition is in progress. By contacting the LES operator using the radiotelephone Distress procedure "Mayday"... etc.
B What must be done to disconnect an incoming TELEX message when voice Distress communications is desired via Inmarsat-B? Momentarily turn off the system power. Depressing the "Distress" button automatically disconnects incoming messages. It is necessary to wait until the incoming TELEX message has concluded otherwise a system fault will occur. Depress the "Reset" switch to disconnect incoming message.
B Why is the correct selection of the LES in the Distress Alert Setup menu so crucial? Each LES has its own dedicated RCC whose jurisdiction is defined by the LES selected. The LES choice determines the nationality of the RCC which will handle the Distress traffic and perform SAR duties or that will pass the particulars to the RCC with jurisdiction to coordinate the SAR situation. Inmarsat terminals use the GPS position to automatically determine the optimum LES choice in a Distress situation. The vessel need not select the LES because the NCS will always intercept a Distress priority call and route it to the closest RCC.
C If an Inmarsat Distress Alert Setup menu contains an incorrect or unavailable LES code what happens to the Distress Alert transmission? The NCS updates the LES codes in the Distress Alert Setup menu as the vessel moves from ocean region to ocean region to ensure this does not happen. The NCS will send a Distress Alert Setup incorrect LES code alarm so the alert can be corrected and re- transmitted. The NCS will step in and route the Distress Alert transmission to an RCC. Ifthe LES code is incorrect then the Distress Alert transmission is not detected by any RCC.
B What action would be the swiftest and most certain way to notify a RCC of a Distress situation aboard your vessel? Compose a detailed Distress message and send it to the closest LES to optimize the SAR response. Confirm the information in the Distress Alert Message setup menu is correct and use the "hot-key" or Distress Message Generator function on your satellite terminal. Press all of the "hot keys" available in the GMDSS console to ensure the RCC is notified. Send a multi-frequency DSC alert to ensure the RCC is notified regardless of Ionospheric propagation conditions.
C What information should be sent in a detailed satellite Distress message that was not included in the initial "hot-key" Distress Alert? Name/Call sign of vessel, vessel position, vessel description, all vessel 1.D. numbers, LES code for Distress traffic. POB, vessel description, all vessel I.D. numbers, shore contact numbers, vessel position. Name/Call sign of vessel, POB, vessel description, all vessel 1.D. numbers and shore contact numbers. Name/Call sign of vessel, nature of Distress, POB, vessel description and all vessel 1.numbers.
D What actions should be taken to transmit a detailed satellite Distress message to a RCC handling your vessel's initial "hot-key" Distress Alert? Compose a detailed Distress message, attach it to the Distress Alert Setup menu and re-transmit the "hot- key" Distress Alert. Call up and edit a pre-saved detailed Distress message, select the closest LES and press the "hot-key." Compose a detailed Distress message, use the Transmit or SEND/REC menu and send it to both the RCC and to your company emergency contact number. Call up and edit a pre-saved detailed Distress message, use the Transmit or SEND/REC menu, select Distress priority and choose the best LES for the situation.
A What is your vessel's obligation upon receipt of a Distress priority EGC message requesting that vessels report in response to a SAR situation? Silence the alarm, notify the master and send a message to the RCC with your vessel's position and capabilities. Read the Distress EGC message and if the situation is too far away no response is required. Read the Distress EGC message and await instructions from the RCC as how to proceed to the distress. Silence the alarm, notify the Master and await instructions from the RCC as to whether your vessel is requisitioned or released from participation in SAR activity.
D What statement is true regarding LES codes and Inmarsat "hot-key" alert default menus? Automatic or manual updating of the LES code only needs to be done when switching to a different ocean region satellite. The SAR jurisdiction software automatically updates the "hot-key" menu based on the vessel's position. The LES code is determined by the NAVAREA based on the vessel's current position. The LES code must be updated by the officers depending upon the SAR jurisdictions the vessel transits and current ocean region satellite in use.
A Detailed Distress situation report message formats can best be found in? IAMSAR manual Volume 111. ITU list of RCC and Coast stations. AMVER chapter of NGA PUB 117. FCC Title 47, Part 80.
A Upon receipt of SafetyNET™ messages of the Distress or Urgency category on the ship's EGC receiver, what action is required by the GMDSS Radio Operator? Manually reset the alarm. No immediate action is required as an audible tone will be generated at the beginning and end of the transmission and a paper printout of the message will be generated. No immediate action is required by the operator since the transmission will be automatically acknowledged by the receiving vessel. A periodic alarm tone will be heard until the radio operator prints the message from the unit's memory.
B What can be defined as the service that allows terrestrial information providers to send general information messages to pre-defined groups of subscribers? SafetyNETTM FleetNETTM COSPAS-SARSAT InfoNET
C What additional equipment provides the maximum availability for receiving SafetyNET™ broadcasts when the associated Inmarsat-C SES is engaged in communications? An integrated EGC receiver with the existing Inmarsat-C equipment. HF SSB can be used to receive voice MSI broadcasts. A separate EGC receiver. Automatic switching between Inmarsat-C and EGC functions.
D Which of the following is utilized to transmit Enhanced Group Calls? COSPAS satellite HF SITOR (NBDP) shore stations NAVTEX shore stations Inmarsat satellite
A What is the equipment arrangement that provides the maximum availability for reception of MSI broadcasts when using Inmarsat-C for TELEX communications? Separate EGC receiver. Integrating EGC receiver with the existing Inmarsat-C equipment. Redundancy using HF SSB to receive voice broadcasts. Automatic switching between Inmarsat-C and EGC functions.
B Which of the following statements concerning EGC configuration is false? NAVAREA selection should be monitored and appropriately updated. The originator of MSI information cannot specify receipt only by vessels within a specific geographical area, circular or rectangular. The originator of MSI information can specify receipt only by vessels within a specific geographical area, circular or rectangular. GMDSS operators generally can select additional NAVAREAS to acquire EGC messages of interest to the vessel.
D A vessel with an 18-hour ETA to the Panama Canal on a voyage from Miami loses the ability to communicate via Inmarsat. The most likely cause is? The vessel has sailed beyond the coverage area of the Southbury Land Earth Station. The vessel has sailed beyond the coverage area of the Eastern Atlantic satellite. The vessel has sailed beyond the coverage area of the Western Atlantic satellite. An equipment fault resulting in a loss of signal from the satellite.
A A vessel, before transiting the Panama Canal, on a voyage from San Diego to Miami, loses the ability to communicate via Inmarsat. The most likely cause is: The vessel has sailed beyond the coverage area of the Pacific satellite. The vessel has sailed beyond the coverage area of the Southbury Shore Station. The vessel has sailed beyond the coverage area of the Western Atlantic satellite. The satellite orbit is beyond the usable range of the SES.
B What can be the visual observation of a condition whereby it may be possible to initiate a reliable Inmarsat TELEX transmission but a voice transmission may not be possible? An indication on a meter or on the display terminal of high transmit power. An indication on a meter or on the terminal of low receiver signal strength. An indication of a very high antenna elevation on the display terminal screen. An indication that the antenna has reached its maximum travel in one direction.
C What maintenance function may the holder of a GMDSS Radio Operator license perform, or supervise the performance of, on an Inmarsat-C SES? Adjust the station's E1RP (power output ) for improved signal levels. Enter Azimuth and Elevation values correctly so the Inmarsat-C SES can find the NCS CC. Remove stack deposits and other debris from the antenna to prevent degraded performance. D. Adjust a reference oscillator or synthesizer to tune the unit to the NCS CC. Adjust a reference oscillator or synthesizer to tune the unit to the NCS CC.
C What maintenance function may the holder of a GMDSS Radio Operator license perform, or supervise the performance of, on an Inmarsat-C SES? Painting the antenna dome. Adjust the station's EIRP. Adjust any front panel controls. Adjust a reference oscillator or synthesizer.
D Which functions may the holder of a GMDSS Radio Operator License NOT perform on the Inmarsat-C equipment? Maintain the antenna clear of soot, paint, etc. Log-on, traffic and log-off functions. Entry of position data and selection of LES. Optimize performance by adjusting the transmitter EIRP.
C What immediate remedy can be used to correct shadowing of the satellite signal by a shipboard obstruction? Relocate the mast or other obstruction. Raise the transmit power level. Change the ship's course. Increase the receiver gain.
D A vessel loses Inmarsat-B SES operation after a large course change. Which of the following could most likely cause this? Shadowing of the SES antenna by clouds or other weather formations. Misalignment of the shadow correction filter. The vessel sailed beyond the footprint of the satellite coverage. System loses gyro follow-up or a gyro system malfunction.
A Which statement is correct regarding a method that a vessel experiencing problems with shadowing of an Inmarsat-B (or F77) SES antenna by an on-board obstruction could use to attempt reliable communications? Change course to remove the shadow. Change the Coast Station ID programming. Install a shadow correction filter. Switch from TELEX to voice mode.
B Which of the following conditions does not typically impair Inmarsat-B communications ? An obstruction, such as a mast, blocking the signal between the satellite and the SES antenna when the vessel is steering a specific course. Normal precipitation from gales and storms. A satellite whose signal is on a low elevation, below the horizon. Travel beyond the effective radius of the satellite.
B A vessel is experiencing problems tracking the satellite in an Inmarsat-C SES while at sea. The problem is least likely caused by? Extremely heavy rain/snow storms. Local RF interference by in-port cell phone or other radio systems. Shadowing caused by an obstacle, such as a mast, between the SES antenna and the satellite. The vessel is on the fringe of the coverage area of the satellite.
C Which functions may the holder of a GMDSS Radio Operator License not perform on the Inmarsat-C equipment? Selection of LES for routine traffic. Maintain the antenna clear of soot, paint, etc. Adjust the azimuth and elevation values. Logon, traffic and logoff functions.
B Which modes could be selected to receive vessel traffic lists from high seas coast radio stations: AM and VHF-FM SSB and FEC ARQ and FEC VHF-FM and SSB
C MF/HF Transceiver Power levels should be set __________. To the highest possible level to ensure effective communications. To the lowest possible level at all times regardless of whether communications are effective. To the lowest level needed to achieve the necessary propagation radius and communications range. To the highest level possible so as to ensure other stations cannot "break-in" on the channel during use.
D Which statement regarding GMDSS MF/HF Transceiver frequency set-up is true: Manual keypad entries are quicker and more certain than using the database. All consoles ensure that manual keypad entries are checked for legal outcomes. All consoles ensure that ITU channel recall from a database produces accurate and legal outcomes. Using the manufacturer's database typically produces a more swift and certain result.
A Which statement regarding GMDSS MF/HF Transceiver frequency set-up is true: Some consoles allow both manual keypad entry and ITU channel recall from a database or memory. Transmit and receive frequencies must always be manually entered from the keypad. Transmit and receive frequencies must always be recalled from a database or memory. Frequencies in the manufacturer's databases are always accurate and legal for use.
D To set-up the MF/HF Transceiver for a TELEX call to a coast station, the operator must: Select J3E mode for proper SITOR (NBDP) operations. Select F1B/J2B modes or J3E mode, depending on whether ARQ or FEC is preferred. Select J3E mode for ARQ and H3E mode for FEC. Select F1B mode or J2B mode, depending on the equipment manufacturer.
A To set-up the MF/HF Transceiver for a voice call to a coast station, the operator must: Select J3E mode for proper voice operations. Select J3E mode for proper SITOR (NBDP) operations. Select F1B mode or J2B mode, depending on the equipment manufacturer. Select F1B/J2B modes or J3E mode, depending on whether FEC or ARQ is preferred.
A How are paired SITOR (NBDP) frequencies normally used? These are normally used for ARQ communications with coast radio stations. These are normally used for FEc communications with coast radio stations. These are normally used only for Distress communications to limit channel interference. These are normally used for DSc communications with coast radio stations.
B How are paired SSB frequencies normally used? These are normally used for FEc communications with coast radio stations. These are normally used for J3E communications with coast radio stations. These are normally used for ARQ communications with coast radio stations. These are normally used for DSc communications with coast radio stations.
C For general communications purposes, paired frequencies are: Normally used with private coast stations. Normally used between ship stations. Normally used with public coast stations. Normally used between private coast and ship stations.
D For general communications purposes, simplex frequencies are: Normally used with public coast stations for routine telephone communications. Normally used between ship stations as well as public coast stations simultaneously. Normally used with public coast stations for routine SITOR (NBDP) communications. Normally used between ship stations and private coast stations or for ship-ship communications.
A An ITU simplex channel frequency assignment is defined as: Transmit and receive frequencies must be identical. Transmit and receive frequencies must be different. Transmit and receive frequencies may be different, depending on whether communications are ship-shore or ship to ship. Transmit and receive frequencies are different regardless of emission mode.
B An ITU duplex channel frequency is defined as: Transmit and receive frequencies may be identical if communications are ship-shore. Transmit and receive frequencies must be different. Transmit and receive frequencies must be identical. Transmit and receive frequencies may be different if communications are ship-ship.
D Which of the following defines "ITU Channel 1216"? Ch-12 in the 16 MHz band. Ch-1216 in the MF band. Ch-12 in the 16 kHz band. Ch-16 in the 12 MHz band.
A Which of the following is a valid 22-MHz ITU Channel? HF Ch-2206 VHF Ch-22 Ch-22A VTS Ch-70 (DSC only)
B What is meant by the term duplex "ITU channel"? This refers to a vessel's SELCAL number. A standardized series of frequency pairings for common use. This refers to VHF channels 1-28 and 60-88. A series of frequency pairings used for ship-ship communications.
C ITU duplex channel 1604 would mean: Ch-16 in the 4 MHz band. Ch-1604 in the MF band. Ch-4 in the 16 MHz band. Ch-4 in the 6 MHz band.
C What is a potential danger of using the manufacturer's database to set up for operations using MF-HF ITU frequency assignments? All manufacturer's frequency databases are required to be standardized & accurate so that there is no danger. Using the manufacturer's frequency database to setup the transceiver typically takes longer and is more prone to error than a manual setup. Different manufacturers use different database numbering systems to incorporate the ITU assignments into the transceiver and therefore a potential for confusion exists. Using the manufacturer's frequency databases ensures that the transceiver is always restricted to the vessel's licensed frequencies so that there is no danger.
D ITU Duplex channels are: Frequency assignments specific to U.S. vessels only. VHF-FM frequencies. International Traffic Utility frequencies. Internationally standardized assignments of frequency pairs for common use.
C Which statement regarding bandwidth and channel spacing is correct: Bandwidth values are a function of channel spacing values. Channel spacing values are not a function of bandwidth values. Channel spacing values are a function of bandwidth values. Bandwidth & channel values do not vary with emission mode.
D The purpose of ITU channel spacing is: To minimize the number of voice & TELEX channels available. To make most efficient use of the radio spectrum, by using voice channels rather than TELEX channels. To make most efficient use of the radio spectrum, by using TELEX channels rather than voice channels. To minimize the possibility of interference from adjacent channels.
A Which statement regarding channel spacing and bandwidth is true? Both TELEX bandwidth and channel spacing values are less than voice bandwidth and channel spacing values. Voice bandwidth is greater than TELEX bandwidth and therefore voice channel spacing values are less than TELEX channel spacing values. TELEX bandwidth is greater than voice bandwidth and therefore TELEX channel spacing values are less than voice channel spacing values. Both TELEX bandwidth and channel spacing values are greater than voice bandwidth and channel spacing values.
B The proper sequence of channel spacing from narrow to widest is: SSB voice, SITOR (NBDP), VHF-FM voice. SITOR (NBDP), SSB-voice, VHF-FM voice. VHF-FM voice, SITOR (NBDP), SSB-voice. SITOR (NBDP), VHF-FM voice, SSB-voice.
B Communications with an emission of F1B/J2B would typically have a channel spacing of: 0.3 kHz 0.5 kHz 2.8 kHz 3.0 kHz
C Communications with an emission of J3E would typically have a channel spacing of: 0.5 kHz 0.3 kHz 3.0 kHz 2.8 kHz
B For RF communications, "modulation" is best defined as: Using a single carrier frequency with the proper power level. The combination of information or intelligence on to a radio carrier frequency. Setting up the transceiver with the correct bandwidth to ensure proper communications. The combination of the received frequency and oscillator frequency in the mixer.
C For RF communications, "bandwidth" is best defined as: The modulation technique required to insure proper ITU channel spacing. The emission designation resulting from the desired modulation technique. The portion of the radio spectrum consumed by a particular emission selection and modulation technique. The portion of the radio spectrum reserved for frequency allocations by the ITU.
D In an AM broadcast signal using voice: Varying the amplitude of the carrier and employing both sidebands without the carrier. Varying only the amplitude of the carrier, depending on Double or Single-Sideband operations. Varying the amplitude of the carrier and employing both sidebands and the carrier. There is a carrier with constant amplitude and frequency with complex upper and lower sidebands varying in amplitude and frequency.
A In FM communications, the information is applied to the carrier by: Varying the frequency of the carrier to convey the information to other stations. Varying the amplitude or the frequency of the carrier, depending on Double or Single-Sideband operations. Varying only the frequency of the carrier, depending on Double or Single-Sideband operations. Varying the amplitude of the carrier and keeping the frequencies in the sidebands constant.
D The proper sequence of emissions corresponding to the sequence AM-Voice DSB, SSB-Voice without carrier, USB-Voice with carrier and FM-Voice, is: A3E, H3E, J3E, F3E. J3E, H3E, A3E, F3E. H3E, A3E, J3E, F3E. A3E, J3E, H3E, F3E.
A The proper sequence of emissions corresponding to the sequence SSB-Voice without carrier, USB-Voice with carrier, FM-Voice and SITOR (NBDP) TELEX is: J3E, H3E, F3E, F1B. H3E, J3E, F3E, F1B. J3E, H3E, F1B, F3E. H3E, J3E, F1B, F3E.
A When placing a SSB MF/HF call to a Coast Station, you should always: Make sure the frequency is not occupied. Choose the closest station to ensure a quick connection. Tune the transmitter on another frequency. Wait until the coast station sends his traffic list.
B How are high seas (HF) radiotelephone communications initially established between a vessel and a public correspondence station? The vessel listens for "free signals" and calls the public correspondence station on the SITOR (NBDP) calling channel with the strongest marker signal. The vessel calls and establishes voice contact with the public correspondence station on a channel that the station is known to monitor, and the two stations then proceed with their business. The vessel calls the public correspondence station on VHF Channel 16, and the two stations then switch to the working channel. Public Correspondence Stations operate SITOR (NBDP) only.
C What is the best procedure for calling another ship station using HF radiotelephone when the signals are weak but readable? Give the name of the ship being called three times, and the words "this is" followed by the name of the ship initiating the call three times, and ending with "over." Instruct the nearest public correspondence station to add the desired ship's call sign to the station's traffic list. On a properly selected ITU channel, give the call sign of the ship being called three times using phonetics, then "this is" followed by the call sign of the ship initiating the call three times, using phonetics, and ending with "over." Notify the local vessel traffic service control station of your intention to contact a specific vessel, and request the VTS operator place the call on channel 22A.
D What is the correct procedure for calling a coast radio station using HF radiotelephone? On a properly selected ITU channel, give the name of the coast radio station being called three times, and the words "this is" followed by the name of the ship initiating the call three times, and ending with "over". Contact the nearest U.S.C.G. station to add the desired ship's call sign to the station's traffic list. Request the VTS operator place the call on channel 22A. On a correct ITU channel, give the call sign of the coast radio station three times using phonetics, the words "this is", followed by the ship's call sign three times using phonetics and ending with "over".
A Through which coast radio station(s) may a U.S.-flag merchant vessel communicate? Any coast radio station in the world that is licensed to provide such communications. Any coast radio station in the world that is licensed to provide such communications, but prior authorization must be obtained for a U.S.-flag merchant vessel to communicate through a non-U.S. station. The U.S. Coast Guard coordinates the communications and assigns the working channel. U.S. flag ships are licensed to communicate only with U.S. coast radio stations.
B What is the best source of information to find changes or additions to the routine communications frequencies of a Commercial Radio Station? GMDSS Master Plan of Shore-Based Facilities. ITU List of Coast Stations and Special Service Stations. (List IV) FCC Part 80, Subpart W Coast Radio Stations. ITU List of Ship Stations and Maritime Mobile Service Identities. (List V)
D Which of these is not a primary purpose of the MF/HF DSC controller? It provides for the electronic memory of incoming and outgoing DSC calls. It permits control of transceiver operations in response to an incoming DSC call. It provides for the formatting and transmission of outgoing DSC calls. It provides the scanning watch receiver capability on the 6 MF/HF DSC frequencies.
A A "Distress Hot Key" MF/HF DSC Distress alert: Will be transmitted on 2187.5 kHz or another DSC frequency, depending on the manufacturer. Will always be transmitted on 2187.5 kHz and 8414.5 kHz to trip DSC alarms on the mandatory MF/HF DSC watch frequencies. Will always be transmitted on 2187.5 kHz first to alert the nearest vessels and coast stations. Will always be sent first on 16804.5 kHz to reach coast stations far away from the vessel.
B A Distress Priority DSC call may be formatted and transmitted specifying and requesting: Nature of Distress, vessel position, follow-on frequency, only voice follow-on communications. Nature of Distress or alternate frequency but not both in a single call, vessel position or alternate frequency/emission but not both in a single call, voice or TELEX follow-up communications. Nature of Distress or alternate frequency but not both in a single call, vessel position or alternate frequency/emission but not both in a single call, only TELEX follow-up communications Nature of Distress, vessel position, follow-on frequency, only TELEX follow-on communications.
C Which of these statements regarding a multi-frequency MF/HF DSC Distress alert is false? Some units transmit on the mandatory MF/HF DSC watch frequencies first, and then on the others. Some units transmit in ascending order of propagation radius to alert nearby ships or shore stations first. May be transmitted in any order programmed by the GMDSS operator. Multi-frequency alerts work quite differently, depending on the manufacturer and require care and understanding of the feature.
C To make a call to another vessel requesting voice communications regarding important company business, the GMDSS operator should: Select Routine priority, enter other vessel's MMSI, specify legal alternate frequency, J2B emission and transmit the properly formatted DSC call. Select Routine priority, enter own vessel's MMSI, specify legal alternate frequency, J3E emission and transmit the properly formatted DSC call. None of these answers is correct. Select Urgency priority, enter other vessel's MMSI, specify legal alternate frequency, F1B emission and transmit the properly formatted DSC call.
D To make a call to another vessel requesting TELEX communications regarding important company business, the GMDSS operator should: Select Urgency priority, enter other vessel's MMSI, specify legal alternate frequency, F1B emission and transmit the properly formatted DSC call. Select Routine priority, enter own vessel's MMSI, specify legal alternate frequency, F1B emission and transmit the properly formatted DSC call. Select Routine priority, enter other vessel's SELCAL for TELEX specify legal alternate frequency, F1B emission and transmit the properly formatted DSC call. Select Routine priority, enter other vessel's MMSI, specify legal alternate frequency, F1B emission and transmit the properly formatted DSC call.
C What is meant by describing a Coast Station with the acronym ATOR? The station's BFEC operations are computerized and a rigid operating sequence must be followed correctly. The station will control all of the ARQ operations and it will generate the proper service request codes at the correct time in the sequence. The station's ARQ operations are computerized and a rigid operating sequence must be followed correctly. The station will control all of the ARQ operations and it will provide prompts for the operator to request time & charges at the correct time in the sequence.
D Which of the following acronyms refers to a communications system that does not use ARQ and/or FEC modes? NAVTEX SITOR (NBDP) HF MSI SART
A What does the term FEC mean in SITOR (NBDP) communications? Forward Error Correction Field Effect Correction Forward Error Check Forward Error Character
B A Coast Station that is described as ATOR typically provides what type of services? NAVTEX BFEC weather broadcasts. ARQ SITOR (NBDP) connections to shoreside terminals. BFEC SITOR (NBDP) connections to shoreside terminals. SITOR (NBDP) HF weather broadcasts.
B The acronym SITOR (NBDP) stands for: Simplified Telephony Over Radio Simplex TELEX Over Radio Simplified TELEX Over Radio Simplex Telephony Over Radio
C What does the term "ARQ" mean in SITOR (NBDP) operations? Automation Repeat Request Automaton Repeat Request Automatic Repeat Request Automatic Request Repeat
B Which of the following statements concerning SITOR (NBDP) communications is true? ARQ transmissions are acknowledged by the Information Receiving Station only at the end of the message. ARQ transmissions are made in data groups consisting of three-character blocks. ARQ communications rely upon error correction by time diversity transmission and reception. Forward error correction is an interactive mode.
C Which of the following statements concerning SITOR (NBDP) communications is true? FEC transmissions are made in data groups consisting of three-character blocks. FEC transmissions require a "phasing" or "handshaking" process prior to character transmission. FEC transmissions rely upon an error correction technique, which transmits each character twice. FEC transmissions rely upon parity and "repeat requests" to ensure error correction.
D Which statement best defines the SITOR (NBDP) acronym "ARQ"? Error Correction when 2 stations are in direct & phased telephone communications with each other. Error correction for one-way TELEX transmissions to a single other station. Error correction for one-way TELEX broadcasts of weather or navigation information. Error Correction with 2 stations are in direct & phased TELEX communications with each other.
A Which statement best defines the SITOR (NBDP) acronym "BFEC"? Error correction for one-way TELEX broadcasts of weather or navigation information. Error Correction when 2 stations are in direct & phased telephone communications with each other. Error Correction when 2 stations are in direct & phased TELEX communications with each other. Error correction when 1 station sends a one-way TELEX to a single other station.
D The purpose of "parity" in SITOR (NBDP) communications is? Error correction in HF BFEC MSI broadcasts, SFEC NAVTEX broadcasts and two way ARQ exchanges. Error correction in SFEC NAVTEX broadcasts, HF SFEC MSI broadcasts and two-way ARQ exchanges. Error correction in BFEC NAVTEX broadcasts, HF BFEC MSI broadcasts and one-way ARQ broadcasts. Error correction in HF BFEC MSI broadcasts, NAVTEX broadcasts and two-way ARQ exchanges.
A "SITOR (NBDP)" communications are based on a digital code system: Consisting of 7 bits, with four zeros and three ones. Consisting of 7 bits, with four ones and three zeroes. Consisting of 5 bits TELEX and 8 bits Fax. Consisting of 5 bits for ship stations and 7 bits for shore stations.
A Which of the following would be a valid SELCAL for use in ARQ communications? 1106 212420 WHAQ X. Four marks (ones) and three spaces (zeroes) forming the binary signal "1001101". This is established by the communications protocol used with the modem.
B Once ARQ communication with the coast radio station has been established, which of the following events will most likely take place? The vessel then requests the coast radio station's SELCAL so that communication can be set up on the appropriate working channel. After exchanging answer-backs (WRU-AAB) with the vessel, the coast radio station transmits GA+?. Since communication has already shifted to the working channel, the vessel then transmits the subscriber number and text of the message to be sent for the coast radio station to store and forward. The coast radio station will transmit a menu of commands such as DIRTLX+, OBS+ or AMV+ so the ship can select the desired action.
C Which of the following methods will give a GMDSS Radio Operator the best indication of whether ARQ communication can be established with a coast radio station? Referring to propagation charts will tell the Operator when the eruption of communication shattering solar flares will occur. Selecting a frequency in the MF band averts interference from severe static discharges. Monitor the coast radio station's "free signals" and call on the frequency on which the loudest and most consistent signals are heard. Re-position the antenna toward the coast radio station and press the "call request" button.
D When placing a TELEX call to a Coast Station, you should always: Choose the closest station. Tune the transmitter on another frequency. Wait until the coast station sends his traffic list. Make sure the frequency is not occupied with normal traffic.
A What is the best method for a GMDSS Radio Operator to determine which SITOR (NBDP) station to contact for the purpose of sending a chargeable message or cable? Listen to each station's "free signals" and call the strongest station. Listen to each station's voice announcement and determine which channel(s) will be monitored. Listen to each station's MSI broadcast to determine which public correspondence station to contact. Listen to the U.S. Coast Guard's traffic list to determine which Coast Guard station will handle commercial traffic.
B Which statement is correct regarding HF SITOR (NBDP) under GMDSS? Distress communications other than directly to the Coast Guard or other coast stations on the channels that they normally guard, should be in the broadcast SSB mode. Safety communications by direct-printing telegraphy should be in the ARQ mode when communicating with the U.S. Coast Guard or other coast stations on channels that they normally guard. The ARQ mode may not be used subsequently to the FEC mode even when it is advantageous to do so. Distress communications other than directly to the Coast Guard or other coast stations on the channels that they normally guard, should be in the broadcast FEC or SSB mode.
D Which of the following events will take place immediately after phasing with a Coast Station on an available channel? The ship station begins transmitting the text to the destination telex terminal. The vessel will request the shore station's answerback to confirm it has reached the correct station. The ship will transmit the desired command (such as DIRTLX+, OBS+ or AMV+) to the coast radio station. After exchanging answer-backs (WRU/AAB) the coast station transmits GA+? and awaits a command.
A During SITOR (NBDP) ARQ operations through a coast station, what should the GMDSS operator do during the "Automatic Exchange of Answerbacks"? Do nothing other than wait for a GA+? prompt and then issue the desired command. Send a "WRU" then a "Here is" when requested to do so by the coast station. Send the necessary message file and then wait for time and charges. Wait for the coast station to transmit a menu of possible commands and select the desired action.
B Assuming sending a direct TELEX message to a shore-based office, which sequence of events best describes a complete ARQ TELEX exchange with a coast station? Initiate call, observe phasing, wait for exchange of answer-backs (WRU-AAB), send message, send KKKK to terminate the TELEX link, receive MRN & time and charges. Initiate call, observe phasing, wait for exchange of answer-backs (WRU-AAB), send DIRTLX command with zero, TELEX country code, TELEX number, send message, send KKKK to terminate the TELEX link, receive MRN and time and charges. Initiate call, observe exchange of answer-backs (WRU-AAB), send message, send KKKK to terminate radio link, receive MRN & time and charges. Initiate call, observe phasing, exchange of answer-backs (WRU-AAB), send message, send KKKK to terminate radio link, receive MRN & time and charges.
C For ARQ communications with a Public Correspondence Coast Station, which sequence of events best describes reaching the point in time where the text of TELEX communications should be sent __________. Transceiver setup, SELCAL selection, Initiate Call, wait for exchange of answer-backs (WRU-AAB), OPR+, operator entry of the appropriate automatic TELEX code. Transceiver setup, wait for exchange of answer-backs (WRU-AAB), OPR+, operator entry of the appropriate automatic TELEX code. Transceiver setup, SELCAL selection, Initiate Call, wait for exchange of answer-backs (WRU-AAB), GA+?, enter DIRTLX xy+, MOM, MSG+? and exchange of terminal answerbacks. Transceiver setup, wait for exchange of answer-backs (WRU-AAB), GA+? and then send message text.
C During ARQ communications, which of these is least likely to cause a coast station to immediately break the phased radio connection: If the automatic exchange of answerbacks is interrupted by keyboard entries. If the error percentage of repeat requests becomes too high. If the operator enters "KKKK" at the end of the message. If the BRK+? Command is transmitted after time and charges are received.
D During ARQ communications, which of these is least likely to cause a coast station to break the phased radio connection: If the error percentage of repeat requests becomes too high. If the operator enters "BRK+". If the automatic exchange of answerbacks is interrupted by keyboard entries. If the operator enters "NNNN" at the end of the message.
C What sequence of ARQ codes matches the following: manual over, end of message text, end of connection to land destination terminal? NNNN, KKKK, GA+ KKKK, DIRTLX, NNNN +?, NNNN, KKKK +?, . . . . ., BRK+
D What sequence of ARQ codes matches the following: request establishment of a landline, end radio link after traffic is done, request connection for AMVER messages? NNNN, KKKK, AMV+ KKKK, BRK+, AMV+ BRK+, DIRTLX+, AMV+ DIRTLX, BRK+, AMV+
A What sequence of ARQ codes matches the following: Send synoptic weather, switch control from a station currently transmitting over to the receiving station, terminate phasing with coast station. OBS+, +?, BRK+ OBS+, WRU then Here is, BRK+ OBS+, GA+?, KKKK AMV+, +?, KKKK
B A manual ship-ship ARQ message would typically begin with what sequence of ARQ procedures? Initiate call with CS selcal, observe exchange of answer-backs (WRU-AAB), send message. Initiate call with ship selcal, enter "HERE IS" then "WRU" commands and send message. After phasing with the other vessel, observe exchange of answer-backs (WRU-AAB), send message and then KKKK to end the transmission. After phasing with the other vessel then send GA+? to determine if they are ready to receive your message.
B What sequence of ARQ codes matches the following: shore station indicates it is awaiting instructions from the vessel, vessel requests a live connection to a land telex number, vessel indicates it is done with the coast station? WRU-AAB, DIRTLX, NNNN GA+?, DIRTLX, BRK+ GA+?, DIRTLX, KKKK WRU-AAB, DIRTLX, .....
C All country codes which are transmitted by MF/HF SITOR (NBDP) are preceded by which character(s)? OO + O DIR
B Which of the following statements concerning SITOR (NBDP) communications is true? In ARQ, each character is transmitted twice, about 250 milliseconds apart. In ARQ, the "information sending station" will transmit a block of three characters that the receiving station will subsequently acknowledge or request be retransmitted. In ARQ, the "information sending station" transmits a block of three characters twice, about 250 milliseconds apart. SITOR (NBDP) communications can be used to contact a NAVTEX transmitting station when requesting a repeat transmission of a missed NAVTEX message.
C What statement is true regarding the exchange between two parties engaged in SITOR (NBDP) communications? In ARQ, each character is transmitted twice, with the second displaced in time from the first. In ARQ, the "sending" station transmits a block of three characters and the "receiving" station responds with a one character Repeat Request. Following this the "transmitting" station will send a new block. In ARQ, the ISS transmits a block of 3 characters and the IRS checks for parity. If the received block is correct a one-character control signal is sent notifying the ISS to proceed with the next block. If the parity check fails the block must be resent. Broadcasts of Maritime Safety Information, traffic lists, etc. can be copied by the receiving station in ARQ mode.
D Of the following, which is true of SITOR (NBDP) ARQ mode? The acceptance code consists of three characters. A continuous data stream is transmitted. Forward error correction reduces the number of errors. Each data block consists of three characters.
A Which of the following is true of SITOR (NBDP) ARQ mode? This is an interactive mode. Each character is repeated three times. Each character is transmitted twice. This mode is generally used to broadcast messages.
D In ARQ, when the information sending station (ISS) receives a signal that the parity check failed what happens? The next block will be sent. The data link will break. The acknowledge light should illuminate. The last block will be resent.
A Which of the following is true of SITOR (NBDP) ARQ mode: The ship station sends a group of 3 characters, the shore station checks for proper parity. If parity is OK, the shore station indicates readiness for transmission of the next 3 characters. The ship station sends a group of 3 characters twice and then waits for an "RQ" signal to indicate proper receipt before continuing transmission. The Ship station sends each character twice, using a time diversity system to ensure proper parity. The ship station sends a group of 3 characters, the shore station checks for proper parity and then requests the same group be resent to enable error correction.
A Which of the following keystrokes or characters is sent as part of ARQ communications to request a direct TELEX call to a shore-based location? DIRTLX023123456+ MSG+ GA+? ENTER
B The DIRTLX command should be given to a coast station at what point in time? Immediately following the automatic exchange of answerbacks. After receipt of the Go Ahead indication following the automatic exchange of answerbacks. After receipt of the Go Ahead indication followed by the automatic exchange of answerbacks. Sent at the beginning of the message to request a direct TELEX connection.
C Which automatic TELEX command should be sent by a ship station during SITOR (NBDP) operations to properly address a weather report to the national weather authority? WX+ followed by the text of the report. OBS+ then MSG+ to indicate weather report. OBS+ and then send the weather report. GA+? then OBS+ and then send the weather report.
D Which of the following would be a valid automatic TELEX code and number for a request for a real-time TELEX connection to a shore-based TELEX terminal? DIRTLX then waits for MSG+ to dial 023419645+. DIRTLX23122445+. DIRTLX followed by GA+? and the TELEX number. DIRTLX023424998+.
A When requesting a direct TELEX connection to a vessel's Inmarsat-B terminal, (i.e. making a SITOR (NBDP) call to an Inmarsat-B unit) the GMDSS operator must: DIRTLX, 0, Ocean Area Code, TELEX number, +. DIRTLX, 00, Ocean Area Code, TELEX number, +. DIRTLX, 0, Country code, TELEX number, +. DIRTLX, 00, Country code, TELEX number.
B To correctly address a SITOR (NBDP) message and transmit it "live" to a shore-based office the GMDSS operator would dial which sequence: DIRTLX0051440344+ DIRTLX051440344+ DIRTLX581440344+ DIRTLX05811500260+
D Which of the following statements concerning SITOR (NBDP) communications is true? Communication is established on the working channel and answerbacks are exchanged before FEC broadcasts can be received. All of these answers are true. Weather broadcasts cannot be made in FEC because sending each character twice would cause the broadcast to be prohibitively long. Two-way communication with the coast radio station using FEC is not necessary to be able to receive the broadcasts.
A Which of the following is true of SITOR (NBDP) Mode B (FEC), in the presence of static crashes. FEC reduces the error rate by transmitting each character twice. Data flow rate depends on signal propagation. Idle characters are sent upon request. Transmitter and receiver cannot synchronize.
B Which of the following statements concerning SITOR (NBDP) communications is true? FEC requests are first acknowledged by the vessel's transmitter before broadcasts can be received. FEC mode broadcasts can be passively received without the transmitter being active. Weather broadcasts are always made in ARQ mode to ensure reception. ARQ mode communications can be passively received without the transmitter being active.
C The sequence ARQ, FEC, SFEC best corresponds to which of the following sequences? One-way communications to a single station, one-way communications to all stations, two-way communications. One-way communications to all stations, two-way communications, one-way communications to a single station. Two way communications, one-way communications to all stations, one-way communications to a single station. Two way communications, One way communications to a single station, One way communications to all stations.
C The sequence BFEC, SFEC, ARQ best corresponds to which of the following sequences? One-way communications to a single station, one-way communications to all stations, two-way communications. One-way communications to all stations, two way communications, one-way communications to a single station. One-way communications to all stations, one-way communications to a single station, two way communications. Two way communications, one-way communications to all stations, two-way communications.
D Selective FEC communications (SFEC) are employed when: Multiple stations without a group SELCAL must receive communications without using their transmitters (Radio Silence). Multiple stations must receive communications by using their transmitters to achieve phasing. An individual station must receive communications by using their transmitter to achieve phasing and block other stations from breaking in. An individual station must receive communications without using any transmitters (Radio Silence).
C If the vessel is within range of NAVTEX broadcasts and both the Inmarsat-C and the NAVTEX receiver are inoperative the GMDSS operator should: Select 518 kHz ARQ TELEX on the MF/HF console to receive MSI. Request repairs of the Sat-C system and wait until within range of NAVTEX. Select 518 kHz FEC TELEX on the MF/HF console to receive MSI. Select an HF MSI frequency and ARQ TELEX mode to receive MSI.
D If the vessel is beyond range of NAVTEX broadcasts and the Inmarsat-C system fails, the GMDSS operator must: Select 518 kHz ARQ TELEX on the MF/HF console to receive MSI. Request repairs of the Sat-C system and wait until within range of NAVTEX. Select 518 kHz FEC TELEX on the MF/HF console to receive MSI. Select an HF MSI frequency and FEC TELEX mode to receive MSI.
A If the vessel is experiencing atmospheric interference with NAVTEX broadcasts, especially in the tropics, the GMDSS operator should: Select one of the 8 HF MSI frequencies and set-up the transceiver in FEC TELEX mode. Select one of the 6 HF MSI frequencies and set-up the transceiver in ARQ TELEX mode. Select one of the 6 MF MSI frequencies and set-up the transceiver in FEC TELEX mode. Select the MF MSI frequency dedicated to tropical MSI and set-up the transceiver in FEC TELEX mode.
B Why must the GMDSS operator be able to set-up FEC & ARQ modes and differentiate between them? The proper mode of SFEC must be selected for reception of HF MSI and ARQ selected for initial follow-on TELEX communications in a Distress situation The proper mode must be selected for follow-on TELEX communications in a Distress situation and for the reception of HF MSI. The ARQ TELEX mode must be selected for follow-on TELEX communications in a Distress situation and the BFEC TELEX mode selected for reception of HF MSI. The BFEC TELEX mode must be selected for follow-on TELEX communications in a Distress situation and the ARQ TELEX mode selected for reception of HF MSI.
B BFEC SITOR (NBDP) transmissions might be used to? Receive Coast station traffic lists, NAVTEX and VHF MSI broadcasts. Send and receive Distress TELEX communications and receive HF MSI or NAVTEX. Receive weather messages or download ARQ Coast Station traffic lists. Receive weather messages and send vessel weather OBS messages.
C FEC SITOR (NBDP) transmissions are normally used to? Receive Coast station traffic lists, NAVTEX and VHF MSI broadcasts. Receive weather messages, Coast Station traffic lists and company messages. Receive HF MSI and NAVTEX. Receive company communications sent through coast stations.
B The "short rules" of propagation necessary to select the appropriate frequency band are: Shorter distance = Higher Frequency, Daytime = Higher Frequency. Shorter distance = Lower Frequency, Daytime = Higher Frequency. Shorter distance = Higher Frequency, Daytime = Lower Frequency. Shorter distance = Lower Frequency, Daytime = Lower Frequency.
C GMDSS operators should learn which of the following propagation "rules of thumb"? Longer distance = lower frequency, Shorter distance = higher frequency; Daytime = higher frequency, nighttime = lower frequency. Longer distance = lower frequency, Shorter distance = lower frequency, Daytime = lower frequency, nighttime = higher frequency. Longer distance = higher frequency, Shorter distance = lower frequency, Daytime = higher frequency, nighttime = lower frequency. Longer distance = higher frequency, Shorter distance = higher frequency, Daytime = lower frequency, nighttime = higher frequency.
D What would be the most appropriate HF bands for communicating from San Francisco to Taiwan or the Philippines? 12 MHz during daylight at each end and 2 MHz when dark at each end. 8 MHz during daylight hours and 16 MHz during darkness. 6 MHz during daylight hours and 8 MHz during darkness. 16 or 22 MHz when daylight at each end and 8 MHz when dark at each end.
A GMDSS operators should routinely focus on the factors affecting propagation in what priority? Distance & time of day, seasonal variations, sunspot cycle, solar flare alerts. Sunspot cycle, distance & time of day, seasonal variations, solar flare alerts. Solar flare alerts, distance & time of day, sunspot cycle, seasonal variations. Solar flare alerts, sunspot cycle, seasonal variations, distance & time of day.
D Which statement concerning frequency band selection and propagation most corresponds with standard practice. Maximum Usable Frequency (MUF) and Optimum Usable Frequency (OUF) should be calculated prior to setting up the transceiver. Propagation rules, Sunspot cycle status and seasonal variations (winter & summer) must always be employed to calculate the proper band selection. Propagation "rules of thumb" should be combined with Solar flare alerts to determine band selection. Propagation "rules of thumb" are usually sufficient for routine operations.
A To ensure effective HF communications, GMDSS operators should: Employ the short rules of propagation selection or rely on previous successful communications as a guide. Disregard the short rules of propagation selection and try different bands until one works. Do not rely on previous successful communications on the selected frequency band - the ionosphere changes too rapidly and inconsistently. Rely on the equipment calculations of Optimum Usable Frequency.
A A ship has been communicating effectively on 16 MHz during daylight hours with a shore station at a distance of 3500 miles. Toward late afternoon and evening what effect would be noticed? Communications should gradually deteriorate and become impossible on this frequency at night. Communications should be maintained with slight improvement in the signal received from the shore station. The gray line effect will prevent communications after dark. Communications should improve and peak at night.
B A ship at anchor has been communicating with a shore station approximately 200 miles distant on a frequency in the 4 MHz band through mid-morning. Toward the late afternoon and evening, what effect should be noticed? Communications should be maintained with slight improvement in the signal received from the shore station. Communications should gradually improve and peak at night and early morning. Communications should slowly deteriorate but may be continued throughout the night. Communications should gradually deteriorate and become impossible on this frequency by night.
C At mid-day in the summer, what would be the best choice in attempting to communicate, using SITOR (NBDP) with a shore station some 1800 miles distant? VHF-FM Lower HF bands Higher HF bands MF
D At mid-night, what would be the best choice in attempting to communicate, using SITOR (NBDP) with a shore station some 800 miles distant? VHF-FM Higher UHF bands Communications are impossible at this distance. HF bands
A At mid-day, what would be the best choice in attempting to communicate with a shore station some 75 miles distant? MF VHF-FM 22 MHz band 16 MHz band
B How can a GMDSS operator determine the best frequency band to choose for a SITOR (NBDP) transmission to a shore station? By consulting propagation tables. Listen to shore station "free signals" and choose the band with the strongest signal. If static interference is present, try lower bands first. During nighttime, choose higher frequencies. Choose lower frequencies in the daytime.
D The Distress Alarm sounds and the screen readout no longer contains the particulars of the Distress: The particulars are rarely routed to the printer so it is of no use to examine the printer. Examine the Transmitted Data Directory or the printer to recover the information. Call the Distress vessel on the DSC frequency and request a retransmission. Examine the Received Data Directory or the printer to recover the information.
A A DSC Distress call is received by your vessel and your transceiver frequency display reads: Transmit = 4207.5 kHz and Receive = 4207.5 kHz with J3E emission -- what information can you infer from this? The DSC call requested voice on 4207.5 kHz simplex but the requested alternate frequency is improper. The DSC controller decoded the requested voice frequency as 4207.5 kHz simplex and your DSC controller has automatically set-up your transceiver and therefore the requested alternate frequency is correct. The DSC call came in on 4 MHz DSC. You should set-up your transmitter and respond on the appropriate voice follow-on frequency of 4177.5 kHz. The DSC call came in on 4 MHz DSC. You should set-up your transmitter and respond on the appropriate voice follow-on frequency of 4207.5 kHz.
B What are potential dangers of automatic transceiver switching in response to a DSC call? Incorrectly formatted DSC calls from other vessels result in a warning in the DSC alarm screen but the vessel still might be switched to an illegal or improper outcome. The transceiver may be switched out of the marine bands or the frequency may not match the requested priority or emission. The automatic transceiver switching software always detects improper priorities, emissions & frequencies to ensure there is no danger. The automatic transceiver switching feature always interrupting safe navigation of your own vessel and placing the unit on the GMDSS Distress, Urgency or Safety follow-on frequencies.
C Which of these would not be a bad outcome of an Urgency DSC call to all vessels specifying an alternate TELEX frequency & emission that cause your transceiver to automatically switch? The DSC controller decoded the contents of the DSC call and the requested frequency is not on the DUS table/placard but is in the marine band frequencies in the J2B emission. The DSC controller decoded the contents of the DSC call and the requested frequency is on the DUS table/placard in the J3E emission. The DSC controller decoded the contents of the DSC call and the requested frequency is on the DUS table/placard in the J2B emission. The DSC controller decoded the contents of the DSC call and the requested frequency is not on the DUS table/placard, is not in the marine band or the other vessel requested a J3E emission.
C An incoming Distress priority 16 MHz DSC call requesting J3E emission might automatically switch to what frequency? 16420.0 MHz 16696.5 kHz 16420.0 kHz 16804.5 kHz
D An incoming DSC Distress alert on 8414.5 kHz will have what result? The DSC controller will emit both an audible and visual alarm. The particulars of the alert may be printed out. The Distress information contained in the alert will be sent to the data directory. All of these answers would typically occur.
C What is the normal voltage of a single lead acid battery cell? 1.5 volts 2.5 volts 2.1 volts 1.2 volts
D What is the normal specific gravity of a fully charged lead acid battery cell? What device is used to measuring the electrolyte of a lead acid battery? 1.375 Voltmeter 1.180 Voltmeter 1.210 Hydrometer 1.280 Hydrometer
A What is a gel cell battery and how is it maintained & cared for? Gel cell batteries are typically sealed; special charging rates and voltages may be required. Gel cell batteries are lead acid with a solid electrolyte; each cell is checked with a voltmeter. Gel cell batteries are NICAD with a solid electrolyte; each cell is checked with a voltmeter. Gel cell batteries are have special liquid electrolytes; they are charged with the NICAD charging rates and voltages.
B What will cause an individual battery cell to reverse polarity. High discharge rates without allowing for a cool down period. When discharging the battery string if a cell becomes weaker than the remaining cells, the discharge current will effectively charge the weaker cell in reverse polarity. Insufficient charging which does not bring all of the cells up to full charge. The charging circuits are connected in the correct polarity but all of the cells are equally charged.
B What is the effect of temperature on the specific gravity of lead acid batteries? Higher temperature results in a higher specific gravity reading. Higher temperature results in a lower specific gravity reading. Temperature has no effect on the specific gravity reading. Lower temperature results in a lower specific gravity reading.
C What would be an indication of a malfunction on a GMDSS station with a 24 VDC battery system? All of these symptoms would indicate a potential battery charger malfunction. After testing the station on battery power, a voltmeter reading of 30 volts for a brief period followed by a steady 26 volt reading. A constant 30 volt reading on the GMDSS console voltmeter. After testing the station on battery power, the ammeter reading indicates a high rate of charge that then declines.
B Which of the following conditions would be a symptom of malfunction in a 2182 kHz radiotelephone system which must be reported to the Master? Much lower noise level observed during daytime operation. No indication of power output when speaking into the microphone. When testing a radiotelephone alarm on 2182 kHz into an artificial antenna, the Distress frequency watch receiver becomes unmuted, an improper testing procedure. Failure to contact a shore station 600 nautical miles distant during daytime operation.
C Which would indicate a malfunction in a 2182 kHz radiotelephone system? No discernable traffic has been heard on the 2182 kHz during the radiotelephone silent periods. Dramatic increase in noise level observed during night and early morning hours. Failure to contact another station 60 miles distant during daytime operation. The visual indication of power to the antenna fluctuates while testing the radiotelephone alarm signal generator into an artificial antenna.
D Which would indicate proper operation of a SSB transmitter rated at 60 Watt PEP output in J3E mode? In SITOR (NBDP) communications, the power meter can be seen fluctuating regularly from zero to the 60 watt relative output reading. In SSB (J3E) mode, speaking into the microphone causes the power meter to fluctuate well above the 60 watt reading. A steady indication of transmitted energy on an RF power meter with no fluctuations when speaking into the microphone. In SSB (J3E) voice mode, with the transmitter keyed but without speaking into the microphone, no power output is indicated.
A Which would be an indication of proper operation of a SSB transmitter rated at 60 watt PEP output? In SITOR (NBDP) communications, the power meter can be seen fluctuating regularly from zero to the 60 watt relative output reading. In SSB (J3E) voice mode, with the transmitter keyed but without speaking into the microphone, power output is indicated. In SSB (J3E) mode, speaking into the microphone causes power meter to fluctuate slightly around the 60 watt reading. A steady indication of transmitted energy on an RF Power meter with no fluctuations when speaking into the microphone.
D Your antenna tuner becomes totally inoperative. What would you do to obtain operation on 2 HF bands? Without an operating antenna tuner, transmission is impossible. It is impossible to obtain operation on 2 different HF bands, without an operating antenna tuner. Bypass the antenna tuner and shorten the whip to 15 ft. Bypass the antenna tuner. Use a straight whip or wire antenna approximately 30 ft long.
A Your MF-HF whip antenna breaks off and is carried away in a storm. What would you do to regain operation on MF-HF frequencies? Rig a wire antenna approximately 35-40 ft long per the equipment instruction manual. Rig a wire antenna 10-15 ft long from the antenna tuner to the highest vertical support. Rig a horizontal, center-fed dipole antenna 40 ft long. Rig a long wire antenna at least 200 ft long.
D GMDSS is primarily a system based on? Ship-to-ship distress communications using MF or HF radiotelephony. VHF digital selective calling from ship to shore. Distress, urgency and safety communications carried out by the use of narrow-band direct printing telegraphy. The linking of search and rescue authorities ashore with shipping in the immediate vicinity of a ship in distress or in need of assistance.
C GMDSS is required for which of the following? All vessels capable of international voyages. Vessels operating outside of the range of VHF coastal radio stations. SOLAS Convention ships of 300 gross tonnage or more. Coastal vessels of less than 300 gross tons.
C The primary purpose of the GMDSS is? Allow more effective control of SAR situations by vessels. Provide additional shipboard systems for more effective company communications. Automate and improve emergency communications for the world's shipping industry. Effective and inexpensive communications.
D What is the basic concept of GMDSS? Search and rescue authorities ashore can be alerted to a distress situation. Shipping in the immediate vicinity of a ship in distress will be rapidly alerted. Shoreside authorities and vessels can assist in a coordinated SAR operation with minimum delay. All of these.
A What is the fundamental concept of the GMDSS? It is intended to automate and improve emergency communications in the maritime industry. It is intended to automate and improve existing digital selective calling procedures and techniques. It is intended to provide more effective but lower cost commercial communications. It is intended to provide compulsory vessels with a collision avoidance system when they are operating in waters that are also occupied by non-compulsory vessels.
C What is the responsibility of vessels under GMDSS? Vessels over 300 gross tons may be required to render assistance if such assistance does not adversely affect their port schedule. Only that vessel, regardless of size, closest to a vessel in distress, is required to render assistance. Every ship is able to perform those communications functions that are essential for the safety of the ship itself and of other ships. Vessels operating under GMDSS, outside of areas effectively serviced by shoreside authorities, operating in sea areas A2, and A4 may be required to render assistance in distress situations.
C What equipment is programmed to initiate transmission of distress alerts and calls to individual stations? Navtex. GPS. DSC controller. Scanning Watch Receiver.
B What equipment is associated with the land or terrestrial systems? EPIRB. VHF-MF-HF. Inmarsat-C. GPS.
B What equipment is associated with the space systems? VHF-MF-HF. Inmarsat-C. Navtex. SART.
D What equipment is used in or near the survival craft? Navtex. Fathometer. Cospas-Sarsat. EPIRB.
C What is the primary equipment for receiving MSI. SART. EPIRB. Navtex. Inmarsat-A.
A What system provides accurate vessel position information to the GMDSS equipment? GPS. Cospas-Sarsat. EPIRB. Inmarsat-A.
C SITOR (NBDP) equipment is a partial or alternate carriage requirement under GMDSS for vessels operating in which sea area(s)? A1. A1 and A2. A3 and A4. A1, A2, A3 and A4.
C What is defined as an area, excluding sea areas A1 and A2, within the coverage of an Inmarsat geostationary satellite in which continuous alerting is available? Ocean Area Regions AOR-E, AOR-W, POR or IOR. Sea Area A4. Sea Area A3. Coastal and Inland Waters.
A What is defined as the area within the radiotelephone coverage area of at least one VHF coast station in which continuous DSC alerting is available as defined by the IMO regulation for GMDSS? Sea Area A1. Ocean Area Regions AOR-E, AOR-W, POR or IOR. Sea Area A2. Coastal and Inland Waters.
A What sea area is defined as being within range of a shore-based MF station that provides for continuous DSC alerting? Sea area A2. Coastal waters. Sea area A3. Sea area A1.
D Which of the following region lies outside Sea Areas A1, A2, and A3? Sea Areas only apply to Inmarsat footprint areas. Sea Area A3-I (Inmarsat coverage) and Sea Area A3-S (HF SITOR coverage). There are no additional Sea Areas. Sea Area A4.
D If a vessel is engaged in local trade and at no point in its voyage travels outside of the range of a VHF shore station with continuous DSC alerting then the vessel is operating in what area? Coastal and international zones. Inland and coastal waters. Sea areas A1 and A2. Sea area A1.
D GMDSS-equipped ships will be required to perform which of the following communications functions? Distress alerting and maritime safety information. Search and Rescue coordination and on-scene communications. Bridge-to-bridge and general radio communications. All of these.
D What equipment can be used to receive Maritime Safety Information? Navtex. EGC receiver. HF NBDP. All of the above.
B Which communications functions must all vessels be capable of performing under GMDSS as defined by the International Maritime Organization? Radio Direction Finding. Distress alerting to and from vessels, search and rescue coordination, on-scene communications, signals for locating, maritime safety information, general and bridge-to-bridge communications. Communications in each of the operational ocean areas. All communications possible within the International Safety-Net service.
D Which of the following is a functional or carriage requirement for compulsory vessels? A compulsory vessel must carry at least two (2) licensed GMDSS Radio Operators. A compulsory vessel must satisfy certain equipment carriage requirements that are determined by where the vessel sails. A compulsory vessel must be able to transmit and respond to distress alerts. All of the above.
A Which of the following is a required GMDSS function? Bridge-to-Bridge communications. Reception of weather map facsimile broadcasts. Both of the above. None of the above.
C Which of the following is a required GMDSS function? Transmit and receive locating signals. Transmit and receive general communications. Both of the above. None of the above
B Vessels operating in which sea area(s) are required to carry either Inmarsat or HF equipment or a combination thereof under GMDSS? All sea areas. A3 A4 A1
D If operating within Ocean Area A1, and outside of NAVTEX coverage, a GMDSS-equipped vessel must carry? An Inmarsat-A terminal. A GPS receiver. Equipment capable of maintaining a continuous DSC watch on 2187.5 kHz. Equipment capable of reception of maritime safety information by the Inmarsat enhanced group call system, or HF NBDP.
D What is the equipment carriage requirement for survival craft under GMDSS? At least three approved two-way VHF radiotelephones on every passenger ship and cargo ships of 500 gross tons and upwards. At least two approved two-way VHF radiotelephones on every cargo ship between 300-500 gross tons. At least one radar transponder must be carried on every cargo ship of 300-500 gross tons and two transponders (one for each side) of every passenger ship and every cargo ship of 500 gross tons and upward. All of these.
B What statement is true regarding the additional equipment carriage requirement imposed for the survival craft of vessels over 500 gross tons? Additional carriage of two radio equipped lifeboats aft. A second radar transponder is required. Four additional portable VHF radios are required. The ability to communicate in all modes with any shore station.
A Which statement is true regarding a vessel equipped with GMDSS equipment that will remain in Sea Area A1 at all times? The vessel must be provided with a radio installation capable of initiating the transmission of ship-to-shore distress alerting from the position from which the ship is normally navigated. VHF DSC alerting may be the sole means of distress alerting. HF or MF DSC may satisfy the equipment requirement. HF SSB with 2182 kHz automatic alarm generator may satisfy the equipment requirement.
B Within a single sea area, what is the primary reason GMDSS imposes carriage requirements for different radio subsystems? Redundancy in duplicating all operational functions in the event of a system failure. Each subsystem has a specific purpose and capabilities that generally cannot be duplicated by other subsystems. Different radio systems may be used by the various authorities. The ability to communicate in all modes with any of the shore stations.
D A ship operating in sea area A-1 must have the following provisions for maintenance: Shore maintenance. Duplication of equipment. At Sea Maintenance. Any one of the above.
D A ship operating in sea area A-3 must have the following provisions for maintenance: Duplication of Equipment. Shore Maintenance. At Sea Maintenance. Any two of the above.
D What statement is generally correct regarding the maintenance requirements for ships under GMDSS? Redundancy of functions of certain equipment will partially meet this requirement. On-board maintenance provided by a person holding a GMDSS Maintainer's license will partially meet the requirements. Shoreside maintenance and scheduled tests and inspections will partially meet this requirement. All of the above.
D Which of the following is a requirement, under GMDSS, for all vessels over 300 gross tons operating within range of a MF-DSC equipped shore station? Ship's Master or radio officer must be on duty at all times. At least 2nd class Telegraphy license or GMDSS Element 9 is required for the radio officer. Spare parts and maintenance kit for repairs. Only one of the three maintenance options is required.
C Which of the following statements concerning maintenance requirements is false? Compulsory vessels sailing in Sea Areas A1 and A2 must provide any one of the three maintenance options which are duplication of equipment, shore-based, or at-sea maintenance capability. Compulsory vessels sailing in Sea Areas A3 and A4 must provide any two of the three maintenance options which are duplication of equipment, shore-based, or at-sea maintenance capability. If shore-based maintenance is used, maintenance services do not have to be completed or performance verified unless the vessel will be sailing to a non-US port. Equipment warranties do not satisfy GMDSS maintenance requirements.
A Which of the following statements concerning maintenance requirements is true? The options are duplication of equipment, at-sea maintenance, and shore-based maintenance. Compulsory vessels between 300-500 gross tons are required only to provide one maintenance option, while compulsory vessels larger than 500 gross tons and all passenger vessels are required to provide any two of the three maintenance options. The "at-sea" maintenance may be waived if the compulsory vessel carries at least three licensed GMDSS Radio Operators. Compulsory vessels operating in Sea Area A4 are required to carry at least one licensed GMDSS Radio Maintainer.
B What is the frequency range for Medium Frequency? 30-300 kHz 300-3,000 kHz 1,000-10,000 kHz 10-30 MHz
A What is the frequency range for High Frequency? 3-30 MHz 300-3,000 kHz 30-300 MHz 10-30 MHz
C What is the frequency range for Very High Frequency? 3-30 MHz 300-3,000 kHz 30-300 MHz 10-30 MHz
B What is the frequency range for Ultra High Frequency? 3-30 MHz 300-3,000 MHz 30-300 MHz 10-30 MHz
D What is the frequency range for Super High Frequency? 30-300 GHz 300-3,000 MHz 30-300 MHz 3-30 GHz
A What is the primary frequency range for long distance skywave communications? 3-30 MHz 300-3,000 kHz 30-300 MHz 10-30 MHz
A Which of the following systems is most likely to be subject to fading or static interference? HF SITOR. Inmarsat. Digital Selective Calling on channel 70. VHF ARQ.
A Which system is most likely to be affected by atmospheric disturbances? MF/HF radiotelephony. VHF DSC. Inmarsat. SafetyNET.
B Which of the following systems is least likely to be subject to fading or static interference? HF SITOR. Inmarsat. MF-HF DSC Controller. VHF ARQ.
B Which system is least likely to be affected by atmospheric disturbances? NAVTEX. Inmarsat. MF NBDP. HF NBDP.
B Which of the following frequency bands would most likely provide reliable communications between two stations that are 100 miles (160 km) apart? The Low Frequency (LF) band. The Medium Frequency (MF) band. The High Frequency (HF) band. The Very High Frequency (VHF) band.
C Which system has the least effective radius of operation? HF SITOR. MF NBDP. VHF DSC. NAVTEX.
C Which statement best describes amplitude modulation? The character data from the terminal is changed to audio tones. The frequency is varied in synchronization with the modulating signal. The information signal changes the amplitude but does not change the carrier frequency. The amplitude of the carrier is changed but there is still only a single frequency being transmitted.
A What statement best describes modulation? Imposing intelligence onto a radio carrier signal. Changing mark-space to 1 and 0. Adjusting the frequency to the optimum band for long distance communications. Converting the carrier from a low frequency to a higher frequency.
C What statement best describes demodulation? Detuning the receiver to remove interfering signals. Removing atmospheric noise from the signal. Removing the information signal from the carrier. Separating the telex signals from the voice signals.
B What is the emission designation for MF-HF voice signals? F1B J3E J2B F3E
B Which statement best describes frequency modulation? Both the amplitude and frequency are changed by the modulating signal. The frequency is changed by the information signal and the amplitude remains unchanged. Frequency modulation is subject to interference by atmospheric noise. High level mixing of the final amplifier signal and the information signal.
C What is the emission designation for VHF-FM? F3C J2B F3E AME
B Which of the following statements describes the carrier? The carrier consists of at least 3 separate but closely spaced frequencies. The carrier is a Radio Frequency (RF) signal that is modified to carry intelligence. The carrier is used to modulate the information signal. There are always sidebands on either side of the carrier.
C How many sidebands are present in a standard A.M. signal? One Four Two Three
B How many sidebands are present in the J3E mode? Two sidebands and a carrier. One upper sideband. One lower sideband. Two carriers and one sideband.
C What is the signal transmitted in H3E mode? Two sidebands, upper and lower. A reduced carrier and the lower sideband. A full carrier and the upper sideband. A full carrier and both upper and lower sidebands.
C What is the signal transmitted in J2B mode? A full carrier and one sideband. A full carrier and two sidebands. An upper sideband of 2 alternating tones. An upper sideband of a single tone switched on and off.
D Which of the following statements is true? An RF carrier is always required to carry the information. There is only a single tone used in J2B mode. Both of the above. None of the above.
D What is the channel spacing for MF-HF voice frequencies? 2.8 kHz 500 Hz 3.5 kHz 3 kHz
A What is the channel spacing for SITOR frequencies? 500 Hz 170 Hz 300 Hz 3 kHz
A You look up a frequency table and all the listings end in either .0 or .5 kHz. What kind of emission is used with these frequencies? NBDP/SITOR. SSB Voice. Both of the above. None of the above.
C How many SITOR signals can occupy the space of one voice signal? 2 4 6 10
D Which emission mode occupies the most bandwidth? J2B J3E F1B F3E
B Which mode occupies the least bandwidth? H3E J2B AME F3E
C A vertical (whip) antenna has a radiation pattern best described by? A figure eight. A cardioid. A circle. An ellipse.
A A half wave horizontal dipole antenna has a radiation pattern best described by? A figure eight. A cardioid. A circle. An ellipse.
A A vertical quarter wave antenna with a good ground connection will: Act like a half wave antenna. Not function due to being grounded. Only be used in Satellite communications. None of these.
C What is the most common type of antenna for GMDSS MF-HF? Horizontally polarized whip antenna. Long wire antenna. Vertical whip. None of the above.
D What is the most common type of antenna for GMDSS VHF? Horizontally polarized circular antenna. Long wire antenna. Both of the above. None of the above.
B What advantage does a vertical whip have over a long wire? It radiates more signal fore and aft. It radiates equally well in all directions. It radiates a strong signal vertically. None of the above.
B How often must a compulsory vessel's GMDSS radio station be inspected? Annually, by the U.S. Coast Guard. Annually, by the FCC or designated authority. Annually, by the FCC, and every six months if the vessel sails outside of Sea Areas A1 and A2. The FCC's annual inspection may be waived if and only if monthly inspections are performed by the vessel's on-board GMDSS Radio Maintainer.
B What periodic inspection is required in order to remain in compliance with the regulations regarding GMDSS ship radio station inspections? U.S. Coast Guard annual inspection. F.C.C. annual inspection. F.C.C. inspection every five years. Periodic inspections not required if on board maintainers perform routine preventive maintenance.
B Which statement is false regarding a GMDSS-equipped ship? A conditional or partial exemption may be granted, in exceptional circumstances, for a single voyage outside the sea area for which the vessel is equipped. Ships must have the required equipment inspected at least once every five years. The regulations apply to all passenger ships regardless of size and cargo ships of 300 gross tons and upwards. Ships must carry at least two persons holding a GMDSS Radio Operator's license for distress and safety radio-communications purposes.
A Which statement is false regarding a GMDSS equipped ship? A conditional or partial exemption may not be granted, in exceptional circumstances, for a single voyage outside the sea area for which the ship is equipped. Ships must have the required equipment inspected at least once every 12 months. The regulations apply to all passenger ships regardless of size and cargo ships of 300 gross tons and upwards. Ships must carry at least two persons holding a GMDSS Radio Operator's license for distress and safety radio-communications purposes.
D During an annual FCC inspection __________. All required documents and publications might have to be produced. Licensed GMDSS operators may be required to demonstrate equipment competencies. All required equipment must be fully operational. All of the above.
C Foreign governments or administrations may inspect the radio installation: When the ship's station license cannot be produced without delay. When irregularities are observed. Both of the above. None of the above.
D Where can GMDSS regulations pertaining specifically to U.S.-flag vessels be found? These are located in CCIR #476. These are located in FCC Part 83. These are published only by the U.S. Coast Guard. These are located in 47 CFR 80.
B Which of the following references should the GMDSS Radio Operator consult for information on the proper operation of equipment? ITU List of Equipment Operations. The manufacturer's operator manuals. 47 CFR Part 80. Information is available through SafetyNET channels.
D What should the GMDSS Radio Operator consult to review the proper procedures to be followed in distress situations under GMDSS? IMO Recommendations. The manufacturer's instruction manuals. Part 90 of the FCC Rules and Regulations. Part 80, Subpart W of the FCC Rules and Regulations.
D Which of the following documents are required by GMDSS for vessels on international voyages (other than the Great Lakes)? A copy of the IMO master plan of shore-based facilities. Station logs. Part 80 FCC rules and regulations. All of these.
D Which of the following documents are required by Part 80 of the FCC rules for vessels on international voyages (other than the Great Lakes)? Appropriate Safety Convention Certificate. List of Call Signs and Numerical Identities. (ITU List VII-A) List of Radiodetermination and Special Service Stations. (ITU List VI) All of these.
B What publication/s should a GMDSS Operator consult regarding the proper set-up and operation of vessel equipment? ITU Publications. The manufacturer's instruction manuals. Part 90 of the FCC Rules and Regulations. Code of Federal Regulations, Title 47, Part 80, Subpart W.
D Which of the following maintenance functions is not the responsibility of the GMDSS Radio Operator? Visual inspection of equipment, including the antenna and associated components. Perform on-the-air verification checks. Perform scheduled testing of the battery's charged condition. Aligning the power output stage for maximum power.
C When may a compulsory vessel not be allowed to leave port? When the vessel is in an over-carriage condition. When the vessel has arranged for both duplication of equipment AND shore-based maintenance. When the vessel has replaced a required piece of GMDSS-related equipment but its performance has not been verified or logged. When the vessel is carrying only two licensed GMDSS Radio Operators and is capable of performing all required functions.
A Which statement is false regarding the maintenance of GMDSS equipment at sea? The GMDSS maintainer may not be the person designated to have primary responsibility for radio-communications during distress incidents even if licensed as an operator. Ships must carry at least one person who qualifies as a GMDSS maintainer for the maintenance and repair of equipment if the at-sea maintenance option is selected. All at-sea maintenance and repairs must be performed by, or under the supervision of a person holding a GMDSS Maintainer license. The GMDSS maintainer may be the person responsible for ensuring that the watches are properly maintained and that the proper guard channels and the vessel's position are entered into the DSC equipment.
C Which of the following service or maintenance functions may NOT be performed by the holder of a GMDSS Radio Operator License? Reset tripped circuit breakers or replace defective fuses. Routine battery maintenance if used as part of the GMDSS station. Any adjustments or maintenance that may affect the proper operation of the station. Replacement of consumable items such as paper, ribbons, etc.
A What are the conditions, under GMDSS, whereby a ship is NOT allowed to depart from any port? The vessel is not capable of performing all required distress and safety functions. The vessel is carrying more than the required number of qualified GMDSS radio operators. The vessel has a temporary waiver of its radio license and Safety Certificate. The vessel is not carrying a GMDSS radio maintainer, but has provided for shoreside maintenance plus duplication of equipment if required.
A What determines the spares and maintenance materials requirements for the SITOR equipment under GMDSS? The recommendations of the manufacturer. FCC Part 80. IMO Circular "Equipment Spares". The GMDSS Maintainer's requirements.
C Which of the following is the minimum license requirement of a GMDSS Radio Operator? Holding the Marine Radio Operator Permit is a pre-requisite before the GMDSS Radio Operator Endorsement can be obtained. Holding the General Radiotelephone Operator License with RADAR endorsement is sufficient. Holding a valid GMDSS Radio Operator license is sufficient. Holding either the General Radiotelephone Operator License or the First or Second Class Radiotelegraph license with GMDSS Radio Maintainer's endorsement is sufficient.
C Which of the following statements concerning GMDSS Radio Operator requirements is false? Each compulsory vessel must carry at least two licensed GMDSS Radio Operators at all times while at sea. Each compulsory vessel must carry at least two licensed GMDSS Radio Operators at all times while at sea and may elect to carry a GMDSS Radio Maintainer as well. Communications involving safety of life at sea do not have to be logged as long as the compulsory vessel was not involved in such communications. While at sea, adjustments to, and the maintaining of, GMDSS equipment may be performed by the GMDSS Radio Operator as long as the work is supervised by an on-board licensed GMDSS Radio Maintainer.
D What is the minimum operator license required to perform or supervise the performance of at-sea adjustments, servicing or maintenance which may affect the proper operation of the GMDSS station? Marine Radio Operator Permit. General Radiotelephone Operator license and Radar endorsement. Designated maintainer possessing a GMDSS Radio Operator license. GMDSS Radio Maintainer's license.
C Which statement is false regarding the radio operator requirements for a GMDSS-equipped ship station? One of the qualified GMDSS radio operators must be designated to have primary responsibility for radio-communications during distress incidents. A qualified GMDSS radio operator, and a qualified backup, must be designated to perform distress, urgency and safety communications. Maintaining a record of all incidents connected with the radio-communications service that appear to be of importance to safety of life at sea is not required. While at sea, all adjustments or radio installations, servicing or maintenance of such installations that may affect the proper operation of the GMDSS station must be performed by, or under the supervision of, a qualified GMDSS radio maintainer.
D What are the vessel equipment and personnel requirements of GMDSS? Two licensed GMDSS radio operators. Equipment carriage requirements. Distress alerting and response. All of these.
C What is the minimum requirement of a GMDSS radio operator? Marine Radio Operator Permit and GMDSS Endorsement. General Radiotelephone Operator license and Radar endorsement. GMDSS Radio Operator license. General Radiotelephone license or First or Second Class Radiotelegraph license with GMDSS Radio Maintainer's endorsement.
B Which statement is false regarding the GMDSS requirement for ship sources of energy? At all times while the vessel is at sea, a sufficient supply of electrical energy to operate the radio installations and charge any batteries which may be part of the reserve source of energy is required. The reserve sources of energy need to supply independent MF and HF radio installations at the same time. An uninterruptible power supply or other means of ensuring a continuous supply of electrical power to all GMDSS equipment that could be affected by normal variations and interruptions of ship's power is required. If an uninterrupted input of information is required from the ship's navigational or other equipment to a GMDSS radio station to ensure its proper performance, a means must be provided to ensure such continuous supply of information in the event of failure of the ship's main or emergency source of energy.
A What is the meaning of “Reserve Source of Energy”? The supply of electrical energy sufficient to operate the radio installations for the purpose of conducting distress and safety communications in the event of failure of the ship's main and emergency sources of electrical power. High caloric value items for lifeboat, per SOLAS regulations. Diesel fuel stored for the purpose of operating the powered survival craft for a period equal to or exceeding the U.S.C.G. and SOLAS requirements. None of these.
B What is the term which refers to the supply of electrical energy required to supply radio installations on every ship for the purpose of conducting distress and safety radio-communications in the event of failure of the ship's main and emergency sources of electrical power? Emergency power. Reserve source of energy. Ship's emergency diesel generator. Ship's standby generator.
D What are the characteristics of the Reserve Source of Energy under GMDSS? Supplies independent HF and MF installations at the same time. Cannot be independent of the propelling power of the ship. Must be incorporated into the ship's electrical system. Must be independent of the ship's electrical system when the RSE is needed to supply power to the GMDSS equipment.
D What is the requirement for emergency and reserve power in GMDSS radio installations? An emergency power source for radio communications is not required if a vessel has proper reserve power (batteries). A reserve power source is not required for radio communications. Only one of the above is required if a vessel is equipped with a second 406 EPIRB as a backup means of sending a distress alert. All newly constructed ships under GMDSS must have both emergency and reserve power sources for radio communications.
A Which of the following terms is defined as a back-up power source that provides power to radio installations for the purpose of conducting distress and safety communications when the vessel's main and emergency generators cannot? Reserve Source of Energy (RSE). Emergency Diesel Generator (EDG). Reserve Source of Diesel Power (RSDP). Emergency Back-up Generator (EBG).
D The best way to test the MF-HF NBDP system is? Make a radiotelephone call to a Coast Station. Initiate an ARQ call to demonstrate that the transmitter and antenna are working. Initiate an FEC call to demonstrate that the transmitter and antenna are working. Initiate an ARQ call to a Coast Station and wait for the automatic exchange of answerbacks.
A Under GMDSS, a compulsory VHF-DSC radiotelephone installation must be tested at what minimum intervals at sea? Daily. Annually, by a representative of the FCC. At the annual SOLAS inspection. Monthly.
D Testing of a compulsory radiotelephone station should be done? Into an artificial antenna. May be accomplished by using the radiotelephone for normal business. On 2182 kHz and must be heard clearly under normal conditions at a range of 150 nautical miles. Either a) or b).
A DSC controllers have a facility that allows the unit to be routinely tested without the associated transmitter being activated. The unit should be tested while underway at least? Once a Day. Once a week. Twice a week. Once a month.
B At sea, all required equipment (other than Survival Craft Equipment) must be proven operational by? Daily testing. By either a) or c). Operational use of the equipment. Testing at least every 48 hours.
B The best way to test the Inmarsat-C terminal is? Send a message to a shore terminal and wait for confirmation. Compose and send a brief message to your own Inmarsat-C terminal. Send a message to another ship terminal. If the send light flashes, proper operation has been confirmed.
D A vessel certified for service in Sea Area A3 is required to maintain a watch on? VHF Channel 70. MF Frequency 2187.5. HF on 8414.5 kHz and one other HF DSC frequency. All of these.
B A vessel certified for service in Sea Area A-2 is required to maintain watch on? 2174.5 kHz 2187.5 kHz 2182.0 kHz 2738.0 kHz
C What are the mandatory DSC watchkeeping bands/channels? VHF Ch-70, 2 MHz MF DSC, 6 MHz DSC and 1 other HF DSC. 2 MHz MF DSC, 8 MHz DSC, VHF Ch-16 and 1 other HF DSC. 8 MHz HF DSC, 1 other HF DSC, 2 MHz MF DSC and VHF Ch-70. None of the above.
D Proper watchkeeping includes the following: All required frequencies are being monitored in the proper mode. After silencing an alarm all displays and/or printouts are read. Notifying the Master of any distress alerts. All of the above.
D Proper watchkeeping includes the following: Understanding the GMDSS console's normal operational indicators. Maintaining a proper GMDSS radio station log. Responding to and comprehending alarms. All of the above.
D Which is true concerning a required watch on VHF Ch-16. It is compulsory at all times while at sea until Feb 2005 unless the vessel is in a VTS system. When a vessel is in an A1 sea area and subject to the Bridge-to-Bridge act and in a VTS system, a watch is not required on Ch-16 provided the vessel monitors both Ch-13 and VTS channel. It is always compulsory in sea areas A2, A3 and A4. All of the above.
C Which of the following statements are true? GMDSS Radio Logs are required to contain entries pertaining to all incidents connected with the radio-communications service that appear to be of importance to the safety of life at sea. All distress communications must be entered in the GMDSS radio log. Both of the above. None of the above.
D Which of the following statements are true? Key letters or abbreviations may not be used in GMDSS Radio Logbooks under any circumstances. Urgent communications do not need to be entered in the GMDSS radio log. Both of the above. None of the above.
C Where should the GMDSS radio log be kept on board ship? Captain's office. Sea cabin. At the GMDSS operating position. Anywhere on board the vessel.
D How long must the radio log be retained on board before sending it to the shoreside licensee? At least one year after the last entry. At least two years after the last entry. At least 90 days after the last entry. At least 30 days after the last entry.
C How long must the radio log be archived by the licensee? Two years if there is no distress or urgent entries. Three years if there are any distress or urgent entries. Both of the above. None of the above.
B Which of the following log keeping statements is true? Entries relating to pre-voyage, pre-departure and daily tests are required. Both a) and c). All distress, urgent and safety communications must be logged. Routine daily MF-HF and Inmarsat-C transmissions do not have to be logged.
B A typical call sign for a large container ship under U. S. flag would be: WBX1469 KBZY NADN KPH
C What would the number 1090 indicate? A ship MMSI number. A coast station MMSI number. A coast station SELCALL number. A ship station SELCALL number.
A Which one of the following is a ship station SELCALL? 11243 1104 1502352 230364973
C Which of the following is the call sign for a U.S.C.G. coast station? NERK KPH NMN WCC
C What type of station would be assigned the call sign WAB2174? Container ship Passenger ship Tug boat Bulk Tanker
B What number will a ship station use to identify itself using SITOR? Four digit SELCALL. Five digit SELCALL or 9 digit SELCALL number identical to MMSI. 9 digit Inmarsat-B I. D. number. 9 digit Inmarsat-C I.D. number.
B What is the MID? Mobile Identification Number. Maritime Identification Digits. Marine Indemnity Directory. Mobile Interference Digits.
C How many digits are in the MID (Maritime Identification Digits)? 7 9 3 10
B What does the MID (Maritime Identification Digits) signify? Port of registry. Nationality. Gross tonnage. Passenger vessel.
C Which of the following numbers indicates a U.S. flag ship station? 36627934 243537672 338426791 3382315
B Which of the following MMSI numbers indicates a U.S. flag ship station? a) 430326890
b) 303236824
c) 033609991
d) 257326819
B Which of the following numbers indicates a ship station MMSI? 3372694 623944326 30356328 3384672
C A DSC call is received from a station with a MMSI number of 003669991. Which of the following types of stations is it from? A vessel operating in Sea Area A3. A group ship station. A U.S. coast station. An Intercoastal vessel.
B A valid MMSI number for a DSC call to a specific group of vessels is: 3664523 30327931 338462941 3036483
C MMSI 030346239 indicates what? A group ship MMSI issued in Hawaii. Coast station on Atlantic Coast. A group ship MMSI issued in Alaska. Ship station registered from San Francisco.
B Which of the following statements concerning MMSI is true? Coast station MMSI numbers have 9 digits starting with 4. All MMSI numbers are 9 digits and contain an MID. Ship station MMSI numbers can be 7 digits or 9 digits depending on the Inmarsat terminal. Group MMSI numbers must begin with 2 zeros.
D Which of the following statements concerning MMSI is true? All Coast Station MMSI must begin with 2 zeros. The first 3 digits of a ship MMSI comprise the MID. A group call must begin with a single zero followed by the MID. All of the above.
C Which of the following statements concerning MMSI is true? All ship station MMSI must begin with a single zero and include the MID. All group station MMSI must begin with the MID. All Coast Station MMSI must be 9 digits and begin with 2 zeros followed by the MID. None of the above.
C Which of the following would indicate an Inmarsat-B terminal? A 9 digit number beginning with the MID. A 7 digit number A 9 digit number always starting with 3. A 9 digit number always starting with 4.
C Which of the following would indicate an Inmarsat-B terminal? 3662517 436682011 325468325 1500241
B Which of the following would indicate an Inmarsat-B terminal? 150036 366832922 430364290 1502460
C Which of the following would indicate an Inmarsat-B terminal? 3668202 436682433 325468263 1500270
B Which of the following would indicate an Inmarsat-B terminal? 150036 366632824 430363275 1502460
A Which of the following would indicate an Inmarsat-B terminal? A 9 digit number always starting with 3. A 12 digit number starting with the MID. A 9 digit number ending with the MID. A 9 digit number always starting with 4.
A You receive a telex with the senders I.D. of 433863491. What type of terminal sent this message to your vessel? Inmarsat-C. Land telex terminal. Inmarsat-B. Inmarsat-A.
D You receive a telex with the senders I.D. of 1502340. What type of terminal sent this message to your vessel? Inmarsat-C. Land telex terminal. Inmarsat-B. Inmarsat-A.
B Which of the following would indicate an Inmarsat-C terminal? 3668202 436682433 325468263 1500270
C Which of the following would indicate an Inmarsat-C terminal? 150036 366294824 430346275 1502690
B Which of the following would indicate an Inmarsat-C terminal? 3662517 436682011 325468325 1500241
D Which of the following would indicate an Inmarsat-C terminal? A 9 digit number beginning with the MID. A 7 digit number. A 9 digit number always starting with 3. A 9 digit number always starting with 4.
B What is first sent by all MF-HF DSC transmissions? Distress alert character. A string of dots to stop the scan receiver. Priority of transmission character. Ship's position.
D VHF-DSC transmissions are encoded and transmitted in what emission mode? H3E F1B/J2B A3E None of the above.
B When sending a DSC call: Vessel’s position will automatically be sent. Vessel’s position will automatically be sent if the vessel is sending a “Distress Hot Key” alert. Vessel’s MMSI will indicate its ocean region. None of these.
B DSC transmissions are encoded: Using J3E mode for proper follow-on communications. Using a special digital format. Using F1B mode to ensure proper reception. Using J2B mode for correct transmission.
C DSC transmissions are received: Using voice or telex modes as appropriate. Using J3E or H3E modes as appropriate. Using digital decoding by the DSC controller. Using F1B and/or J2B decoding by the transceiver.
C Properly formatted DSC transmissions can request which of the following emissions for follow on communications? J3E/H3E telex emissions. F1B/J2B voice emissions. J3E/H3E voice emissions. None of the above.
D Which of the following statements concerning DSC equipment is true? The GMDSS Radio Operator is responsible for properly selecting HF DSC guard channels. All equipment must be type accepted. The vessel's navigational position must be updated, either automatically or manually, no less often that every four (4) hours. All of the above.
B What is the action that a GMDSS Radio Operator should take when a DSC distress alert is received? No action is necessary, as the DSC control unit will automatically switch to the NBDP follow-on communications frequency. The Operator should immediately set continuous watch on the radiotelephone frequency that is associated with frequency band on which the distress alert was received. The Operator should immediately set continuous watch on VHF channel 70. The Operator should immediately set continuous watch on the NBDP frequency that is associated with frequency band on which the distress alert was received.
C What does the DSC control unit do if the GMDSS Radio Operator fails to insert updated information when initiating a DSC distress alert? It will abort the transmission and set off an audible alarm that must be manually reset. It will initiate the DSC distress alert but, as no information will be transmitted, rescue personnel will not be able to identify the vessel, its position, or its situation. It will initiate the DSC distress alert, and default information will automatically be transmitted. It will initiate the DSC distress alert, but any station receiving it will have to establish contact with the distressed vessel to determine its identity, position, and situation.
C A DSC Distress alert: Must always be sent on VHF Ch-70. Must always be sent on MF 2 MHz plus one other HF DSC frequency. Will always be sent on one or more of the DSC distress frequencies. Must always be sent on VHF Ch-70, then 2 MHz MF then 8 MHz HF.
C In all cases, the transmit frequency of a MF/HF console DSC distress alert: Will go out first on 2187.5 kHz. Will go out on 8 MHz and 2 MHz and one other DSC distress frequency. It depends upon operator DSC Call set up entries. None of the above.
C DSC relays of Distress Alerts by vessels: Should be done for all Distress Alerts. Should be transmitted to ships involved in Distress traffic. Should be avoided, however after repeated alerts, should be relayed to a Coast Station nearest the distress incident. Are the best means to provide for a relay of Distress communications.
C Which of the following is not a DSC watch frequency? 2187.5 kHz. 6312 kHz. 2182 kHz. 12577 kHz.
D Which of the following channels and modes should be used when initiating a distress alert transmission? Ch-6 DSC. Ch-6 Radiotelephony. Ch-13 Radiotelephony and Ch-16 DSC. Ch-70 DSC.
D How many total frequencies are available for DSC distress alerting? One (1). Two (2). Five (5). Seven (7).
D Which of the following watches must a compulsory vessel maintain when sailing in Sea Area A1? A continuous DSC watch on 8414.5 kHz plus one other HF DSC frequency. A continuous DSC watch on 2187.5 kHz. A continuous DSC watch on Ch-16. A continuous DSC watch on Ch-70.
C Which of the following are the MF/HF DSC Distress watch frequencies 2177.5, 4210.0, 6314.0, 8416.5 12579.0, 16806.5 2182.0, 4125.0, 6215.0, 8291.0, 12290.0, 16420.0 2187.5, 4207.5, 6312.0, 8414.5, 12577.0, 16804.5, 156.525 Mhz (VHF 70) 2174.5, 4177.5, 6268.0, 8376.5, 12520.0, 16695.0
B How many HF DSC distress watch channels must be guarded by a compulsory vessel underway. 1 2 3 4
B What is usually the first step for a GMDSS Radio Operator to take when initiating a distress priority message via Inmarsat? By dialing the correct code on the telephone remote unit. By pressing a "Distress Button" or "Distress Hot Key(s)" on the equipment. By contacting the CES operator and announcing a distress condition is in existence. By contacting the CES operator using the radiotelephone distress procedure "Mayday"... etc.
A Which of the following statements is true regarding distress alerting under GMDSS? The distress alert must identify the station in distress and its position and may additionally include information regarding the nature of the distress, the type of assistance required, the course and speed of the mobile unit and the time this information was recorded. Ship to shore distress alerts are used to alert other ships in port of navigational hazards. Ship-to-ship distress alerts are used to alert other ships in the vicinity of navigational hazards and bad weather. The vessel nearest to the emergency must notify the Coast Guard before leaving the vicinity.
C If a GMDSS Radio Operator initiates a DSC distress transmission but does not insert a message, what happens? The transmission is aborted and an alarm sounds to indicate this data must be provided by the operator. The transmission is not initiated and "ERROR" is indicated on the display readout. The transmission will be made with "default" information provided automatically. The receiving station will poll the DSC unit of the vessel in distress to download the necessary information.
C Repetition of a DSC Distress call is normally automatic if not acknowledged after a delay of: 1 - 2 minutes. 2- 5 minutes. 3.5 - 4.5 minutes. Not at all.
D A MF/HF DSC Distress call: Will send the minimal necessary information using the "Distress Button" or "Distress Hot Key" Contains all the information normally of interest in on-scene Distress communications. Will send a more detailed Distress format if time permits and operator data entries are correctly performed. Both A and C are true
D Which statement regarding an MF/HF DSC Distress call is true: Follow on communications should be presumed to take place on the telex frequency associated with the specific DSC frequency used. Follow on communications should be presumed to take place on the voice frequency associated with the specific DSC frequency used. An alternate emission and frequency may be specified for follow-up communications by the vessel originating the Distress Alert. Both b) and c) are true.
D You send a VHF-DSC distress alert. What channel do you use for the follow-on voice transmission? Ch-12 Ch-70 Ch-13 Ch-16
C You send a MF-DSC distress alert. What frequency do you use for the follow-on voice transmission? 2760 kHz 2187.5 kHz 2182 kHz 2174.5 kHz
B You send a HF-DSC alert on 8414.5 kHz. What frequency do you use for the voice transmission? 8376.5 kHz 8291.0 kHz 8401.5 kHz 8201.0 kHz
D What is the proper format for a distress follow on voice transmission? (3x is three times), All Ships 3x this is Ship's Name/Call Sign 3x Mayday Position. Mayday 3x this is Ship's Name/Call Sign 3x Distress category. Both of the above. None of the above.
D What information should be included in a distress follow on voice transmission? Ship's Name and Call Sign. Ship's position. Ship's MMSI number. All of the above.
C What information should be included in a distress follow on voice transmission? Follow on working frequency. ETA at next port. None of the above. Both of the above.
A Which statement is true regarding the receipt and acknowledgement of distress alerts by ship stations and ship earth stations (SES)? A ship or ship earth station that receives a distress alert must, as soon as possible, inform the Master or person responsible for the ship of the contents of the distress alert. Ship stations in receipt of distress alert should not defer acknowledgement for a short interval, so that receipt may be acknowledged by the coast station. Ship stations operating in areas where reliable communications with a coast station are not practicable, that receive a distress alert from a vessel in their vicinity, must either acknowledge receipt of and inform the Rescue Coordination Center or, if received by HF, wait for 3 minutes for a coast station acknowledgement before performing any relay of the distress alert. Alerts concerning navigational hazards are second only to safety traffic.
B What does the acronym 'EOS" indicate in the received message? Error of Sequence. End of Sequence. End of Signals. Equal Operating Signals.
B What is the proper procedure to be followed upon receipt of a distress alert transmitted by use of Digital Selective Calling techniques? Set watch on the DSC alerting frequency in the band of frequencies the alert was received. Set watch on the radiotelephone distress and safety frequency associated with the distress and safety calling frequency on which the distress alert was received. Set a continuous watch on VHF-FM Channel 13, 16 and DSC on Channel 70. Ship stations equipped with narrow-band direct-printing equipment should respond to the distress alert as soon as practicable by this means.
C What does the acronym "ECC" indicate in the received message? Every Cipher Counted. Error Cannot Confirm. Error Check Character. Even Characters Counted.
D What action should be taken on receipt of a distress alert? Read the display screen and/or printout. Silence the alarm. Listen for any follow on voice/telex transmission on the appropriate frequency. All of the above.
B What action should be taken if a distress alert is received on the 12 MHz DSC frequency? Use DSC to acknowledge the alert using the 12 MHz DSC frequency. Set the receiver to 12290.0 kHz J3E. Do nothing. Ship is too far away to render assistance. Set the receiver to 12520.0 kHz F1B.
C Your ship received a distress relay from the U.S. Coast Guard on DSC freq. 2187.5 kHz. You would acknowledge by radiotelephony on what frequency? 4207.5 kHz 8290.0 kHz 2182.0 kHz 6312.0 kHz
D Your ship received a Distress relay on DSC VHF channel 70, on what channel would you reply? Ch-70 Ch-06 Ch-13 Ch-16
C Under what conditions would you relay a DSC distress alert? If the mobile unit in distress is incapable of further Distress Alert communications. If no Coast Station/Mobile Unit acknowledgement is observed. Answers a) and b) are both possible. You should never relay such an alert -- the Coast Station & RCC will do that.
D The relay of DSC Distress Alerts: Has completely overburdened the GMDSS system with improperly formatted or inappropriately relayed DSC calls. Was originally an intended function of the GMDSS system. Is no longer the preferred method for passing Distress message traffic to an RCC or Coast Station. All of the above.
D Transmission of a distress alert by a station not in itself in distress should occur: When the mobile unit actually in distress is not itself in a position to transmit the distress alert. When the Master or responsible person on the mobile unit not in distress so decides. When the responsible person at the Coast Station determines further help is necessary. In some cases, all of the preceding situations may justify a Distress Alert relay.
D Relays of Distress Alerts using DSC may still be done. However, it is now recommended that such relays be done: Only by Inmarsat-C telex with Distress Priority. Only by Inmarsat-A voice or telex with Distress priority. Preferably by MF/HF voice or telex directly to the RCC. By any of the above methods which will effectively provide Distress communications to an RCC or Coast station without further activations of other Mobile Units' DSC controllers.
B What action should you take after sending a false distress alert on VHF? Send a DSC cancellation message on Ch-70. Make a voice announcement to cancel the alert on Ch-16. Make a voice announcement to cancel the alert on Ch-13. Make a voice announcement to cancel the alert on Ch-22A.
C What action should you take after sending a false distress alert on MF? Make a voice announcement to cancel the alert on 2187.5 kHz. Make a voice announcement to cancel the alert on 2174.5 kHz. Make a voice announcement to cancel the alert on 2182.0 kHz. Send another DSC alert and follow on with voice on 2182.0 kHz.
D What action should you take after sending a false distress alert on MF? Send another DSC alert on 2187.5 kHz. and follow on with voice on 2187.5 kHz. No action is necessary. Send a DSC alert on all 7 DSC frequencies and follow on voice on 2174.5 kHz. Make a voice announcement to cancel the alert on 2182.0 kHz.
B What action should you take after sending a false distress alert on 8 MHz? Make an "ALL SHIPS" call on all 5 H.F. telex channels canceling the alert. Make an "ALL SHIPS" call on 8291.0 kHz canceling the alert. Make a "MAYDAY" call on 8414.5 kHz canceling the alert. Make an "URGENT" call on 8614.0 kHz canceling the alert.
C What action should you take after sending a false distress alert on 12577.0 kHz? No action is necessary. Make an "ALL SHIPS" call on all 5 H.F. telex frequencies canceling the alert. Make an "ALL SHIPS" call on the associated 12 MHz J3E frequency canceling the alert. Send a message to the nearest RCC via Inmarsat canceling the alert.
B What action should you take after sending a false distress alert on Inmarsat-C? Press the "Distress Hot Keys” then press the "cancel" key. Select "Transmit" menu and send a cancel message via the CES used for the distress alert. Both of the above. None of the above.
D What is the fundamental purpose for imposing radio silence? To ensure that interference to proprietary communications is minimized. To ensure that only voice communications can be effected on the distress frequency or channel. To ensure that a distressed vessel will have a "window" twice each hour for transmitting routine messages. To ensure that interference on a particular frequency or channel for communications concerning emergency traffic is minimized.
C When can routine communications be resumed when radio silence has been imposed? After determining that the frequency or channel appears to be no longer in use. After determining that geographic distance from the distress situation will prohibit any other signal from interfering with emergency communications. Routine communications can resume after the Rescue Coordination Center transmits a message on the frequency or channel being used for emergency communications stating that such traffic has concluded. If, in the master's opinion, communications on that frequency will interfere with emergency communications.
A What is meant by the term "radio silence"? Stations not directly involved with the on-going distress communications may not transmit on the distress frequency or channel. Stations remaining off the air to safeguard proprietary information. Two three-minute silent periods, at 15 and 45 minutes after the hour, that provide a transmitting "window" for distressed vessels to transmit distress alerts using J3E. Communications on a distress frequency or channel is banned for 24 hours following the cessation of the distress traffic.
A How is "radio silence" imposed? By the On Scene Coordinator (OSC). By the Coast Earth Station (CES) controlling the distress communications on that frequency. By the nearest Public Correspondence Coast Station. By the vessel first responding to the distress call.
A How are normal working conditions restored on a narrow band direct printing (NBDP) frequency on which radio silence had been imposed? The RCC that imposed the radio silence must transmit a NBDP message stating "SILENCE FINI". The CES that imposed the radio silence must transmit a NBDP message stating "SILENCE FINI". The Public Correspondence Station (PCS) that imposed the radio silence must transmit a narrow band direct printing message on the distress frequency stating "SILENCE FINI". The High Seas Service (HSS) that imposed the radio silence must transmit a narrow band direct printing message on the distress frequency stating "SILENCE FINI".
A How are normal working conditions restored after radio silence has been imposed? The Rescue Coordination Center (RCC) that imposed the radio silence must transmit a voice message on the distress frequency stating "SEELONCE FEENEE". The Coast Earth Station (CES) that imposed the radio silence must transmit a voice message on the distress frequency stating "SILENCE FINI". The Public Correspondence Station (PCS) that imposed the radio silence must transmit a voice message on the distress frequency stating "SILENCE FINI". None of the above.
B The Radiotelephone Urgency signal is: Mayday Pan Pan Securite Seelonce Feenee
D Which of the following situations would normally use the Urgency priority? A crewmember over the side. A serious medical situation involving a crewmember. A cargo shift or weather situation considered to be of greater hazard than would justify a SAFETY priority designation. Answers b) and c).
B Which of the following situations would normally use the Urgency priority? Leaking oil from a minor tank fracture. Treatment of crewmember breaking a leg in a cargo hold. A fire in the generator flat/spaces. Answers a) and b) are both possible.
B Which of the following situations would normally use the urgency priority? A crewmember over the side. A serious medical situation involving a crewmember. Both a and b Scenarios concerning the Safety of navigation or important meteorological warnings.
C The Urgent Priority should be used for: Messages concerning the Safety of Life At Sea (SOLAS). Messages detailing important navigational warnings. Messages containing information concerning the safety of a mobile unit or person. Messages concerning On-scene communications .
B If the Watch Officer hears "PAN PAN" spoken 3 times it means: A navigation or important meteorological warning should follow. The station is preparing to transmit an URGENT message possibly concerning the safety of a mobile unit or person. A Mobile unit is in need of immediate assistance. None of the above.
A When the GMDSS Radio Operator on watch hears "SECURITE" spoken three times, he can expect to receive the following information: Message concerning the Safety of navigation or important meteorological warnings. Safety of vessel or person is in jeopardy. Vessel in need of immediate assistance. Coast Station Traffic list.
B Which of the following situations would normally use the Voice designation "Securite"? Messages concerning the Safety of Life At Sea (SOLAS). Messages detailing important navigational warnings. Messages containing information concerning the safety of a mobile unit or person. Messages concerning On-scene communications .
A Which of the following situations would normally use the Safety priority? Loss of 5 containers with lashing gear over the side. Treatment of crewmember breaking a leg in a cargo hold. A fire in the generator flat/spaces. Answers a) and b) are both possible.
D Which of the following situations would normally use the Safety priority? A crewmember over the side. A serious medical situation involving a crewmember. Both a) and b). Scenarios concerning the Safety of navigation or important meteorological warnings.
A The Radiotelephone Safety signal is: "Securite" repeated 3 times. "Safety Safety Safety". "Pan Pan" repeated 3 times. "Securite Securite" repeated 3 times.
C Which of the following situations would normally use the Safety priority? A crewmember over the side. A serious medical situation involving a crewmember. A scenario concerning an important navigational or meteorological warning. All of the above.
D Which of the following frequencies and modes is allocated for distress alerting in GMDSS? 406 MHz via EPIRB. 1626.5-1645.5 MHz via Inmarsat. Channel 70 DSC plus six (6) MF/HF DSC frequencies. All of the above.
D Which of the following frequencies is normally used for distress and safety communications? 490 kHz 518 kHz 4209.5 kHz 2174.5 kHz
D Which channel is designated for GMDSS Digital Selective Calling? Ch-06 Ch-13 Ch-16 Ch-70
A How many MF frequencies are available for DSC distress related calls? One Two Four Five
D How many HF frequencies are available for DSC distress related calls? One Two Four Five
D How many frequencies are available under GMDSS for DSC distress-related calls? Two Four Five Seven
C Which of the following steps should be taken, if possible, when the vessel must be abandoned because of a distress situation? Alert the U.S. Coast Guard by using the survival craft's portable INMARSAT unit. Program the SART and EPIRB to transmit the vessel's location and situation. Place the SART and EPIRB in the "on" position and secure them to the survival craft. No additional steps are needed as the SART and EPIRB will both automatically float free and operate properly.
B Which action is the most appropriate action for a GMDSS radio Operator to take in a distress situation where immediate help is needed, but the vessel is not sinking nor needs to be abandoned? Switch off EPIRB and SART manually. Transmit distress call by HF/MF/VHF DSC or Inmarsat. Notify the RCC (Rescue Coordination Center) through VHF FM on channel 13. Transmit distress call by activating the radiotelegraph automatic alarm signal.
C DSC is used primarily to: Receive weather warnings, navigational notices and other marine safety information. Provide routine communications with the ship owner. Transmit and receive distress, urgent and safety alerts to and from other ships and shore stations via radio. Report ship's position to search-and-rescue authorities via satellite.
D GMDSS vessels equipped for A2, A3 or A4 must maintain a continuous DSC watch on 2187.5 kHz. Only in areas beyond Inmarsat coverage. Only outside of areas covered by VHF-DSC. When directed to do so by a cognizant rescue authority. At all times when underway.
D Which statement is true regarding distress communications under GMDSS? Distress communications by NBDP should be in the ARQ mode when in communications with the Coast Guard or other coast stations. The Rescue Coordination Center (RCC) is responsible for controlling a search and rescue operation and will also coordinate the distress traffic relating to the incident. The Rescue Coordination Center may appoint another station to coordinate distress traffic relating to the incident. All of these.
D When operating in coastal waters (sea area A1), a GMDSS-equipped vessel must: Maintain a continuous DSC watch on 8514.5 kHz. Maintain a continuous aural watch on 2182 kHz. Maintain a continuous DSC watch on VHF channel 16. Maintain a continuous DSC watch on VHF channel 70.
A What is the purpose of the SART's audible tone alarm? It informs survivors that assistance may be nearby. It informs survivors when the battery's charge condition has weakened. It informs survivors when the SART switches to the "standby" mode. It informs survivors that a nearby vessel is signaling on DSC.
A What indication is given to the personnel of survival craft of the approach of another vessel? The SART will provide a visual or audible indication of interrogation by a 3-cm radar. The Satellite EPIRB will emit an audible signal. The VHF portable radio will emit an audible alarm signal on 156.525 MHz. The VHF portable will provide a visual indication.
B How can a SART's effective range be maximized? The SART should be placed in water immediately upon activation. The SART should be held as high as possible. Switch the SART into the "high" power position. If possible, the SART should be mounted horizontally so that its signal matches that of the searching radar signal.
B In a lifeboat or liferaft, what is a method of maximizing the effectiveness of an SART? Place the SART into the sea as soon as possible to begin transmitting. Hold or mount the unit as high as possible. Extend the length of the transmitting antenna. Replace the internal battery with the AC power adapter.
C At what point does a SART begin transmitting? It immediately begins radiating when placed in the "on" position. It must be manually activated. If it has been placed in the "on" position, it will respond when it has been interrogated by a 9-GHz radar signal. If it has been placed in the "on" position, it will begin transmitting immediately upon detecting that it is in water.
C What causes the SART to begin a transmission? When activated manually, it begins radiating immediately. It is either manually or water activated before radiating. After being activated the SART responds to radar interrogation. It begins radiating only when keyed by the operator.
B How does the searching vessel's radar interrogate a survival craft SART? Activate the IFF interrogation system. The SART responds automatically when it detects the search craft or other vessels' X-Band radar signal. Maintain watch on VHF-FM Ch-70 for the SART's unique identifier. The SART responds automatically when it detects the search craft or other vessel's 3.5 GHz radar signal.
B What does a SART signal sound or look like? It transmits "SOS" and the vessel's name and position in slow speed Morse Code. It will appear on a radar unit's PPI as a line of dots radiating outward with the innermost dot indicating the SART's position. It will appear on a radar unit's PPI as a line of dots radiating outward with the outermost dot indicating the SART's position. None of the above.
A How can rescue personnel detect that a SART is transmitting in the immediate vicinity? The SART's dots on the PPI will begin arcing and eventually become concentric circles. The DSC unit will react to the SART's signal and respond with the two-tone auto alarm. The SART can provide an approximate location to within a two nautical mile radius, per IMO standards. The SART signal appears as a target which comes and goes; the effect of heavy swells on a SART.
B What signal is detected as originating from an SART? The Morse code distress series S-O-S repeated three times followed by DE and the vessel's call sign. A line of dots on a radar screen outward from the SART's position along its line of bearing. A line of dots on a radar screen inward from the SART's position to its own ship along its line of bearing. None of these.
A How can vessel personnel detect the operation of a SART in its vicinity? A unique radar signal consisting of a 12 dots radiating outward from a SART's position along its line of bearing. A unique two-tone "warbling" signal heard on VHF-FM Ch-70. A unique two-tone alarm signal heard upon the automatic un-muting of the 2182 kHz radiotelephone automatic watch receiver. The SART signal appears as a target which comes and goes; the effect of heavy swells on an SART.
C How should the signal from a Search and Rescue Radar Transponder appear on a radar display? A series of dashes. A series of spirals all originating from the range and bearing of the SART. A series of 12 equally spaced dots. A series of twenty dashes.
B In which frequency band does a search and rescue transponder operate? 3 GHz 9 GHz S-band 406 MHz
C Which of the following would most likely prevent a SART's signal from being detected? Signal absorption by the ionosphere. Heavy sea swells. The rescue personnel were monitoring the 10-CM radar. The rescue personnel were monitoring the 3-CM radar.
C Which statement is NOT true regarding the SART? Responds to interrogations by a vessel's X-Band radar. Transmits on the 9 GHz band reserved for navigational radar. Operates in conjunction with a vessel's S-Band radar. Transmits a distinctive code for easy recognition.
B Which statement is true regarding the SART required for GMDSS compliance? This is a performance monitor attached to at least one X-band navigational radar system. This is a 9 GHz transponder capable of being received by vessel's X-band navigational radar system. This is a 9 GHz transponder capable of being received by another vessel's S-band navigational radar system. This is a performance monitor attached to at least one X-band navigational radar system.
B Which statement is NOT true regarding the SART? Responds to interrogations by a vessel's X-Band radar. This is a 6 GHz transponder capable of being received by a vessel's X-band navigational radar system. This is a 9 GHz transponder capable of being received by another vessel's X-band navigational radar system. Transmits a distinctive signal for easy recognition.
C A SART's signal cannot be detected: In poor visibility or at night. In heavy seas. By a search vessel's 10 cm Radar. By a search vessel's 3 cm Radar.
B Which of the following statements concerning testing and maintenance of SARTs is true? An at-sea GMDSS maintainer is not able to test a SART as it is hermetically sealed. Testing a SART should be performed only in controlled environment as a test signal may be misinterpreted as a genuine distress situation. A SART's battery must be replaced within ninety (90) days after the expiration date imprinted on the unit. All of the above.
B Why is it important to limit the duration of testing a SART? Excessive testing causes "burn in" on the vessel's radar PPI. Testing a SART should be performed only in controlled environment as a test signal may be misinterpreted as a genuine distress situation. To prevent overheating, a SART requires sufficient ventilation that is significantly reduced when the SART is being tested. If another SART is testing at the same time, the two signals will cause damage to the unit that transmitted them.
D What statement is true regarding tests and maintenance that could be provided for the SART? To fully verify operation within manufacturer's specifications would require measuring equipment to generate 9 GHz signals; generally beyond the scope of on-board maintenance. Extreme care should be exercise because testing of the SART may be received by other vessels and may be interpreted as a distress condition or provide interfere with other vessels' safe navigation. Battery should be replaced with a new one before the manufacturer's expiration date shown on the SART. All of these.
D Why should functional testing of an SART be minimized? Potential interference with safe navigation. Minimize power consumption of the battery. Possibility of misinterpretation by other vessels as a distress situation. All of these.
A Which is not a valid maintenance and testing function for a SART? Operational test with several vessels to determine effective transmitting range. Inspection of container for apparent damage. Inspect battery expiration date and the lanyard condition. Brief operational test utilizing own ship's radar.
C The SART is required to have sufficient battery capacity to operate in the stand-by mode for what period of time? Eight hours. Three days. Four days. Forty-eight hours.
D Which is a function of a satellite under COSPAS-SARSAT using satellite EPIRBs? Vessel information recovered from the digital encoded message provided by the satellite EPIRB. Doppler shift of EPIRB signal is measured. Information received from EPIRBs are time-tagged and transmitted to any Local User Terminal in the satellite's view. All of these.
A Which of the following satellite systems is of particular importance to search and rescue missions under GMDSS? COSPAS/SARSAT. AMSAT. NASA/Arienne. COMSAT.
D Which of the following statements concerning COSPAS-SARSAT is true? EPIRBs are units that are used as alerting devices. These are satellites in a low-earth polar orbit that detect EPIRB beacons on 406 MHz and relay the information to an earth-side Local User Terminal (LUT). The Doppler frequency measurement concept is used to determine the EPIRB's location. All of the above.
D Which of the following statements concerning COSPAS-SARSAT is false? EPIRBs are used primarily for distress alerting. These satellites are looking specifically for EPIRB signals on 406 MHz. These satellites use Doppler shift measurement to determine the location of the beacons. After initiating a call request and selecting the CES, these satellites may be used for commercial messages.
C A distress signal transmitted from which EPIRB is relayed by an INMARSAT satellite __________. Class A EPIRBs. Class B EPIRBs. L-band EPIRBs on the designated frequency. 406 MHz EPIRBs broadcast to all vessels for relay to a CES.
A Which of the following statements concerning satellite EPIRBs is true? Once activated, these EPIRBs transmit a signal for use in identifying the vessel and for determining the position of the beacon. The coded signal identifies the nature of the distress situation. The coded signal only identifies the vessel's name and port of registry. If the GMDSS Radio Operator does not program the EPIRB, it will transmit default information such as the follow-on communications frequency and mode.
D What feature(s) may be found on certain satellite EPIRB units? Strobe light. Emergency transmission on 406 MHz. Float-free release bracket. All of these.
B What feature is not found on 406 MHz satellite EPIRB units? 121.5 MHz emergency homing transmitter. Aural locator signal. Emergency transmission on 406.025 MHz. Float-free release bracket.
C What statement is true regarding 406 MHz EPIRB transmissions? Allows immediate voice communications with the RCC. Coding permits the SAR authorities to know if manually or automatically activated. Transmits a unique hexadecimal identification number. Radio Operator programs his I.D. into the SART immediately prior to activation.
D Which of the following is normally found on EPIRBs that are detected by satellites? A strobe light. A 5-watt 406-MHz beacon. A bracket designed to allow the EPIRB to automatically float-free. All of the above.
C Which EPIRB transmits a distress alert that is received and relayed by an INMARSAT satellite? Class A EPIRBs. Class B EPIRBs. L-band EPIRBs. Category I EPIRBs.
B Which of the following EPIRBs is most likely to be used to transmit a distress alert signal? S-Band EPIRBs. 406 MHz EPIRBs. Class A EPIRBs. 121.5/243 MHz EPIRBs.
B Which of the following would best be used for visual detection of a distressed vessel? A 9-GHz SART's beacon. An EPIRB's strobe light. A 121.5-MHz EPIRB beacon. A 406-MHz EPIRB beacon.
B Which piece of required GMDSS equipment is the primary source of transmitting locating signals? Radio Direction Finder (RDF). An EPIRB transmitting on 406 MHz. Survival Craft Transceiver. A SART transmitting on 406 MHz.
C What may be used as a homing signal by the search and rescue vessels in the immediate vicinity of the ship in distress? Flare gun. Strobe Light. A 121.5 MHz emergency transmitter in a satellite EPIRB. 406 MHz signal from a satellite EPIRB.
B What part of a satellite EPIRB may function as a visual aid to rescue vessels? A 121.5 MHz emergency transmitter in a satellite EPIRB. Strobe light. 406 MHz signal from a satellite EPIRB. Loud beeping tone emitted by the unit once activated.
D What is an example of a locating signal? SSB phone traffic. Ship to shore transmissions. Loran C. A Float-Free EPIRB.
B Which device provides the main means in the GMDSS for locating ships in distress or their survival craft? Radio Direction Finder. Satellite EPIRBs. MF/HF DSC. VHF homing device.
D With what other stations may portable survival craft transceivers communicate? Communications is permitted between survival craft. Communications is permitted between survival craft and ship. Communications is permitted between survival craft and rescue unit. All of the above.
A Equipment for radiotelephony use in survival craft stations under GMDSS must have what capability? Operation on Ch-16. Operation on 457.525 MHz. Operation on 121.5 MHz. Any one of these.
D Equipment for radiotelephony use in survival craft stations under GMDSS must have what characteristic(s)? Operation on Ch-16. Watertight. Permanently-affixed antenna. All of these.
B Which statement is NOT true regarding the requirements of survival craft portable two-way VHF radiotelephone equipment? Operation on Ch-16. Effective radiated power should be a minimum of 2.0 Watts. Simplex (single frequency) voice communications only. All of these.
A Which statement is NOT true regarding the requirements of survival craft portable two-way VHF radiotelephone equipment? Operation on Ch-13. Effective radiated power should be a minimum of 0.25 Watts. Simplex (single frequency) voice communications only. Operation on Ch-16.
A Which statement is NOT true regarding the requirements of survival craft portable two-way VHF radiotelephone equipment? Operates simplex on Ch-70 and at least one other channel. Watertight to a depth of 1 meter for 5 minutes. Effective radiated power should be a minimum of 0.25 Watts. All of these.
A Which action should the GMDSS radio operator take in a distress situation when embarking in survival craft? Switch on EPIRB and SART immediately and leave on. EPIRB and SART switched on manually prior to embarking; remain aboard vessel in distress. Notify RCC (Rescue Coordination Center) through VHF DSC in portable equipment. Communicate via Inmarsat-C from the survival craft.
A Which is the key part of the search and rescue system under GMDSS? COSPAS/SARSAT satellites. AMSAT satellites. NASA satellites. US Space Agency satellites.
D Which statement is true regarding the COSPAS-SARSAT system? EPIRBs are used as satellite beacons aboard vessels as alerting devices. Signals received by low altitude near-polar orbiting satellites are relayed to a ground receiving station, called a Local User Terminal. Doppler shift is used to locate the beacons. All of these.
D Which statement is NOT true regarding the COSPAS-SARSAT system? EPIRBs are used as satellite beacons aboard lifeboats as alerting devices. Locates distress beacons transmitting on 406 MHz. Doppler shift is used to locate the beacons. May be used to transmit public correspondence.
B What information is transmitted by a 406 MHz EPIRB alert? Vessel position and nature of distress. A unique Hexadecimal I.D. number. Vessel name and identification. None of the above.
D Which statement is true regarding the COSPAS-SARSAT system and EPIRB operations? The EPIRB's position is calculated by the system and passed to the RCC. The EPIRB transmits a unique Hex I.D. and vessel position that is passed to the RCC. The EPIRB transmits a unique Hex I.D. that is passed to the RCC. Both a) and c) are true.
D Which of the following has been designated for “On-scene” communications in GMDSS? Ch-24 Ch-2182 Ch-70 Ch-16 on VHF radiotelephone and 2174.5 kHz using MF SITOR.
C Which of the following channels is designated as the VHF follow-on communications channel and is required in all portable survival craft equipment? Ch-6 Ch-13 Ch-16 Ch-70
D Which of the following frequencies have been designated for “On-scene” communications in the Global Maritime Distress and Safety System? VHF Ch-22. HF radiotelephone on 21.820 MHz. NBDP on 2177.0 kHz and VHF Ch-16. VHF Ch-16 and NBDP on 2174.5 kHz.
C Which of the following frequencies have NOT been designated for “On-scene” communications in the Global Maritime Distress and Safety System? VHF Ch-16. MF radiotelephony on 2182 kHz. NBDP on 2182.0 kHz. None of these.
A “On-scene" communications would best be represented by? NBDP on 2174.5 Sending DSC alert on VHF Ch-70. Using Inmarsat-C "hot-key" function. None of the above.
B For “On-scene” communications, vessels in distress and SAR Aircraft should use? VHF Ch-70, 4125 kHz J3E, 5680 kHz J3E VHF Ch-16, 4125 kHz J3E, 3023 kHz J3E VHF Ch-16, 4125 kHz F1B, 3023 kHz J3E None of the above.
C Which of the following control selections may result in limited receiving range? Setting the squelch control to its minimum level. The power switch is set to the "high" output position resulting in receiver overloading. Setting the squelch control to its maximum level. Setting the channel selection switch midway between channels 6 and 16.
A At mid-day, what would be the best choice in attempting to communicate with a shore station 15 miles (24 km) distant? VHF-FM. 16 MHz band. 12 MHz band. 22 MHz band.
C Which factors normally determine the range of VHF transmissions: Channel frequency. Power level. Both b) and d). Vessel antenna height.
B Causes of much longer than normal VHF transmissions are: Changing power from 1W to 25 W. Atmospheric ducting. Ionospheric activity in layers F1/F2. None of the above.
D Describing VHF transmissions as "line of sight" means: VHF communications are effective only with nearby stations within visual range of the bridge. Vessel antenna height will affect the radius of propagation. The normal transmission range to a coast station is approximately 25 NM. Both b and c are true.
D The effectiveness of VHF communications is maximized by: The adjustment of squelch for maximum receiver sensitivity. Appropriate setting of the transmitter power. Selecting an appropriate channel. All of the above.
D A VHF frequency channel pair of TX 157.200 MHz and RX 161.800 MHz would most likely be: A Public Correspondence Coast Radio Station frequency. Simplex. Duplex. Both A and C are correct.
C Which channel is utilized for the required bridge-to-bridge watch? DSC on Ch-70. VHF-FM on Ch-16. VHF-FM on Ch-13 in most areas of the continental United States. The vessel's VHF working frequency.
B While conducting routine communications using the wheelhouse VHF with a station 1 mile distant, your recommended power setting would be: 25 watts after dark. 1 watt, day or night. 25 watts during a clear sunny day. 1 watt using DSC at night.
B The USA-INT control on VHF units: Selects Duplex operations for U.S. coastal waters and Simplex operations in non-U.S. waters on the "alpha" channels. Changes selected international duplex channels to simplex channels for use in U.S. waters. Both of the scenarios above may be set up and selected by the operator. None of the above.
D The USA-INT control on VHF units: Was made necessary by a desire for more simplex channels in the U.S. Correctly set, will result in Duplex operations in U.S. Coastal waters on the "alpha" channels. Correctly set, will result in Simplex operations in U.S. Coastal waters on the "alpha" channels. Both a) and c) are true.
D Proper and legal VHF operations require: The channel must be designated as valid for the nature or type of communications desired. The correct bandwidth must be selected by the operator. The power level must be appropriately chosen. Both answers a) and c) are correct.
D How is mutual interference among NAVTEX stations avoided? Stations are limited to daytime operation only. Transmitter power is limited to that necessary for coverage of assigned area. Transmissions by stations in each NAVAREA are arranged in a time-sharing basis. Both b) and c).
B When do NAVTEX broadcasts typically achieve maximum transmitting range? Local noontime. Middle of the night. Sunset. Post sunrise.
A What should a GMDSS Radio Operator do if a NAVTEX warning message is received but it contains too many errors to be usable? Do nothing. Vital NAVTEX messages will be repeated on the next scheduled broadcast. Contact the NAVAREA coordinator and request a repeat broadcast. The hurricane will be upon the vessel; they're in big trouble. Listen to appropriate VHF weather channel for repeat warnings.
B What does a NAVTEX receiver do when it runs out of paper? The unit cannot operate, and all subsequent MSI broadcasts are missed until the paper is replaced. It will give off either an audible and/or visual alarm. The system will automatically change from receiving MSI by NAVTEX to receiving it by SafetyNET so that no messages will be lost. All of the above.
A Which of the following is the primary frequency that is used exclusively for NAVTEX broadcasts internationally? 518 kHz 2187.5 kHz 4209.5 kHz VHF channel 16 when the vessel is sailing in Sea Area A1, and 2187.5 kHz when in Sea Area A2.
D What is the transmitting range of most NAVTEX stations? Typically 50-100 nautical miles (90-180 km) from shore. Typically upwards of 1000 nautical miles (1800 km) during the daytime. It is limited to line-of-sight or about 30 nautical miles (54 km). Typically 200-400 nautical miles (360-720 km).
B How is a NAVTEX receiver programmed to reject certain messages? The transmitting station's two-digit identification can be entered to de-select reception of its broadcasts. By choosing a message category's single letter (A-Z) identifier and then deselecting or deactivating. By entering the selcall of the transmitting station. By pressing "00" in the transmitter's ID block.
B How can reception of certain NAVTEX broadcasts be prevented? Stations are limited to daytime operation only. The receiver can be programmed to reject certain stations and message categories. Coordinating reception with published broadcast schedules. Automatic receiver desensitization during night hours.
B Which of the following statements is true? The GMDSS Radio Operator can program the NAVTEX receiver to automatically reject any category of messages under the master's authority. The GMDSS Radio Operator can program the NAVTEX receiver to reject all messages except navigation warnings, meteorological warnings, and search and rescue information. The GMDSS Radio Operator can select the "None" option in the message category menu. Upon entering a new NAVTEX station's broadcast range, the GMDSS Radio Operators enters the station's selcall number.
B What means are used to prevent the reception of unwanted broadcasts by vessels utilizing the NAVTEX system? Operating the receiver only during daytime hours. Programming the receiver to reject unwanted broadcasts. Coordinating reception with published broadcast schedules. Automatic receiver de-sensitization during night hours.
B What statement "is true" regarding the control the operator can exercise over the NAVTEX receiver's operation? The operator can set the unit to automatically reject any and all categories of messages if the ship desires to not receive them. The operator can set the unit to reject all messages except navigation, weather and sea warnings, and search and rescue messages. To reduce the number of messages, the operator can select code 00 to indicate "not in coastal passage". Upon entering a coastal area for the first time, the operator enters code KK to indicate "ready to receive NAVTEX".
C Which message subject matter can be programmed to be rejected or disabled by the operator of a NAVTEX receiver? Navigational warnings. Meteorological warnings. Pilot Service Messages. All of these.
A The NAVTEX message header contains the following? A single letter (A-Z) indicates the NAVTEX transmitting station. A two-digit number (01-99) indicates the NAVTEX message category. Message numbers include a date/time group along with the transmitting station's numerical ID. None of these.
D Which of the following message categories cannot be disabled by the GMDSS Radio Operator? Navigational warnings. Meteorological warnings. Search and Rescue information. All of the above.
A How are NAVTEX broadcasts transmitted? Using FEC techniques. NAVTEX is transmitted by commercial coast radio stations following their traffic lists. NAVTEX is transmitted only when an urgency or distress broadcast is warranted. No more often than every two hours and should immediately follow the radiotelephone silent periods.
D Which determines whether a NAVTEX receiver does not print a particular type of message content? The serial number and type of message have already been received. The subject indicator matches that programmed for rejection by the operator. The transmitting station ID covering your area has not been programmed for rejection by the operator. Both answers a) and b).
B Which information determines if a NAVTEX message is to be rejected? Transmitter identity (numerals from 1 to 26 identifying transmitting station within the NAVARE. Subject indicator (single letter from A to Z indicating the type of message). The Answerback of the receiving station has not been entered in the NAVTEX receiver. Only messages having a serial number 00 are rejected.
A NAVTEX broadcasts are sent: In categories of messages indicated by a single letter or identifier. Immediately following traffic lists. On request of maritime mobile stations. Regularly after the radiotelephone silent periods.
A Where NAVTEX cannot be feasibly established, what system can be implemented to provide an automated service in coastal waters to receive MSI? SafetyNET. AMVER. VHF DSC. ARQ SITOR.
B What action should a GMDSS Radio Operator take when SafetyNET™ distress or urgency messages are received by the vessel's EGC receiver? No immediate action is required as an audible tone will be generated at the beginning and end of the transmission and a paper printout of the message will be generated. Aural and visual alarms are activated, and require manual deactivation. No immediate action is required by the operator since the transmission will be automatically acknowledged by the receiving vessel. A periodic alarm tone will be heard until the radio operator prints the message from the unit's memory.
A What system can provide an automated service in coastal waters where it may not be feasible to establish the NAVTEX service or where shipping density is too low to warrant its implementation? SafetyNET. AMVER. VHF DSC. ARQ SITOR.
C Aboard ship, SafetyNET™ messages can be received by which equipment? VHF DSC. NAVTEX Receiver. Dedicated receiver or optional receiver integrated in vessel's SES. All of these.
C SafetyNET™ messages can be received by which of the following shipboard equipment? NAVTEX. MF and HF NBDP. EGC receiver. All of these.
B Marine Safety Information is promulgated via satellite through which system? AMVER. SafetyNET. NAVTEX. Inmarsat-M SES.
A SafetyNET™ promulgates what type of information? MSI. Traffic Lists. News advisories. MARAD.
B What kind(s) of broadcasts are not available through SafetyNET™? MSI and messages to specific geographic areas. Vessel traffic lists. Storm warnings. Distress and urgent bulletins.
B Which satellite system promulgates Maritime Safety Information? AMVER. Inmarsat-C SafetyNET. NAVTEX. Inmarsat-M SES.
A What information is promulgated by the international SafetyNET™? MSI. Traffic Lists. Priority Messages. MARAD.
B A vessel using SafetyNet should: Notify the NAVAREA coordinator you are using SafetyNet for MSI (Maritime safety information). Set the receiver to your present NAVAREA. Set the receiver to your destination Ocean Region. Notify the NAVAREA coordinator you are using SafetyNet for MSI (Maritime safety information) and set the receiver to your destination Ocean Region.
D In using SafetyNET™ for MSI (Maritime safety information): If you fail to log-in with your Ocean Region you will receive only unscheduled urgent and distress broadcasts. To receive scheduled and unscheduled broadcasts you must log-in with your Ocean Region Net Control Station (NCS). Your satellite receiver must have Enhanced Group Calling (EG capability. All of these.
D Over what system are Enhanced Group Calls transmitted? By COSPAS satellite. By HF SITOR shore stations. By NAVTEX shore stations. By Inmarsat satellite.
D How are MSI (Marine safety information) broadcasts received in an EGC receiver integrated with existing Inmarsat equipment when the SES is otherwise engaged in communications? The broadcast message is missed and the Radio Operator must request a retransmission. The broadcast message is stored in the EGC memory and will automatically be printed at the conclusion of the ongoing traffic. The radio operator can request retransmission of messages missing from numeric serial number succession. There is no loss of information since broadcasts of "vital" messages will be repeated.
B Which of the following provides a unique automated system capable of addressing messages to pre-determined groups of ships or all vessels in both fixed and variable geographic areas? NAVTEX. EGC. AFRTS. NAVAREAs.
B What system may be useful for messages, such as local storm warnings or a shore-to-ship distress alert, for which it is inappropriate to alert all ships in the satellite coverage area? NAVTEX. EGC. AMVER. DSC.
A What services are available through Enhanced Group Calls? Marine Safety Information and messages to pre-defined groups of subscribers. Marine Safety Information and vessel traffic lists. Hourly NOAA weather broadcasts from the NWS. Coastal weather broadcasts.
A What messages originate from registered information providers anywhere in the world and are broadcast to the appropriate ocean region via a CES? SafetyNET messages. AMVER broadcasts. Urgency messages. NAVTEX broadcasts.
B Which HF SITOR mode would be selected to receive MSI broadcasts from high seas shore stations: AM FEC RTTY ARQ
B The U.S. Coast Guard communications station providing HF MSI broadcast coverage for NAVAREA IV is: NOJ (Kodiak). NMF (Boston). NMC (San Francisco). NMO (Honolulu).
C The U.S. Coast Guard communications station providing HF MSI (Maritime safety information) broadcast coverage for NAVAREA XII is: NMA (Miami). NMF (Boston). NMO (Honolulu). NMR (San Juan).
D Frequencies for receiving HF MSI (Maritime safety information) are: The same as used for NAVTEX. The same as used for general telex using FEC. Specified HF voice frequencies. Specified HF NBDP frequencies.
B Which frequency/mode is authorized for use internationally for Marine Safety Information transmissions? 4125.0 kHz using simplex mode. 4209.5 kHz using FEC mode. 4209.5 kHz using ARQ mode. 4125.0 kHz using FEC mode.
C How many frequencies are assigned specifically for H.F. MSI broadcasts? 6 5 8 7
A Which NAVAREA is associated with the western North Atlantic and the Caribbean Sea? NAVAREA IV. NAVAREA X. NAVAREA XI. NAVAREA XII.
D Which NAVAREA is associated with the Pacific Ocean north of the equator and east of the International Date Line? NAVAREA IV. NAVAREA X. NAVAREA XI. NAVAREA XII.
A NAVAREAs referred to in NAVTEX are the same as used in __________. INMARSAT SafetyNET. GMDSS sea areas. International Vessel Traffic Service. INMARSAT ocean regions.
C A vessel operating in the Western Atlantic or along the East coast of North America and Central America from Canada to Venezuela, including the Caribbean and Panama, would be located in which NAVAREA? X XI IV XIII
C A vessel operating in the Eastern Pacific or along the West coast of North and Central America from Alaska to Ecuador, including Panama and Hawaii, would be operating in which NAVAREA? X XI XII IV
B A vessel on a voyage between Miami and Los Angeles via the Panama Canal would be operating in which NAVAREA(s)? II and III. IV and XII. IV and V. V and VI.
C Which of the following actions should be taken once the vessel is berthed and will not leave port again for several weeks? The GMDSS Radio Operator must notify the NCS that the vessel will be off-line, and wait for the NCS to acknowledge with a confirmation number that must be logged. The Inmarsat-C system can be powered down without taking additional steps once the GMDSS Radio Operator has ensured that all incoming SafetyNET messages have been received and stored. The GMDSS Radio Operator must log out of the Inmarsat-C system. The GMDSS Radio Operator must transmit an all-ships alert to notify all vessels within the satellite's footprint that the vessel will be off-line.
D What action should be taken on arrival at every port? An Inmarsat-C system must be powered down. Send a message to the NCS advising arrival in port. Both of the above. None of the above.
B With most Inmarsat-C systems what should the indicator lamps do when powering up? The power on lamp should light and the others stay off until a message is received. All lamps should illuminate in a particular sequence per the operator’s manual. All lamps should light and stay on. All lamps should light except the RED light.
D Upon power-up, what controls are adjusted on an Inmarsat-C terminal? The antenna Azimuth and Elevation controls. The receiver gain is adjusted for maximum signal. Both of the above. None of the above.
A On an Inmarsat-C system soon after power up what might a blinking lamp indicate? The system is not yet locked on to the NCS signal. An EGC message is being received. There is mail being received. None of the above.
B On an Inmarsat-C system an alarm sounds: When first powered on. When receiving distress traffic. Both of the above. None of the above.
A Which satellite(s) would most likely be selected for use when the vessel is operating off the eastern shore of the United States? AOR-W. IOR-E. POR. Either AOR-W or IOR-E will work.
A Which satellite would be chosen when operating in the Eastern Gulf of Mexico? AOR-W. IOR. POR. Any one of these.
D Which longitude corresponds to the AOR-W satellite for Inmarsat communications? 64.5E. 178E. 15.5W. 54W.
C Which longitude corresponds to the AOR-E satellite for Inmarsat communications? 64.5E. 178E. 15.5W. 54W.
B Which longitude corresponds to the POR's satellite location for Inmarsat communications? 64.5E. 178E. 15.5W. 54W.
A Which longitude corresponds to the IOR's satellite location for Inmarsat communications? 64.5E. 178E. 15.5W. 54W.
C Which action must be taken to ensure that incoming message traffic of all priority levels will be received through Inmarsat-C? The system needs only to be commissioned and turned on. No additional action is necessary after turning on the receiver and aiming the antenna at the desired satellite. The GMDSS Radio Operator must log-in to the desired satellite. The GMDSS Radio Operator must log on to the desired satellite and receive the message reference number (MRN) from the CES.
C When logging into the Inmarsat system using Inmarsat-C, it is necessary to: Enter your IMN. Enter the CES answer back. Select the Ocean Region. Call the CES and inform them that you are now operating in the appropriate ocean region.
C What action should be taken on changing from one ocean region to another? Power the system down and turn the power back on again. Manually realign the antenna. Log out of the current satellite and log in to the correct satellite. Both A and C are correct.
A The process of logging out involves the following: Selecting the proper command from the correct menu. Obtaining confirmation of log out from the NCS. Both of the above. None of the above.
D How do you determine that your Inmarsat-C terminal has accomplished a successful login? The red panel lamp lights. The green synch lamp starts flashing. The green synch lamp turns on steady. None of the above.
C On many Inmarsat-C terminals, which of the following indicates a successful log-in? A message is displayed on the screen indicating a successful log-in. The printer may also print out a notice of a successful log-in. Both of the above. None of the above.
A What is the primary function of an NCS? To monitor and control communications through the Inmarsat satellite for which it is responsible. To provide direct communications between the Inmarsat station placing a call and the station receiving the call. To provide multi-mode communications between the Inmarsat station placing a call and the coast radio station that will deliver it. To determine which satellite is best suited to provide communications between the Inmarsat station placing a call and the station receiving the call.
B What is the primary function of a CES? To monitor and control communications through the Inmarsat satellite for which it is responsible. To provide direct communications between the Inmarsat station placing a call and the station receiving the call. To provide multi-mode communications between the Inmarsat station placing a call and the coast radio station that will deliver it. To determine which satellite is best suited to provide communications between the Inmarsat station placing a call and the station receiving the call.
C Messages are transmitted by the CES according to what criteria? First In, First Out. Last In, First Out. Priority, e.g.: Distress, Urgency, Safety and Routine. Serial Number.
B How is maximum coverage provided by satellites in the maritime satellite service? Four satellites in polar orbit. Four satellites in geo-stationary orbit approximately 22,184 miles above the equator. Four satellites in geo-stationary orbit for each Inmarsat Service (A, B, C and M). Through coordinated use of COSPAS-SARSAT satellites.
B What is meant by the characters "GA+" on an Inmarsat terminal? General Address (to all stations). Go ahead. The instruction to "give address". None of these.
B What is the purpose of compandors? Noise and echo-canceling used in telex operation. Noise and echo canceling in voice operation. To enable distress communications. To enable data communications.
B What is an MRN? A Mobile Registration Number, provided by the FCC. A Message Reference Number, provided by the CES. A Mobile Registration Number, provided by IMO. The vessel's call sign.
D To keep the Inmarsat-A/B antenna pointing at the desired satellite, regardless of the ship's position and course, it has an input from the vessel's: Operational radar. Automated Radar Plotting Aid (ARPA) equipped radar. Steering control system. Gyrocompass.
B The Inmarsat telephone and telex communications channel usage scheme is: Many ships on the same telex analog channel frequency and many ships on the same TDM telephone channel. One ship per telephone channel and many ships per telex channel. One ship per analog telephone channel and one ship per telex analog channel frequency. One ship per channel whether telephone or telex.
B What is an Inmarsat "Subscriber Number"? This identifies the vessel's selective calling (selcall) number. This is the Inmarsat number that is assigned to a unit for incoming calls. This is the vessel's Inmarsat registration number for accounting authority purposes. This number is used for receiving news and other optional services in FleetNET.
C Which of the following statements concerning Inmarsat geostationary satellites is true? They are in a low-earth polar orbit to provide true global coverage. They are in an equatorial orbit to provide true global coverage. They provide coverage to vessels in nearly all of the world's navigable waters. Vessels sailing in equatorial waters are able to use only one satellite whereas other vessels are able to choose between at least two satellites.
B What is meant by "CES"? Coast Earth Satellite. Coast Earth Station. Central Equatorial Station. Coastal Equivalent Station.
A How is a signal radiated from an Inmarsat-B system's antenna? It is a highly focused directional signal that must be beamed at the desired satellite. It is usually radiated in an omni-directional pattern, but an optional feature allows it to be directional for use when the vessel is on the fringe of the satellite's footprint. It is radiated in an omni-directional pattern. It is radiated in an omni-directional pattern that can be reversed by the Operator to attain directional beaming to an alternate satellite.
B Which mode of Inmarsat-B communications may be possible with a lower received signal strength? Fax TELEX Voice Communications Binary computer file transfers
A What is the purpose of the second I.D. in an Inmarsat-B SES? To provide an additional number which may be dedicated to computers, fax, etc. To provide an alternate number which may be called if a busy signal is received by the calling party. To provide an additional speech path, which may be used to communicate while the first channel is engaged in active communications. To provide for an emergency working frequency.
C What is the effect of having five periods (e.g.-meanwhile.....) in the text of a TELEX transmission on an Inmarsat-A SES? Only the first period will be routed to the receiving party. Only the first two periods will be routed to the receiving party. The transmission will automatically terminate after those characters are transmitted. This will automatically trigger the reversal of charges to the receiving party.
A Why is the automatic answerback request (WRU) first used by the CES after an Inmarsat-B TELEX call has been placed from a ship? Identify the SES making the request. Let the CES operator know the printer is functioning correctly. Get the identity of the ship station and start the channel assignment process. Verify that there is a good connection with no transmission errors.
B When engaging in voice communications via an Inmarsat-B terminal, what procedures are used? Noise-blanking must be selected by the operator. CODECs are used to digitize the voice signal. The voice signal must be compressed to fit into the allowed bandwidth. The voice signal will be expanded at the receiving terminal.
D Which mode of communications is NOT possible through an Inmarsat-C SES? Data. Telex. Emergency Activation. Shore-to-ship Facsimile.
B What is the average length of time required for a telex sent by Inmarsat-C to be delivered to the addressee? All Inmarsat-C communications are made with real-time connectivity so there is no delay in message delivery. The average delivery time for a message sent by Inmarsat-C is about 10 minutes. Date/time notification of delivery is possible only through Inmarsat-A. The average delivery time for a telex sent by Inmarsat-C is about 10 minutes, but fax and data messages sent by Inmarsat-C require about 30 minutes for delivery.
C How is a signal radiated from an Inmarsat-C system's antenna? It is a highly focused directional signal that must be beamed at the desired satellite. It is usually radiated in an omni-directional pattern, but an optional feature allows it to be directional for use when the vessel is on the fringe of the satellite's footprint. It is radiated in an omni-directional pattern. It is radiated in an omni-directional pattern that can be reversed by the Operator to attain directional beaming to an alternate satellite.
C What statement is true regarding Inmarsat-C? There is a propagation delay, but a direct connection is made between the ship and shore users. There are delays in establishing the communications then a direct real-time connection is maintained with the other party. This is a store and forward network with an intermediate step that means there is no direct connection between ship and shore users. The telex message is stored until the mailbox is accessed by the station desiring to retrieve their message.
B With an Inmarsat-C CES, how are messages routed to receiving stations? Direct connections are made to the receiving stations via gateways. All messages are forwarded via a store and forward network. Intermediary stations are used to connect the sending station with the receiving station in a real-time mode. Messages are stored until the network is polled by the receiving station.
B What are the directional characteristics of the Inmarsat-C SES antenna? Highly directional parabolic antenna requiring stabilization. Omni-directional. Wide beam width in a cardioid pattern off the front of the antenna. Very narrow beam width straight-up from the top of the antenna.
D Which of the following best describes Inmarsat-C operation? Is an analog-based system. Requires a stabilized directional antenna. Provides for voice, telex, high- and low-speed data and compressed video communications. Is a digital store-and-forward system that also provides Enhanced Group Call, data reporting, polling and distress alerting capabilities.
B Which of the following best describes a shipboard Inmarsat-C system? A satellite communications system that provides real-time connectivity. A small, lightweight terminal capable of providing satellite store-and-forward message communications. A small, lightweight terminal used to transmit messages over high frequency (HF) bands to communicate through a satellite. A satellite communications system that also provides continuous Digital Selective Calling coverage for all ocean regions.
A Which of the following modes of communications are available when using Inmarsat-C? Telex. Fax. 14400 BPS Data. Voice.
B Which mode of communications is possible through an Inmarsat-C SES? SITOR. Telex. Radiotelephone. DSC
C It is possible to transmit all of the following via Inmarsat-C from a vessel except? Telex. Text for delivery by Fax. Voice. Comtex mail and x.400 data services.
D Inmarsat-C provides for which of the following? Polling, enhanced group call, and one-way position and data reporting via satellite. FM voice communications via satellite. Two-way messaging and data communications on a store-and-forward basis. Polling, enhanced group call, one-way position and data reporting via satellite, two-way messaging and data communications on a store-and-forward basis.
B Which statement concerning Inmarsat-B and Inmarsat-C terminals is correct? Both Inmarsat-B and Inmarsat-C units are capable of fax and voice communications. Both Inmarsat-B and Inmarsat-C units can send data as well as send messages to fax machines. Inmarsat-B units are not capable of data communications, but Inmarsat-C units are capable of data communications. None of the above
D When Inmarsat-B and Inmarsat-C terminals are compared: Inmarsat-B antennas are bulkier, but omni-directional, while Inmarsat-C antennas are smaller and parabolic, for aiming at the satellite. Inmarsat-B antennas are parabolic and smaller for higher gain, while Inmarsat-C antennas are larger but omni-directional. Inmarsat-C antennas are smaller, but omni-directional, while Inmarsat-B antennas are parabolic for lower gain. None of the above
C Which statement concerning Inmarsat-B and Inmarsat-C terminals is correct? Inmarsat-B terminals require gyro and GPS input, in order to enable automatic satellite tracking. Inmarsat-C terminals require only GPS input, in order to enable automatic satellite tracking. Inmarsat-B terminals require AZ/EL setup and gyro input, in order to enable automatic satellite tracking. Inmarsat-C terminals require AZ/EL setup and GPS input, in order to enable automatic satellite tracking.
C When Inmarsat-B and Inmarsat-C terminals are compared: Inmarsat-C antennas are smaller, with active parabolic antennas but no rewind capability. Inmarsat-B antennas are larger, with passive non-parabolic antennas that require rewind capability. Inmarsat-C antennas are smaller, with passive non-parabolic antennas but no rewind capability. Inmarsat-B antennas are larger, with stationary parabolic antennas but no rewind capability.
B Which statement concerning Inmarsat-B and Inmarsat-C terminals is correct? Both Inmarsat-B and Inmarsat-C units are subject to shadowing effects due to their omni-directional antennas. Both Inmarsat-B and Inmarsat-C units are subject to shadowing effects, but Inmarsat-B units have directional antennas. Both Inmarsat-B and Inmarsat-C units are subject to shadowing effects, but Inmarsat-C units have directional antennas. Both Inmarsat-B and Inmarsat-C units are subject to shadowing effects, due to their directional antennas.
C When Inmarsat-B and Inmarsat-C terminals are compared: Inmarsat-B units provide greater communications capabilities, with the benefits of greater size, weight, installation expense and initial cost. Inmarsat-C provides lesser communications capabilities, with the trade-offs of greater size, weight, installation expense and initial cost. Inmarsat-B units provide greater communications capabilities, with the trade-offs of greater size, weight, installation expense and initial cost. Inmarsat-C units are of smaller size/weight, installation expense and initial cost and provide greater communications capabilities due to modern technology.
C Which CES should a GMDSS Radio Operator select if his/her vessel is off the Pacific Coast of the United States? Goonhilly. Anatolia. Niles Canyon or "Santa Paula". RCC Alameda.
A Which CES should a GMDSS Radio Operator select if his/her vessel is off the Atlantic Coast of the United States? Southbury or Staten Island. Santa Paula. RCC New York. Anatolia.
B What is the C.E.S. ID Code for the Southbury Earth Station for a vessel operating in the Western Atlantic? 0 1 10 11
D What is the C.E.S. ID Code for the Santa Paula Earth Station for a vessel operating off the Pacific Coast of the U.S.? 0 11 10 1
C Which Earth Station would a vessel be utilizing if operating off the Pacific Coast of the United States? Goonhilly. Southbury. Santa Paula. KPH (San Francisco, CA).
A Which Earth Station would a vessel be utilizing if operating off the Atlantic Coast of the United States? Southbury. Santa Paula. WCC (Chatham, MA). Odessa.
C A vessel is operating in the Western Atlantic. To initiate a ship-to-shore telephone contact to a shoreside party in the U.S.A, number 123-456-7890, through the Southbury Earth Station, using automatic service, a valid Inmarsat dialing sequence is? 118# 11234567890 #002 11234567890# 001# 0011234567890# 123 4567890
B A vessel is operating in the Eastern Atlantic. To initiate a ship-to-shore telephone contact to a shoreside party in the U.S.A., number 202-456-7890, through the Goonhilly Earth Station, using automatic service, a valid Inmarsat dialing sequence would be? 001# 12024567890 002# 0012024567890# 001# 0012024567890# 202 4567890
D Which key would be used to indicate the end of a manually dialed number in a telephone, facsimile or data call via an Inmarsat-A SES? * ENTER + #
B The U.S. Country Code for voice transmission is: 11 1 001 for Southbury, 201 Santa Paula. 581 for AOR-E, 582 for POR and 584 for AOR-W.
A If you are south of the Hawaiian Islands and wish to communicate by voice with another ship 200 miles due west of Lands End, UK. What is the proper procedure? 001# 00871336670492# 004# 11582430315036# 001# 00851636824323# 003+ 00581430326430+
C Which of the following is a correct dialing sequence for a vessel in the western Atlantic to send a FAX to a U.S. destination using an Inmarsat-B terminal? 002# 110313 001+ 110243674932+ 001# 0015036943254# 001# 15036684376#
C Which key is used to indicate the end of a selection in a TELEX call via an Inmarsat-B SES? * ENTER + #
C Which number, if typed at an Inmarsat-B SES by a vessel in the Western Atlantic, would result in placing an automatic TELEX call to a subscriber number of 123456? 00230123456# 0012131234567# 00230123456+ 00 1 213 1234567+
B Which of the following numbers is appropriate for an Inmarsat-B terminal placing a TELEX call via Southbury to another ship logged-in to the AOR-E? 001+ 00584436824246+ 001+ 00581436824246+ 002+ 00584436824246+ 013+ 00581436824246+
C You are on a ship in the Pacific ocean. What keyboard entry would you make on an Inmarsat-B terminal to obtain a TELEX connection via Santa Paula to another ship that is 300 miles west of San Francisco? 001# 00582430353680# 003+ 00582430353680+ 001+ 00582336657450+ 002+ 00582336850450+
C What keyboard entry would you make on an Inmarsat-B terminal for an automatic TELEX connection to 882419 in Nebraska, USA (TELEX country code 230)? 11230882419+ 00230882419# 00230882419+ 230882419
A Which of the following is a correctly formatted Inmarsat-B address for sending communications to a shoreside TELEX terminal number 440122 in the United Kingdom (TELEX country code 51)? 51440122+ 51440122 (51)440122+ 51440122#
D What is the Inmarsat-A Service Code for automatic service? AA 11 1 " 00 "
D What dial sequence is used, after entering the shore ID, to obtain the intervention of the telephone operator in the Inmarsat-A service? 00# 01# 10# 11#
D What must be entered, after the Shore I.D. and "#" sign is entered and a ring-back tone is heard, to obtain the intervention of the telephone operator in the Inmarsat-A service? 00# 01# 10# 11#
A A service code for the international telephone information operator? 12# 11+ 13# 36#
C How would a Radio Operator of a vessel off the California Coast request Operator Assistance from the shore station via telex? Dial 11# on the keypad after receiving the "bong" tone. Type 00+ after receiving the "GA" by the shore station. Type 11+ after receiving the "GA" by the shore station. Type 001 after receiving the "GA" by the shore station.
A What dial sequence is used, after entering the shore ID, to place a call that will be billed to a credit card? 36# The digits 00 followed by the country code, the credit card number, the expiration date and ending with the "#" sign. Credit card billing is impossible via Inmarsat. The Shore I.D. and "#" sign followed by the digits 00, then the Country Code, subscriber's number and ending with the "#" sign.
D From an Inmarsat-C terminal which of the following is a correctly formatted address for sending telex messages to a vessel in AOR-W? a) 5841502773

b) 8741502773
c) 584436671929
d) Either a) or c) are correct, depending on the Inmarsat system addressed.
C Which of the following is a correctly formatted Inmarsat-C address book entry for sending telex communications to a vessel in the AOR-E? a) 871436772983
b) 571436772983

c) 5811509952
d) 5811509952+
A Which of the following is a correctly formatted Inmarsat-C address book entry for sending telex communications to a vessel in the POR? a) 582436559121
b) 872436559121
c) 582436559121+
d) 5821508862+
D Which of the following is a correctly formatted Inmarsat-C address book entry for sending telex communications to a vessel in the IOR? a) 853446323868
b) 873446976519
c) 582446323868
d) 583446976519
B Which of the following is a correctly formatted Inmarsat-C address book entry for sending telex communications to a vessel in AOR-W? a) 8541502927
b) 5841502927
c) 8741502927
d) 5841502927+
D Which of the following is a correctly formatted Inmarsat-C address book entry for sending telex communications to a vessel in the POR? a) 582377125619
b) 5821506692
c) 582436377125
d) All may be correct, depending on the Inmarsat system addressed.
D Which of the following is a correctly formatted Inmarsat-C address book entry for sending communications to a shoreside telex terminal number 45992 in Taiwan (telex country code 769)? a) (769)45992
b) 76945992+

c) 769 45992+
d) None of the above
B Which of the following is a correctly formatted Inmarsat-C address book entry for sending communications to a shoreside telex terminal number 440122 in the United Kingdom (telex country code 51)? a) 51440122+

b) 51440122
c) (51)440122
d) 51440122#
A If your vessel is in the POR, which of the following is a correctly formatted Inmarsat-C address book entry for sending communications to a shoreside telex terminal number 42267 in Ecuador (telex country code 308)? a) 30842267

b) 58230842267
c) 30842267+
d) (582)30842267
A Which of the following is a correctly formatted Inmarsat-C address book entry for sending communications to a shoreside terminal number 276992 in New Jersey via TRT (telex country code 238)? a) 238276992 b) (238)276992
c) 238276992#
d) 238 276992+
C If your vessel is in the IOR, which of the following is a correctly formatted Inmarsat-C address book entry for sending communications to a shoreside telex terminal number 77829 in the Philippines (telex country code 758)? a) 75877829+ b) 87375877829
c) 75877829

d) 58375877829
B If your vessel is in the AOR-E, which of the following is a correctly formatted Inmarsat-C address book entry for sending communications to a shoreside telex terminal number 776424 in Canada via the TWX system (telex country code 26)? a) 58126776424
b) 26776424
c) 582776424

d) 26776424+
A Which of the following is a correctly formatted Inmarsat-C address book entry for sending communications to a fax machine on a vessel in the AOR-W? a) 874336837925 b) 874436871225 c) 5841500292 d) 1 8741500292
B Which of the following is a correctly formatted Inmarsat-C address book entry for sending communications to a fax machine number 516-229-4339 in Long Beach, CA, U.S.? a) 015162294339#

b) 15162294339
c) 015162294339
d) 1 516-229-4339
D Which of the following is a correctly formatted Inmarsat-C address book entry for sending communications to a fax machine on a vessel in the AOR-E? a) 581366269025
b) 871466269025
c) 5811504338

d) 8711504338
B Which of the following is a correctly formatted Inmarsat-C address book entry for sending communications to a fax machine number 1424-8821-902 in the United Kingdom (voice country code 44)? a) 44 1424-8821-902 b) 4414248821902 c) 4414248821902# d) 44+ 14248821902+
D Which of the following is a correctly formatted Inmarsat-C address book entry for sending communications to a fax machine on a vessel in the IOR? a) 5831509987 b) 583442519372 c) 1873442519372 d) 8731509987
C Which of the following is a correctly formatted Inmarsat-C address book entry for sending communications to a fax machine number (045) 334-5678 in Japan (voice country code 81)? a) 810453345678# b) 81(045)3345678 c) 810453345678 d) 81 (045) 334-5678
A Which statement is false regarding a distress request? Any distress request is automatically switched to an Inmarsat distress working frequency. If all satellite channels are busy, one of them will be preempted by a distress request. The NCS in each ocean region automatically monitors the processing of such calls by other CESs in that region, and processes calls if any anomaly exists in the system. Any request message with distress priority is automatically recognized by the CES and a satellite channel is instantly assigned.
C How is a distress message normally initiated through Inmarsat? All Inmarsat units have a dedicated key that can be pressed for immediate action. By adding the word "DISTRESS" in the first line of the message's preamble. Certain Inmarsat units have a dedicated key that can be pressed for immediate action, while other systems provide menu-driven features. By transmitting the distress message on the U.S. Coast Guard's dedicated monitoring channel.
B How is a distress priority message ordinarily initiated on board the vessel? By dialing the correct code on the telephone remote unit. By pressing one or more dedicated "distress key/s" on the equipment. By contacting the CES operator, and announcing a distress condition is in existence. By contacting the CES operator using the radiotelephone distress procedure "Mayday... etc.
B What must be done to disconnect an incoming telex message when voice distress communications is desired via Inmarsat-A? Depress the "Reset" switch to disconnect incoming message. Depressing the "Distress" button automatically disconnects incoming messages. It is necessary to wait until the incoming telex message has concluded, otherwise a system fault will occur. Momentarily turn off the system power.
C Which of the following two-way communications can be made through Inmarsat without charge? A service message that advises a vessel of other ship traffic in its vicinity. SafetyNET. Distress traffic. Vessel position information when the ship's GPS fails.
A How is ship to shore transmission of a distress priority message in most SESs initiated? By provision of a "distress button" or code in the SES. By including the priority code in the preamble of the telex message. By including the priority code in the dialed number in voice communications. By initial voice contact with the Rescue Coordination Center.
B Upon receipt of SafetyNET™ messages of the distress or urgency category on the ship's EGC receiver, what action is required by the GMDSS Radio Operator? No immediate action is required as an audible tone will be generated at the beginning and end of the transmission and a paper printout of the message will be generated. Manually reset the alarm. No immediate action is required by the operator since the transmission will be automatically acknowledged by the receiving vessel. A periodic alarm tone will be heard until the radio operator prints the message from the unit's memory.
D What can be defined as the service that allows terrestrial information providers to send general information messages to pre-defined groups of subscribers? SafetyNET. COSPAS-SARSAT. InfoNET. FleetNET.
B What additional equipment provides the maximum availability for receiving SafetyNET™ broadcasts when the associated Inmarsat-C is being used for telex communications? An integrated EGC receiver with the existing Inmarsat-C equipment. A separate EGC receiver. HF SSB can be used to receive voice MSI broadcasts. Automatic switching between Inmarsat-C and EGC functions.
D What equipment is utilized to transmit Enhanced Group Calls? COSPAS satellite. HF SITOR shore stations. NAVTEX shore stations. Inmarsat satellite.
B What is the equipment arrangement that provides the maximum availability for reception of MSI broadcasts when using Inmarsat-C for telex communications? Integrating ECG receiver with the existing Inmarsat-C equipment. Separate EGC receiver. Redundancy using HF SSB to receive voice broadcasts. Automatic switching between Inmarsat-C and EGC functions.
C Which of the following statement concerning EGC configuration is false? NAVAREA selection should be monitored and appropriately updated. The originator of MSI information can specify receipt only by vessels within a specific geographical area, circular or rectangular. The originator of MSI information cannot specify receipt only by vessels within a specific geographical area, circular or rectangular. GMDSS operators generally can select additional NAVAREAS to acquire EGC messages of interest to the vessel.
D A vessel with an 18-hour ETA to the Panama Canal on a voyage from Miami loses the ability to communicate via Inmarsat. The most likely cause is? The vessel has sailed beyond the coverage area of the Southbury Shore Station. The vessel has sailed beyond the coverage area of the Eastern Atlantic satellite. The vessel has sailed beyond the coverage area of the Western Atlantic satellite. An equipment fault.
D A vessel, before transiting the Panama Canal, on a voyage from San Diego to Miami, loses the ability to communicate via Inmarsat. The most likely cause is: The vessel has sailed beyond the coverage area of the Southbury Shore Station. The vessel has sailed beyond the coverage area of the Western Atlantic satellite. The satellite orbit is beyond the usable range of the SES. The vessel has sailed beyond the coverage area of the Pacific satellite.
D What can be the visual observation of a condition whereby it may be possible to initiate a reliable telex transmission but a voice transmission may not be possible? An indication on a meter or on the display terminal of high transmit power. An indication of a very high antenna elevation on the display terminal screen. An indication that the antenna has reached its maximum travel in one direction. An indication on a meter or on the terminal of low receiver signal strength.
B What maintenance function may the holder of a GMDSS Radio Operator license perform, or supervise the performance of, on an Inmarsat-C SES? Adjust the station's EIRP. Remove stack deposits and other debris from the radome preventing degraded performance. Adjust a reference oscillator or synthesizer. None of these.
D What maintenance function may the holder of a GMDSS Radio Operator license perform, or supervise the performance of, on an Inmarsat-C SES? Painting the radome. Adjust the station's EIRP. Adjust a reference oscillator or synthesizer. Adjust any front panel controls.
A Which functions may the holder of a GMDSS Radio Operator License NOT perform on the Inmarsat-C equipment? Optimize performance by adjusting the transmitter EIRP. Maintain the antenna clear of soot, paint, etc. Logon, traffic and logoff functions. Entry of position data and selection of CES.
A What immediate remedy can be used to correct shadowing of the satellite signal by a shipboard obstruction? Change the ship's course. Relocate the mast or other obstruction. Raise the transmit power level. Increase the receiver gain.
A A vessel loses Inmarsat-A SES operation after a large course change. Which of the following could cause this? System loses gyro follow-up or a gyro system malfunction. Shadowing of the SES antenna by clouds or other weather formations. Misalignment of the shadow correction filter. None of these.
A Which statement is correct regarding a method that a vessel experiencing problems with shadowing of the Inmarsat-A SES antenna by an on-board obstruction could use to attempt reliable communications? Change course to make the communications. Change the Coast Station ID programming. Install a shadow correction filter. Switch from telex to Voice mode which requires lower signal strength for proper operation.
D Which of the following conditions can render Inmarsat-A communications impossible? An obstruction, such as a mast, causing disruption of the signal between the satellite and the SES antenna when the vessel is steering a certain course. A satellite whose signal is on a low elevation, below the horizon. Travel beyond the effective radius of the satellite. All of these.
D A vessel is experiencing problems tracking the satellite in an Inmarsat-C SES. The problem could be: Extremely heavy rain/snow storms. Shadowing caused by an obstacle, such as a mast, between the SES antenna and the satellite. The vessel is on the fringe of the coverage area of the satellite. All of these.
D Which functions may the holder of a GMDSS Radio Operator License perform on the Inmarsat-C equipment? Selection of CES. Maintain the antenna clear of soot, paint, etc. Logon, traffic and logoff functions. All of these.
B Which modes could be selected to receive vessel traffic lists from high seas shore stations: AM and VHF-FM. SSB and FEC. ARQ and FEC. VHF-FM and SSB.
A MF/HF Transceiver Power levels should be set __________. To the lowest level necessary for effective communications. To the level necessary to maximize the propagation radius. To the highest level possible so as to ensure other stations cannot "break-in" on the channel during use. Both A and C are correct.
D Which statement regarding GMDSS MF/HF Transceiver frequency set-up is true: Transmit and receive frequencies may be manually entered from the keypad. ITU channels must be recalled from a database or memory. All consoles allow both manual keypad entry and ITU channel recall from a database. Depending on the manufacturer, either or could be true.
C Which statement regarding GMDSS MF/HF Transceiver frequency set-up is true: Transmit and receive frequencies must always be manually entered from the keypad. Transmit and receive frequencies must always be recalled from a database or memory. Some consoles allow both manual keypad entry and ITU channel recall from a database or memory, while others do not. None of the above.
B To set-up the MF/HF Transceiver for a telex call to a coast station, the operator must: Select J3E mode for proper Sitor operations. Select F1B mode or J2B mode, depending on the equipment manufacturer. Select F1B/J2B modes or J3E mode, depending on whether ARQ or FEC is preferred. None of the above.
D To set-up the MF/HF Transceiver for a voice call to a coast station, the operator must: Select J3E mode for proper Sitor operations. Select F1B mode or J2B mode, depending on the equipment manufacturer. Select F1B/J2B modes or J3E mode, depending on whether FEC or ARQ is preferred. Select J3E mode for proper voice operations.
B How are paired NBDP frequencies normally used? These are normally used for FEC communications with coast radio stations. These are normally used for ARQ communications with coast radio stations. These are normally used only for distress communications to limit channel interference. These are normally used for DSC communications with coast radio stations.
C How are paired SSB frequencies normally used? These are normally used for FEC communications with coast radio stations. These are normally used for ARQ communications with coast radio stations. These are normally used for J3E communications with coast radio stations. These are normally used for DSC communications with coast radio stations.
B For general communications purposes, paired frequencies are: Normally used with private coast stations. Normally used with public coast stations. Normally used between ship stations. Normally used between private coast and ship stations.
D For general communications purposes, simplex frequencies are: Normally used between ship stations and private coast stations. Normally used with public coast stations. Normally used between ship stations. Both A and C are correct.
B An ITU simplex channel frequency assignment is defined as: Transmit and receive frequencies must be different. Transmit and receive frequencies must be identical. Transmit and receive frequencies may be different, depending on whether communications are ship-shore or ship to ship. Transmit and receive frequencies are different regardless of emission mode.
C An ITU duplex channel frequency is defined as: Transmit and receive frequencies may be different, depending on whether communications are ship-shore or ship to ship. Transmit and receive frequencies must be identical. Transmit and receive frequencies must be different. Transmit and receive frequencies may be different, depending on whether communications are ship-shore or ship to ship.
C Which of the following defines "ITU Channel 1216"? Ch-12 in the 16 MHz band. Ch-1216 in the MF band. Ch-16 in the 12 MHz band. This would indicate the 16th channel in the 12 MHz band, but Ch-1216 does not yet exist as there are currently only 15 possible channels.
A Which of the following is a valid 22-MHz ITU Channel? HF Ch-2206. VHF Ch-22. Ch-22A when used for VTS communications. Ch-70 (DSC only).
B What is meant by the term "ITU channel"? This refers to a vessel's selcall number. This refers to an internationally standardized assignment of frequency pairings for common use. This refers to VHF channels 1-28 and 60-88. None of the above.
B ITU channel 1604 would mean: Ch-16 in the 4 MHz band. Ch-4 in the 16 MHz band. Ch-1604 in the MF band. 1604 is the Channel number. It has no relevance to frequency bands.
A Which is a valid ITU Designation? Ch-1604 Ch-706 "Approved for GMDSS Stations". "Type Approved Under FCC Part 80 Rules and Regulations".
D ITU channels are: Frequency assignments specific to U.S. vessels only. VHF-FM frequencies. International Traffic Utility frequencies. An international designation of specific frequencies.
A Which statement regarding bandwidth and channel spacing is correct: Channel spacing values are a function of bandwidth values. Bandwidth values are a function of Channel spacing values. Channel spacing values are not a function of bandwidth values. Bandwidth & channel values do not vary with emission mode.
B The purpose of ITU channel spacing is: To maximize the number of voice & telex channels available. To minimize the possibility of interference from adjacent channels. To make most efficient use of the radio spectrum by using voice channels rather than telex channels. To make most efficient use of the radio spectrum by using telex channels rather than voice channels.
C Which statement regarding channel spacing and bandwidth is true? Voice bandwidth is greater than telex bandwidth and therefore Voice channel spacing values are less than telex channel spacing values. Telex bandwidth is greater than voice bandwidth and therefore telex channel spacing values are less than voice channel spacing values. Both telex bandwidth and channel spacing values are less than voice bandwidth and channel spacing values. Both telex bandwidth and channel spacing values are greater than voice bandwidth and channel spacing values.
D The proper sequence of channel spacing from narrow to widest is: SSB Voice, NBDP Sitor, VHF-FM Voice. VHF-FM Voice, NBDP Sitor, SSB-Voice. NBDP Sitor, VHF-FM Voice, SSB-Voice. NBDP Sitor, SSB-Voice, VHF-FM Voice.
A Communications with an emission of F1B/J2B would typically have a channel spacing of: 0.5 kHz 0.3 kHz 2.8 kHz 3.0 kHz
D Communications with an emission of J3E would typically have a channel spacing of: 0.5 kHz 0.3 kHz 2.8 kHz 3.0 kHz
A For RF communications, "modulation" is best defined as: The combination of information or intelligence with a carrier frequency. Using a single carrier frequency with the proper power level. Setting up the transceiver with the correct bandwidth to ensure proper communications. The combination of the received frequency and oscillator frequency in the mixer.
C For RF communications, "bandwidth" is best defined as: The modulation technique required to insure proper ITU channel spacing. The emission designation resulting from the desired modulation technique. The portion of the radio spectrum consumed by a particular emission selection and modulation technique. The portion of the radio spectrum reserved for frequency allocations by the ITU.
B In an AM signal using voice: Varying the amplitude of the carrier and employing both sidebands without the carrier. There is a constant amplitude carrier with complex upper and lower sidebands varying in amplitude and frequency. Varying only the amplitude of the carrier, depending on Double or Single-Sideband operations. Varying the amplitude of the carrier and employing both sidebands and the carrier.
B In FM communications, the information is applied to the carrier by: Varying the amplitude or the frequency of the carrier, depending on Double or Single-Sideband operations. Varying the frequency of the carrier. Varying only the frequency of the carrier, depending on Double or Single-Sideband operations. Varying the amplitude of the carrier.
B The proper sequence of emissions corresponding to the sequence AM-Voice DSB, SSB-Voice without carrier, USB-Voice with carrier and FM-Voice, is: a) A3E, H3E, J3E, F3E.
b) A3E, J3E, H3E, F3E.
c) J3E, H3E, A3E, F3E.
d) H3E, A3E, J3E, F3E.
C The proper sequence of emissions corresponding to the sequence SSB-Voice without carrier, USB-Voice with carrier, FM-Voice and SITOR TELEX is: a) H3E, J3E, F3E, F1B.
b) J3E, H3E, F1B, F3E.
c) J3E, H3E, F3E, F1B.
d) H3E, J3E, F1B, F3E.
B When placing a SSB MF/HF call to a Coast Station, you should always: Choose the closest station to ensure a quick connection. Make sure the frequency is not occupied. Tune the transmitter on another frequency. Wait until the coast station sends his Traffic List.
D How are high seas (HF) radiotelephone communications initially established between a vessel and a public correspondence station? The vessel listens for "free signals" and calls the public correspondence station on the NBDP calling channel with the strongest marker signal. The vessel calls the public correspondence station on VHF Channel 16 and the two stations then switch to the working channel. Public Correspondence Stations operate NBDP only. The vessel calls and establishes voice contact with the public correspondence station on a channel that the station is known to monitor, and the two stations then proceed with their business.
A What is the best procedure for calling another ship station using HF radiotelephone when the signals are weak but readable? On a properly selected ITU channel, give the call sign of the ship being called three times using the ICAO alphabet, and the words "this is" followed by the call sign of the ship initiating the call three times, using the ICAO alphabet, and concluding with "over." Give the name of the ship being called three times, and the words "this is" followed by the name of the ship initiating the call three times, and concluding with "over." Instruct the nearest public correspondence station to add the desired ship's call sign to the station's traffic list. Notify the local vessel traffic service control station of your intention to contact a specific vessel, and request the VTS operator place the call on channel 22A.
C What is the correct procedure for calling a coast radio station using HF radiotelephone? On a properly selected ITU channel, give the name of the coast radio station being called three times, and the words "this is" followed by the name of the ship initiating the call three times, and concluding with "over". Contact the nearest U.S.C.G. station to add the desired ship's call sign to the station's traffic list. On a properly selected ITU channel, give the call sign of the coast radio station being called three times using the ICAO alphabet, and the words "this is" followed by the call sign of the ship initiating the call three times, using the ICAO alphabet, and concluding with "over". Request the VTS operator place the call on channel 22A.
A Through which coast radio station(s) may a U.S.-flag merchant vessel communicate? Any coast radio station in the world that is licensed to provide such communications. Any coast radio station in the world that is licensed to provide such communications, but prior authorization must be obtained for a U.S.-flag merchant vessel to communicate through a non-U.S. station. The U.S. Coast Guard coordinates the communications and assigns the working channel. Any coast radio station in the world that has been commissioned to provide such communications.
A What is the best source of information to find changes or additions to the routine communications frequencies of a Commercial Radio Station? ITU List of Coast Stations Part IV. GMDSS Master Plan of Radio Stations. FCC Part 80. ITU List of Ship Stations Part VII.
D The purpose of the MF/HF DSC controller is? Providing for the formatting and transmission of outgoing DSC calls. Permitting control of transceiver operations via an interface. Providing the scanning watch receiver capability on the 6 MF/HF DSC frequencies. Both a) and b) are crucial functions of the DSC Controller.
A A "Distress Hot Key" MF/HF DSC Distress Alert: May go out on 2187.5 kHz or may go out on another DSC frequency, depending on the manufacturer. Must go out on 2187.5 kHz and 8414.5 kHz to trip DSC alarms on the mandatory MF/HF DSC watch frequencies. Must go out on 2187.5 kHz to alert the nearest vessels and coast stations. None of the above.
B A Distress Priority DSC call may be formatted and transmitted specifying and requesting: Nature of Distress, vessel position, follow-on frequency, only voice follow-on communications. Nature of Distress or alternate frequency but not both in a single call, vessel position or alternate frequency/emission but not both in a single call, voice or telex follow-up communications. Nature of Distress or alternate frequency but not both in a single call, vessel position or alternate frequency/emission but not both in a single call, only telex follow-up communications Nature of Distress, vessel position, follow-on frequency, only telex follow-on communications.
C A multi-frequency MF/HF DSC Distress Alert: Must be transmitted in ascending order of propagation radius to alert nearby ships first. May be transmitted on the mandatory MF/HF DSC watch frequencies first, and then on the others. Either or , depending on the manufacturer. May be transmitted in any order programmed by the GMDSSS operator.
D To make a call to another vessel requesting voice communications regarding important company business, the GMDSS operator should: Select Urgent priority, enter other vessel's MMSI, specify legal alternate frequency, F1B emission and transmit the properly formatted DSC call. Select Routine priority, enter other vessel's MMSI, specify legal alternate frequency, J2B emission and transmit the properly formatted DSC call. Select Routine priority, enter own vessel's MMSI, specify legal alternate frequency, J3E emission and transmit the properly formatted DSC call. None of the above.
A To make a call to another vessel requesting telex communications regarding important company business, the GMDSS operator should: Select Routine priority, enter other vessel's MMSI, specify legal alternate frequency, F1B emission and transmit the properly formatted DSC call. Select Urgent priority, enter other vessel's MMSI, specify legal alternate frequency, F1B emission and transmit the properly formatted DSC call. Select Routine priority, enter own vessel's MMSI, specify legal alternate frequency, F1B emission and transmit the properly formatted DSC call. Select Routine priority, enter other vessel's SelCall for telex specify legal alternate frequency, F1B emission and transmit the properly formatted DSC call.
A What is meant by the acronym ATOR? Automatic Telex Over Radio. AMVER Transmittals Over Radio. Amateur Telex Over Radio. None of the above.
C Which of the following acronyms refers to a communications system that is not based on some form of telex? NAVTEX. SITOR. NAVCOMSTA. NBDP.
D What does the term FEC mean in SITOR communications? Field Effect Correction Forward Error Check Forward Error Character. None of the Above.
B What term is nearly synonymous with ATOR? NAVTEX. SITOR. AMTOR. NBDP.
B The acronym SITOR stands for: Simplified Telephony Over Radio. Simplex Telex Over Radio or Simplex Teleprinter over Radio. Simplified Telex Over Radio. Simplex Telephony Over Radio.
B What does the term "ARQ" mean in SITOR operations? Automation Repeat Request. Automatic Repeat Request. Automaton Repeat Request. Automatic Request Repeat.
A Which of the following statements concerning SITOR communications is true? ARQ transmissions are made in data groups consisting of three-character blocks. ARQ transmissions are acknowledged by the Information Receiving Station only at the end of the message. ARQ communications rely upon error correction by time diversity transmission and reception. Forward error correction is an interactive mode.
C Which of the following statements concerning SITOR communications is true? FEC transmissions are made in data groups consisting of three-character blocks. FEC transmissions require a "phasing" or "handshaking" process prior to character transmission. FEC transmissions rely upon an error correction technique, which transmits each character twice. FEC transmissions rely upon parity and "repeat requests" to ensure error correction.
D Which statement best defines the SITOR acronym "ARQ"? Error correction for one-way TELEX broadcasts of weather or navigation information. Error Correction when 2 stations are in direct & phased telephone communications with each other. Either a) or b), depending on transceiver mode selection. None of the above
A Which statement best defines the SITOR acronym "FEC"? Error correction for one-way TELEX broadcasts of weather or navigation information. Error Correction when 2 stations are in direct & phased telephone communications with each other. Error Correction when 2 stations are in direct & phased TELEX communications with each other. None of the above
D The purpose of "parity" in SITOR communications is? Error correction in FEC weather and navigational broadcasts. Error correction in ARQ communications. Error correction in NAVTEX broadcasts. All of the above.
B "SITOR" communications are based on a digital code system: Consisting of 7 bits, with four ones and three zeros. Consisting of 7 bits, with four zeros and three ones. Consisting of 5 bits TELEX and 8 bits Fax. Consisting of 5 bits for ship stations and 7 bits for shore stations.
A Which of the following would be a valid selcall for use in ARQ communications? 1106 212420 WHAQ X. Four marks (ones) and three spaces (zeroes) forming the binary signal "1001101". This is established by the communications protocol used with the modem.
C Once ARQ communication with the coast radio station has been established, which of the following exchanges will most likely take place? The vessel then requests the coast radio station's selcall so that communication can be set up on the appropriate working channel. Since communication has already shifted to the working channel, the vessel then transmits the subscriber number and text of the message to be sent for the coast radio station to store and forward. After exchanging answerbacks with the vessel, the coast radio station transmits "GA+". None of the above.
C Which of the following methods will give a GMDSS Radio Operator the best indication of whether ARQ communication can be established with a coast radio station? Referring to propagation charts will tell the Operator when the eruption of communication shattering solar flares will occur. Selecting a frequency in the MF band averts interference from severe static discharges. Monitor the coast radio station's "free signals" and call on the frequency on which the loudest and most consistent signals are heard. Re-position the radome's antenna toward the coast radio station and press the "call request" button.
B When placing a telex call to a Coast Station, you should always: Choose the closest station. Make sure the frequency is not occupied with normal traffic. Tune the transmitter on another frequency. Wait until the coast station sends his Traffic List.
B What is the best method for a GMDSS Radio Operator to determine which SITOR station to contact for the purpose of sending a chargeable message or cable? Listen to each station's voice announcement and determine which channel(s) will be monitored. Listen to each station's free signals, and call the station generating the loudest free signal marker. Listen to each station's MSI broadcast to determine which public correspondence station to contact. Listen to the U.S. Coast Guard's traffic list to determine which Coast Guard station will handle commercial traffic.
A Which statement is correct regarding HF NBDP under GMDSS? Safety communications by direct-printing telegraphy should be in the ARQ mode when communicating with the U.S. Coast Guard or other coast stations on channels that they normally guard. Distress communications other than directly to the Coast Guard or other coast stations on the channels that they normally guard, should be in the broadcast SSB mode. The ARQ mode may be used subsequently to the FEC mode when it is advantageous to do so. Distress communications other than directly to the Coast Guard or other coast stations on the channels that they normally guard, should be in the broadcast FEC or SSB mode.
C After establishing contact with a Coast Station on an available channel, which of the following exchanges take place? The vessel will request the shore station's answerback to confirm it has reached the correct station, then the shore station operator will request billing authorization. The ship station begins transmitting the text. The coast station transmits its answerback, asks for the ship's answer back and then transmits GA+. None of these.
B During Sitor ARQ operations through a coast station, what should the GMDSS operator do during the "Automatic Exchange of Answerbacks." Send a "WRU" then a "Here is" when requested by the coast station. Do nothing other than wait for a GA+ prompt. Send the necessary message file and then wait for time and charges. None of the above.
B Assuming sending a direct telex message to a shore-based office, which sequence of events best describes a complete ARQ telex exchange with a coast station? Initiate call, observe phasing, Automatic Exchange of Answerbacks, send message, send KKKK to terminate the telex link, receive MRN & Time and charges. Initiate call, observe phasing, Automatic Exchange of Answerbacks, send DIRTLX command with zero then telex country code, telex number, send message, send KKKK to terminate the telex link, receive MRN & Time and charges. Initiate call, observe Automatic Exchange of Answerbacks, send message, send KKKK to terminate radio link, receive MRN & Time and charges. Initiate call, observe phasing, Automatic Exchange of Answerbacks, send message, send KKKK to terminate radio link, receive MRN & Time and charges.
C For ARQ communications with a Public Correspondence Coast Station, which sequence of events best describes reaching the point in time where the text of telex communications should be sent __________. Transceiver setup, SelCall selection, Initiate Call, Automatic Exchange of Answerbacks, OPR+, operator entry of the appropriate automatic telex code. Transceiver setup, Automatic Exchange of Answerbacks, OPR+, operator entry of the appropriate automatic telex code. Transceiver setup, SelCall selection, Initiate Call, Exchange of Station Answerbacks, GA+?, enter DIRTLX xy+, MOM, MSG+? and exchange of terminal answerbacks. Transceiver setup, Automatic Exchange of Answerbacks, GA+, operator entry of the appropriate automatic telex code.
C During ARQ communications, A Coast station will likely break the phased radio connection: If the error percentage of repeat requests becomes too high. If the automatic exchange of answerbacks is interrupted by keyboard entries. If either a) or b) takes place If the operator enters "KKKK."
D During ARQ communications, A Coast station will likely break the phased radio connection: If the error percentage of repeat requests becomes too high. If the automatic exchange of answerbacks is interrupted by keyboard entries. If the operator enters "BRK+." If any of the above actions takes place.
C Which of the following keystrokes or characters is sent as part of ARQ communications to end the radio link? NNNN KKKK BRK+ . . . . .
B Which of the following keystrokes or characters is sent as part of ARQ communications to signal the end of communications with a land based TELEX terminal? NNNN KKKK BRK+ . . . . .
A What character(s) are transmitted to switch control from a station currently transmitting, over to the receiving station during ATOR communications in the ARQ mode? a) +?
b) +
c) ENTER
d) END
A Which character(s) must follow most command codes to a coast station using ATOR? + # * GA+?
B Which character(s) are sent by the shore station to indicate it has shifted control to, and is awaiting instructions from, the vessel? WRU GA+? KKKK . . . . .
A All country codes which are transmitted by MF/HF SITOR are preceded by which character(s)? 0 00 + DIR
A Which of the following keystrokes or characters is sent as part of ARQ communications to switch information transmission control from one station to the other? The plus and question mark keys ("+", "?"). The go-ahead ("GA") command. The "ENTER" key. The "END" key.
A Which of the following keystrokes or characters follows most commands in ARQ communications when working an automated Coast Station? The plus ("+") key. The go-ahead ("GA") command. The "ENTER" key. The "END" key.
C Which characters are sent by the ship station when operating HF telex, to indicate a desire to send an AMVER message? MSG+ DIRTLX+ AMV+ OPR+
B Which characters are sent by the ship station to indicate a desire to send a message via a direct connection to a shoreside telex subscriber? MSG+ DIRTLX0xxyyyy+ AMV+ OPR+
C What is the procedure used to terminate the radio circuit? The END command. CONTROL, ALT AND DELETE keys. BRK+. Shut off the telex printer.
B What are the characters that are transmitted to terminate a direct telex connection in SITOR operation? ". . . ." KKKK END+ EOM
C Which of the following statements concerning SITOR communications is true? In ARQ, each character is transmitted twice, about 250 milliseconds apart. In ARQ, the "information sending station" transmits a block of three characters twice, about 250 milliseconds apart. In ARQ, the "information sending station" will transmit a block of three characters that the receiving station will subsequently acknowledge or request be retransmitted. SITOR communications can be used to contact a NAVTEX transmitting station when requesting a repeat transmission of a missed NAVTEX message.
C What statement is true regarding the exchange between two parties engaged in SITOR communications? In ARQ, each character is transmitted twice, with the second displaced in time from the first. In ARQ, the "sending" station transmits a block of three characters and the "receiving" station responds with a one character acknowledgement. Following this the "transmitting" station will send a new block. In ARQ, the "sending" station transmits a block of three characters and the "receiving" station responds with either a one-character acknowledgement of the receipt of a valid block or a negative acknowledgement indicating that the block should be re-sent. Following this the "transmitting" station will send a new block or retransmit the previous block, as required. Broadcasts of Marine Safety Information, traffic lists, etc. can be copied by the receiving station in ARQ mode.
A Of the following, which is true of SITOR ARQ mode direct printing radioteletype transmission? Each data block consists of three characters. The acceptance code consists of three characters. A continuous data stream is transmitted. Forward error correction reduces the number of errors.
A Which of the following is true of SITOR ARQ mode? This is an interactive mode. Each character is repeated three times. Each character is transmitted twice. This mode is generally used to broadcast messages.
A What should happen when an RQ code is received by a SITOR station which is sending message traffic? The last block will be resent. The next block will be sent. The data link will break. The acknowledge light should illuminate.
D Which of the following is true of Sitor ARQ mode: The ship station sends a group of 3 characters twice and then waits for an "RQ" signal to indicate proper receipt before continuing transmission. The Ship station sends each character twice, using a Time Diversity system to ensure proper parity. The ship station sends a group of 3 characters, the shore station checks for proper parity and then requests the same group be resent to enable error correction. The ship station sends a group of 3 characters, the shore station checks for proper parity. If parity is ok, the shore station indicates readiness for transmission of the next 3 characters.
D Which of the following keystrokes or characters is sent as part of ARQ communications to request a direct telex call to a shore-based location? "MSG+" "GA+" "ENTER" "DIRTLX023123456+"
D The DIRTLX command should be given to a coast station at what point in time? Immediately following the Automatic Exchange of Answerbacks. After receipt of the Go Ahead indication followed by the Automatic Exchange of Answerbacks. Sent at the beginning of the message to request a direct telex connection. After receipt of the Go Ahead indication following the Automatic Exchange of Answerbacks.
B Which Automatic telex command should be sent by a ship station during Sitor operations to properly address a weather report to the national weather authority? WX+ followed by the text of the report. OBS+ OBS+ then MSG+ to indicate weather report. GA+ then OBS+.
A Which of the following would be a valid automatic telex code and number for a request for a real-time telex connection to a shore-based telex terminal? DIRTLX023424998+. DIRTLX then waits for MSG+ to dial 023419645+. DIRTLX23122445+. DIRTLX then after GA+ then sends telex number.
B When requesting a direct telex connection to a vessel's Inmarsat-A terminal, (i.e. making a sitor call to an Inmarsat-A unit) the GMDSS operator must: DIRTLX, 00, Ocean Area Code, telex number, +. DIRTLX, 0, Ocean Area Code, telex number, +. DIRTLX, 0, Country code, telex number, +. None of the above.
A To correctly address a Sitor message and transmit it "live" to a shore-based office the GMDSS operator would dial which sequence: DIRTLX051440344+ DIRTLX0051440344+ DIRTLX581440344+ DIRTLX05811500260+
B Which of the following statements concerning SITOR communications is true? Communication is established on the working channel and answerbacks are exchanged before FEC broadcasts can be received. Two-way communication with the coast radio station using FEC is not necessary to be able to receive the broadcasts. Weather broadcasts cannot be made in FEC because sending each character twice would cause the broadcast to be prohibitively long. None of the above.
D Which of the following is true of SITOR FEC in the presence of impulse-type interference? Data flow rate depends on signal propagation. Idle characters are sent upon request. Transmitter and receiver cannot synchronize. The error rate is reduced by time diversity reception.
B Which of the following statements concerning SITOR communications is true? FEC requests are first acknowledged by the vessel's transmitter before broadcasts can be received. FEC mode broadcasts can be received without the transmitter being turned on. Weather broadcasts are always made in ARQ mode to ensure reception. None of these.
C The sequence ARQ, FEC, SFEC best corresponds to which of the following sequences? One-way communications to a single station, one-way communications to all stations, two-way communications. One-way communications to all stations, two-way communications, one-way communications to a single station. Two way communications, one-way communications to all stations, one-way communications to a single station. Two way communications, One way communications to a single station, One way communications to all stations.
D The sequence BFEC, SFEC, ARQ best corresponds to which of the following sequences? One-way communications to a single station, one-way communications to all stations, two-way communications. One-way communications to all stations, two way communications, one-way communications to a single station. Two way communications, one-way communications to all stations, two-way communications. None of the above.
C Selective FEC communications (SFEC) are employed when: Multiple stations without a group selcall must receive communications without using their transmitters (Radio Silence). Multiple stations must receive communications by using their transmitters to achieve phasing. An individual station must receive communications without using any transmitters (Radio Silence). An individual station must receive communications by using their transmitter to achieve phasing and block other stations from breaking in.
D If the vessel is within range of NAVTEX broadcasts and both the Inmarsat-C and the NAVTEX receiver are inoperative the GMDSS operator should: Select 518 kHz ARQ telex on the MF/HF console to receive MSI. Request repairs of the Sat-C system and wait until within range of NAVTEX. Select an HF MSI frequency and ARQ telex mode to receive MSI. Select 518 kHz FEC telex on the MF/HF console to receive MSI.
C If the vessel is beyond range of NAVTEX broadcasts and the Sat-C system fails, the GMDSS operator must: Select 518 kHz ARQ telex on the MF/HF console to receive MSI. Request repairs of the Sat-C system and wait until within range of NAVTEX. Select an HF MSI frequency and FEC telex mode to receive MSI. Select 518 kHz FEC telex on the MF/HF console to receive MSI.
C If the vessel is experiencing atmospheric interference with NAVTEX broadcasts, especially in the tropics, the GMDSS operator should: Select one of the 6 HF MSI frequencies and set-up the transceiver in ARQ telex mode. Select one of the 6 MF MSI frequencies and set-up the transceiver in FEC telex mode. Select one of the 8 HF MSI frequencies and set-up the transceiver in FEC telex mode. Select the MF MSI frequency dedicated to tropical MSI and set-up the transceiver in FEC telex mode.
D Why must the GMDSS operator be able to set-up FEC & ARQ modes and differentiate between them? The proper mode must be selected for reception of HF MSI. The proper mode must be selected for follow-on telex communications in a distress situation. The ARQ telex mode must be selected for follow-on telex communications in a distress situation and the FEC telex mode selected for reception of HF MSI. Both a) and b) are correct
D FEC Sitor transmissions might be used to? Receive Coast station traffic lists, NAVTEX and VHF MSI broadcasts. Send and receive distress telex communications, receive HF MSI and NAVTEX. Receive weather messages or Coast Station traffic lists. Either b or c might be possible.
B FEC NBDP transmissions are normally used to? Receive Coast station traffic lists, NAVTEX and VHF MSI broadcasts. Receive HF MSI and NAVTEX. Receive weather messages, Coast Station traffic lists and company messages. None of the above.
C The "short rules" of propagation necessary to select the appropriate frequency band are: Shorter Distance = Higher Frequency, Daytime = Higher Frequency. Shorter Distance = Higher Frequency, Daytime = Lower Frequency. Shorter Distance = Lower Frequency, Daytime = Higher Frequency. Shorter Distance = Lower Frequency, Daytime = Lower Frequency.
C GMDSS operators should learn which of the following propagation "rules of thumb"? Longer distance = lower frequency, Shorter Distance = higher frequency; Daytime = higher frequency, nighttime = lower frequency. Longer distance = lower frequency, Shorter Distance = lower frequency, Daytime = lower frequency, nighttime = higher frequency. Longer distance = higher frequency, Shorter Distance = lower frequency, Daytime = higher frequency, nighttime = lower frequency. Longer distance = higher frequency, Shorter Distance = higher frequency, Daytime = lower frequency, nighttime = higher frequency.
A Which of the following is a source of solar flux conditions that can be used with propagation software as an aid in predicting the best frequency band to use for HF radio communications? Hourly announcements from WWV and WWVH. Hourly forecast broadcasts from CHU. Every public correspondence station provides propagation forecasting services. Doppler weather satellite broadcasts.
A GMDSS operators should routinely focus on the factors affecting propagation in what priority? Distance & Time of Day, Seasonal variations, Sunspot cycle, Solar flare alerts. Sunspot cycle, Distance & Time of Day, Seasonal variations, Solar flare Alerts. Solar flare alerts, Distance & Time of Day, Sunspot cycle, Seasonal variations. Solar flare alerts, Sunspot cycle, Seasonal variations, Distance & Time of Day.
C Which statement concerning frequency band selection and propagation most corresponds with standard practice. Maximum Usable Frequency (MUF) and Optimum Usable Frequency (OUF) should be calculated prior to setting up the transceiver. Propagation rules, Sunspot cycle status and seasonal variations (winter & summer) must always be employed to calculate the proper band selection. Propagation "rules of thumb" are usually sufficient for routine operations. Propagation "rules of thumb" should be combined with Solar flare alerts to determine band selection.
D To ensure effective communications, GMDSS operators should: Rely on the equipment calculations of Optimum Usable Frequency. Employ the short rules of propagation selection. Rely on previous successful communications on the selected frequency band. Taken together, both b and c are good operational practices.
C A ship at anchor has been communicating effectively with a shore station approximately 500 miles (805 km) distant on a frequency in the 16 MHz band periodically throughout the day. Toward the late afternoon and evening, what effect should be noticed? Communications should be maintained with slight improvement in the signal received from the shore station. The gray line effect will prevent communications after dark. Communications should gradually deteriorate and become impossible on this frequency at night. Communications should improve and peak at night.
D A ship at anchor has been communicating marginally with a shore station approximately 200 miles (322 km) distant on a frequency in the 4 MHz band periodically throughout the day. Toward the late afternoon and evening, what effect should be noticed? Communications should be maintained with slight improvement in the signal received from the shore station. Communications should slowly deteriorate but may be continued throughout the night. Communications should gradually deteriorate and become impossible on this frequency by night. Communications should gradually improve and peak at night and early morning.
B At mid-day in the summer, what would be the best choice in attempting to communicate, using NBDP with a shore station some 1800 miles distant? VHF-FM. Higher HF bands. Lower HF bands. MF
C At mid-night, what would be the best choice in attempting to communicate, using NBDP with a shore station some 800 miles distant? VHF-FM. Higher UHF bands. HF bands. Communications is impossible at this distance.
D At mid-day, what would be the best choice in attempting to communicate with a shore station some 75 miles distant? VHF-FM. 22 MHz band. 16 MHz band. MF
C How can a GMDSS operator determine the best frequency band to choose for a SITOR transmission to a shore station? By consulting propagation tables. If static interference is present, try lower bands first. Listen to shore station "free signals"; choose the band with the strongest. During nighttime, choose highest frequencies. Choose lower frequencies in the daytime.
C The Distress Alarm sounds and the screen readout no longer contains the particulars of the Distress: Examine the printer to determine if the particulars were routed to the printer. Examine the Transmit Data Directory. Both a) and b) are correct. Examine the Received Data Directory.
D A DSC Distress call is received by your vessel and your transceiver frequency display reads: Transmit = 4207.5 kHz and Receive = 4207.5 kHz -- what information can you infer from this? The DSC controller decoded the requested voice frequency as 4207.5 kHz Simplex and your DSC controller has automatically set-up your transceiver. The DSC controller decoded the contents of the DSC call but the request is illegal. Both a) and b) are true. The DSC call came in on 4 MHz DSC and you should set-up your transmitter and respond on the appropriate voice follow-on frequency.
C You receive an Urgent DSC call to all vessels specifying an alternate telex frequency & emission. If your transceiver frequency display reads: 2174.5 kHz transmit & 4207.5 kHz receive: Either the call was incorrectly formatted by the other vessel or the other vessel has presumably lost MF/HF voice capability. The DSC controller decoded the contents of the DSC call but the request is illegal. Both a) and b) are true. The DSC call came in on 4 MHz DSC but either the call was incorrectly formatted by the other vessel or your controller failed to decode the receive field of the alternate frequency entry and only your Transmit set-up is correct.
C You receive a Routine DSC call specifying an alternate working voice frequency & emission. Your transceiver frequency display reads: 4125.0 kHz transmit 4125.0 kHz receive: The requested alternate working channel was 4125.0 kHz Simplex and your DSC controller has automatically set-up your transceiver. The DSC controller decoded the contents of the DSC call and has automatically set-up your transceiver but the request is illegal. Both a) and b) are true. The DSC call came in on 4125 kHz and you should respond on the alternate working frequency.
D You receive a Routine DSC call to your vessel, without specifying an alternate working voice frequency & emission. If your transceiver frequency display reads: 2177 kHz transmit 2177 kHz receive what must you do? Tune the transmitter and make a voice call to the other vessel on 2187.5 kHz. Access the data directory to determine the alternate frequency and then make a voice call to the other vessel on the alternate frequency. Manually Acknowledge the DSC call on the alternate frequency, then make a voice call to the other vessel on 2182.0 kHz. Call other vessel on 2182.0 kHz and shift to a proper working frequency.
D An incoming DSC Distress Alert on 8414.5 kHz will have what result? The DSC controller will emit both an audible and visual alarm. The particulars of the alert may be printed out. The distress information contained in the alert will be sent to the data directory. All of the above.
A What is the normal voltage of a single lead acid battery cell? 2.1 volts 1.5 volts 2.5 volts 1.2 volts
C What is the normal specific gravity of a fully charged lead acid battery cell? 1.375 1.18 1.28 1.21
B What instrument is used for measuring the electrolyte of a lead acid battery? Hygrometer Hydrometer ph Meter Manometer
C What will cause an individual battery cell to reverse polarity. High discharge rates without allowing for a cool down period. Insufficient charging which does not bring all of the cells up to full charge. When discharging the battery string if a cell becomes weaker than the remaining cells, the discharge current will effectively charge the weaker cell in reverse polarity. None of the above
D What is the effect of temperature on the specific gravity of lead acid batteries? Higher temperature results in a higher specific gravity reading. Temperature has no effect on the specific gravity reading. Lower temperature results in a lower specific gravity reading. Higher temperature results in a lower specific gravity reading.
A What would be an indication of a malfunction on a GMDSS station with a 24 VDC battery system? A constant 30 volt reading on the GMDSS console voltmeter. After testing the station on battery power, the ammeter reading indicates a high rate of charge that then declines. After testing the station on battery power, a voltmeter reading of 30 volts for brief period followed by a steady 26 volt reading. None of the above
B Which of the following conditions would be a symptom of malfunction in a 2182 kHz radiotelephone system which must be reported to an onboard or at-sea maintainer under GMDSS? Much higher noise level observed during daytime operation. No indication of power output when speaking into the microphone. When testing a radiotelephone alarm on 2182 kHz into an artificial antenna, the distress frequency watch receiver becomes unmuted, an improper testing procedure. Failure to contact a shore station 600 nautical miles distant during daytime operation.
B Which would indicate a malfunction in a 2182 kHz radiotelephone system? No discernable traffic has been heard on the 2182 kHz during the radiotelephone silent periods. Failure to contact another station 60 miles distant during daytime operation. Dramatic decrease in noise level observed during night and early morning hours. The visual indication of power to the antenna fluctuates while testing the radiotelephone alarm signal generator into an artificial antenna.
B Which would indicate proper operation of a SSB transmitter rated at 60 Watt PEP output in J3E mode? In SITOR communications, the power meter can be seen fluctuating regularly from zero to the 60 watt relative output reading. In SSB (J3E) voice mode, with the transmitter keyed but without speaking into the microphone, no power output is indicated. In SSB (J3E) mode, speaking into the microphone causes the power meter to fluctuate well above the 60 watt reading. A steady indication of transmitted energy on an RF power meter with no fluctuations when speaking into the microphone.
A Which would be an indication of proper operation of a SSB transmitter rated at 60 watt PEP output? In Sitor communications, the power meter can be seen fluctuating regularly from zero to the 60 watt relative output reading. In SSB (J3E) voice mode, with the transmitter keyed but without speaking into the microphone, power output is indicated. In SSB (J3E) mode, speaking into the microphone causes power meter to fluctuate slightly around the 60 watt reading. A steady indication of transmitted energy on an RF Power meter with no fluctuations when speaking into the microphone.
D There are no signals or static heard in the receiver of the SSB Transceiver, although a loud noise is heard if the volume control is advanced all the way up, what is a possible cause? The antenna or feedline has been damaged. The antenna switch has been transferred to the test position. A defective receiver. Any of these.
A What would be an indication of a malfunction on a GMDSS station with a 24 VDC battery system? A constant 30 volt reading on the GMDSS console voltmeter. After testing the station on battery power, the ammeter reading indicates a high rate of charge that then declines. After testing the station on battery power, a voltmeter reading of 30 volts for brief period followed by a steady 26 volt reading. None of the above.